Categorised Module MCQs

You might also like

Download as pdf or txt
Download as pdf or txt
You are on page 1of 177

Categorized Final MBBS Common

MCQs (2008 A/L – 2012 A/L)


CATEGORIZED ACCORDING TO THE MODULES

Project by,
MFSU/MSWS
Content
Page No.
Content .............................................................................................................................................................................. i
1. E&M Module – Final MBBS Common MCQs ................................................................................................................. 1
Medicine – E&M............................................................................................................................................................ 1
Surgery – E&M .............................................................................................................................................................. 4
Paediatrics – E&M ......................................................................................................................................................... 6
2. IPD Module - Final MBBS Common MCQs .................................................................................................................... 9
Medicine – IPD .............................................................................................................................................................. 9
Surgery – IPD ............................................................................................................................................................... 15
Paediatrics – IPD ......................................................................................................................................................... 16
3. CVS Module - Final MBBS Common MCQs ................................................................................................................. 19
Medicine – CVS ........................................................................................................................................................... 19
Surgery – CVS .............................................................................................................................................................. 23
Paediatrics – CVS ......................................................................................................................................................... 26
4. RS Module - Final MBBS Common MCQs.................................................................................................................... 29
Medicine – RS.............................................................................................................................................................. 29
Surgery – RS ................................................................................................................................................................ 33
Paediatric – RS............................................................................................................................................................. 35
5. NGD Module - Final MBBS Common MCQs ................................................................................................................ 41
Paediatrics – Nutrition ................................................................................................................................................ 41
Paediatrics – Growth................................................................................................................................................... 43
Paediatrics – Development ......................................................................................................................................... 45
6. GIT Module - Final MBBS Common MCQs .................................................................................................................. 47
Medicine – GIT ............................................................................................................................................................ 47
Surgery – GIT ............................................................................................................................................................... 49
Paediatrics – GIT ......................................................................................................................................................... 63
7. BLS Module - Final MBBS Common MCQs .................................................................................................................. 67
Medicine – BLS ............................................................................................................................................................ 67
Surgery – BLS............................................................................................................................................................... 68
Paediatrics – BLS ......................................................................................................................................................... 69
8. Nephro-Urology Module - Final MBBS Common MCQs ............................................................................................. 74
Medicine – NU ............................................................................................................................................................ 74
Surgery - NU ................................................................................................................................................................ 77
Paediatrics – NU .......................................................................................................................................................... 80
9. Neurology Module - Final MBBS Common MCQs ....................................................................................................... 82
Medicine – Neurology ................................................................................................................................................. 82
Surgery – Neurology (Neurosurgery) .......................................................................................................................... 85
Paediatrics – Neurology .............................................................................................................................................. 86
10. Special Senses Module - Final MBBS Common MCQs .............................................................................................. 90
Medicine – Special Senses .......................................................................................................................................... 90
Surgery – Special Senses ............................................................................................................................................. 91
Paediatrics – Special Senses ........................................................................................................................................ 92
11. Mental Health Module - Final MBBS Common MCQs .............................................................................................. 96
Psychopathology and MSE .......................................................................................................................................... 96
Bipolar Affective Disorder ........................................................................................................................................... 97
Depressive Disorders .................................................................................................................................................. 97
Schizophrenia and Other Delusional Disorders ........................................................................................................ 100
Child Psychiatry ......................................................................................................................................................... 101

i
Dementia, Delirium and Psychiatric Emergencies .................................................................................................... 103
Anxiety Disorders ...................................................................................................................................................... 104
Suicide and Psychoactive Substance Use .................................................................................................................. 106
Stress related, Personality, Somatoform Disorders .................................................................................................. 108
Disorders of Eating and Sex ...................................................................................................................................... 110
Psychiatry and Law.................................................................................................................................................... 111
Epilepsy in Psychiatry ................................................................................................................................................ 111
Clinical use of Psychotropics ..................................................................................................................................... 112
12. ML Module - Final MBBS Common MCQs .............................................................................................................. 114
13. MSK Module - Final MBBS Common MCQs ............................................................................................................ 115
Medicine – MSK (Rheumatology) ............................................................................................................................. 115
Surgery – MSK (Orthopaedic Surgery) ...................................................................................................................... 116
Paediatrics – MSK...................................................................................................................................................... 118
14. Trauma Module - Final MBBS Common MCQs ....................................................................................................... 120
Surgery – Trauma ...................................................................................................................................................... 120
15. Reproductive Health Module - Final MBBS Common MCQs .................................................................................. 125
Obstetrics – Reproductive Health ................................................................................................................................. 125
Normal and Abnormal Labour .................................................................................................................................. 125
Foetal Distress ........................................................................................................................................................... 126
Instrumental Delivery ............................................................................................................................................... 127
Induction of Labour ................................................................................................................................................... 127
Postpartum Haemorrhage and Obstetric Emergencies ............................................................................................ 128
Antepartum Haemorrhage........................................................................................................................................ 130
Malpresentations and Malposition ........................................................................................................................... 130
Twin Pregnancy ......................................................................................................................................................... 133
Medical Disorders ..................................................................................................................................................... 133
Foetal Growth Restriction ......................................................................................................................................... 135
Deep Vein Thrombosis .............................................................................................................................................. 137
Iron Deficiency Anaemia ........................................................................................................................................... 137
Hyperemesis Gravidarum ......................................................................................................................................... 137
Perinatal Infections ................................................................................................................................................... 138
Prenatal Diagnosis..................................................................................................................................................... 139
Rhesus Isoimmunisation ........................................................................................................................................... 139
Caesarean Section ..................................................................................................................................................... 140
Preterm Labour ......................................................................................................................................................... 140
Prescribing in Pregnancy ........................................................................................................................................... 141
Antenatal Care .......................................................................................................................................................... 142
Gynaecology – Reproductive Health............................................................................................................................. 144
Uterovaginal Prolapse and Urinary Incontinence ..................................................................................................... 144
Pelvic Infections ........................................................................................................................................................ 145
Primary and Secondary Amenorrhoea...................................................................................................................... 146
Abnormal Uterine Bleeding ...................................................................................................................................... 147
Post-Operative Complications .................................................................................................................................. 149
Contraception and subfertility .................................................................................................................................. 150
First Trimester Bleeding ............................................................................................................................................ 154
Cervical Intraepithelial Neoplasia and Cervical Carcinoma ...................................................................................... 156
Benign and Malignant Ovarian Tumours .................................................................................................................. 156
Endometrial Carcinoma and Fibroids........................................................................................................................ 158
Menopause and Hormone Replacement Therapy .................................................................................................... 159
Endometriosis ........................................................................................................................................................... 160
Other ......................................................................................................................................................................... 161
Surgery – Reproductive Health ..................................................................................................................................... 162
ii
Breast Diseases ......................................................................................................................................................... 162
Neonatology .................................................................................................................................................................. 165
New-born Assessment and Prematurity ................................................................................................................... 165
Infections in New-born ............................................................................................................................................. 166
Neonatal Resuscitation ............................................................................................................................................. 166
Seizures in New-Born ................................................................................................................................................ 167
Respiratory Distress in New-Born ............................................................................................................................. 167
Jaundice in New-Born ............................................................................................................................................... 168
16. Therapeutics and Toxicology Module – Final MBBS Common MCQs ..................................................................... 170
Medicine – Toxicology .............................................................................................................................................. 170

iii
This Categorized Final MBBS Common MCQ book was prepared with the contribution of the following students –
Avishka, Harshana Arandara, Pushpika Chathuranga (2014 A/L Batch); Pasan Madusanka, Heshani, Sachinthana,
Jeremy, Thakshila, Anjalee, Tishothman, Janani, Indumini, Wathsala, Thanujaa, Chethaka, Samadhi, Vismi, Nipuni,
Gayani, Chamath Lavinda (2015 A/L Batch); Nisal, Nathasha, Madushanka, Gajaba, Dilshan, Amantha, Chulakshi,
Priyath, Dulani, Induwara, Afrah, Tharindu, Devni, Sanduni, Sajith, Hasini (2016 A/L Batch) – under the guidance of
MFSU/MSWS, Faculty of Medicine, University of Colombo.
ALL THE BEST FOR THE FINAL MBBS EXAM!

iv
1. E&M Module – Final MBBS Common MCQs
Medicine – E&M
1. Regarding GLP (Glucagon Like Peptide) (2009.MED.MCQ.18)
A) It is an incretin
B) Produce by pancreas
C) Increases insulin secretion
D) Metabolized by DPP-4
E) GLP-1 agonists can be used to treat hypoglycaemia
2. 58 years old male with type 2 diabetes mellitus, whose BMI is 30.4 and BP is 150/100mmHg has poor glycaemic
control. His ejection fraction is 25%, FBS level is 165 mg/dl, HbA1c is 8.9, S.Cr is 3.1 mmol/L, eGFR is 25ml/mg.
What are the drugs that he can use? (2010.MED.MCQ.10)
A) Gliclazide
B) Tolbutamide
C) Pioglitazone
D) Metformin
E) Sitagliptin
3. Regarding thyrotoxicosis (2010.MED.MCQ.11)
A) Commonly convert to malignancy
B) Exophthalmos is characteristic in Hashimoto's thyroiditis
C) Can present with chronic diarrhoea
D) Proximal muscle weakness is a well-known feature
E) Propylthiouracil is the drug of choice in 1st trimester of pregnancy
4. Which of the following features can be seen in Hypothyroidism? (2011.MED.MCQ.09)
A) Cerebellar degeneration
B) Macrocytic anaemia
C) External ophthalmoplegia
D) Myocarditis
E) Myxoedema coma
5. Hypokalaemia is associated with (2011.MED.MCQ.10)
A) Addison’s disease
B) Renal artery stenosis
C) Adrenal tumours
D) Cushing’s syndrome
E) Thyrotoxicosis
6. Prolonged use of inhaled corticosteroids cause, (2012.MED.MCQ.09)
A) Hoarseness of voice
B) Fatal growth restriction
C) Oropharyngeal candidiasis
D) Diabetes mellitus
E) Sleep disturbances
7. 25-year-old woman presenting with a history of excessive sweating, loss of weight despite good appetite and
palpitations. O/E - diffuse enlargement of thyroid gland, pulse 120 bpm. What is the most appropriate
Investigation in initial management? (2008.MED.SBR.16)
A) Free T3
B) UST of the neck
1
C) Thyroid peroxidase Ab
D) TSH receptor antibody
E) Thyroid Technetium scan
8. 15 yr old boy presented with polyuria and weight loss. On admission RBS - 500 mg/dl, pH -7.2, HCO3 - 14
mEq/L. his blood sugar was under control after few days of hospital stay. Most appropriate management on
discharge, (2008.MED.SBR.17)
A) Metformin
B) Once a day long acting insulin
C) Glibenclamide
D) Metformin with Glibenclamide with bedtime basal insulin
E) Short acting insulin before main meals and long acting insulin at night
9. 67 yr old female who is a known patient with DM, on oral hypoglycaemic agents presented with RBS - 455
mg/dl. O/E she was dehydrated, PR- 100 bpm, BP - 100/70mmHg. Which of the following is the first step in
management? (2008.MED.SBR.18)
A) Give s/c soluble insulin
B) Give iv normal saline
C) Give 0.45% saline
D) Increase the dose of oral hypoglycaemic drugs
E) Catheterize the patient
10. A 72-year-old male on oral hypoglycaemics presents with fever for 3 days. Examination shows dehydration
and lethargy. Investigations show RBS-620 mg/dl, Na+ 150 mmol/l, K+ 5mmol/l, HCO3- - 25, pH - 7.35. What is
the best next step in the management? (2009.MED.SBR.36)
A) Soluble insulin 20 mg IM
B) 0.9% saline bolus
C) Heparinization
D) Increase oral hypoglycaemics
E) Start mixed insulin
11. A 35-year-old female presented with recent onset malaise, tremor and palpitations for 2 weeks with
tenderness of the neck. TSH <0.01 mmol/l (0.15-3.5). T - 0.35ng/dl, ESR-45mm/1 hour What is the most likely
diagnosis? (2009.MED.SBR.37)
A) Grave's disease
B) Thyrotoxic crisis
C) Subacute thyroiditis
D) Toxic multinodular goitre
E) Carcinoma of thyroid
12. A 55-year-old male driver with diabetes mellitus for 10 years as a history of myocardial infarction & LVF. He is
on metformin 500 mg tds and glibenciemide 10 mg bd. He presents with worsening nocturia. FBS 320 mg/dl.
HbA1c>10%. What is the best management option for him? (2009.MED.SBR.38)
A) Add insulin
B) Low cholesterol diet
C) Increase metformin dose
D) Add acarbose
E) Add pioglitazone
13. 56 years old male came for routine medical check-up. His HbA1c 8.5, Total Cholesterol 260, Serum creatinine
1.1. Doctor advised for dietary control and daily exercise for 3 months. What is the other measure to be
prescribed for him? (2010.MED.SBR.38)
A) Dietary control and daily exercise for another month
2
B) Repeat HbA1c after 2 months
C) Review with lipid profile
D) Start metformin
E) Start gliclazide
14. 38-year-old male diagnosed with DM for 18 years presented with progressive leg swelling and SOB. Which of
the following make diabetic nephropathy the most likely diagnosis? (2010.MED.SBR.39)
A) Proteinuria 3+
B) Leukonychia
C) Creatinine level less than 2 mg/dl
D) Proliferative retinopathy
E) .
15. A 35 years old female with recent weight gain, hirsutism, and hypertension. What is the screening test to
diagnose to have Cushing's syndrome? (2010.MED.SBR.40)
A) Random cortisol level.
B) Inferior petrosal vein cannulation.
C) Serum ACTH level.
D) Overnight Dexamethasone suppression test
E) High dose Dexamethasone suppression test
16. 60-year-old man with diabetes on premixed insulin (70/30) found unconscious at 4am.What is the initial step
of management? (2011.MED.SBR.33)
A) 10% dextrose IV
B) 50% dextrose IV
C) IM glucagon with 50% dextrose
D) IM glucagon
E) IV Glucagon
17. A 35-year-old male, strong family history of diabetes presented for a medical check-up. BP –130/90, CVS and
RS examination normal. Investigation results,
• FBS – 140
• HbA1c – 7.2%
• Total cholesterol - 220 (>200)
• LDL cholesterol – 160 (>150)
• AST – 60 (10-40)
• ALT – 80 (10-60)
• Serum creatinine – normal
What is the management? (2012.MED.SBR.01)
A) Start metformin
B) Start sitagliptin
C) Start gliclazide
D) Start Insulin
E) Review with fasting blood sugar after one month without starting medication
18. In Cushing syndrome, what feature is more suggestive of an ectopic ACTH secreting tumour?
(2012.MED.SBR.06)
A) Hyponatremia
B) Excessive cortisol
C) Glucose intolerance
D) Metabolic alkalosis
E) Refractory hypokalaemia
3
19. 24-year-old female bank officer has a weight gain of 8kg over 3 months. She also complained of lethargy and
oligomenorrhea. BP – 170/100. K+ - 3.3, Na+ - 146, TSH - normal, Hb – normal. What is the most likely cause
for her elevated blood pressure? (2012.MED.SBR.28)
A) Conn syndrome
B) Cushing syndrome
C) Hypothyroidism
D) Metabolic syndrome
E) Obstructive sleep apnoea

Surgery – E&M
1. 20-year-old woman presents with 3 months history of a goitre and hyperthyroidism. Which of the following
features support the diagnosis of Graves’ disease? (2009.SUR.MCQ.2)
A) A simultaneous appearance of hyperthyroid features with goitre
B) Reduction of the features with the enlargement of the thyroid
C) Presence of vitiligo
D) Clubbing of fingers
E) Lid lag
2. 40-year-old female with a right side solitary thyroid nodule of 4 cm; with retrosternal extension and lymph
node enlargement. Investigations indicated are, (2010.SUR.MCQ.10)
A) Hormone assessment
B) USS neck
C) Thyroid antibodies
D) FNAC
E) CT neck and chest
3. Which of the following is/are true of manifestation of RLN palsy after total thyroidectomy? (2011.SUR.MCQ.9)
A) Hoarseness of voice
B) Stridor
C) Snoring
D) .
E) .
4. 25-year-old women underwent total thyroidectomy, presented to the ward 3days after the surgery with
Carpopedal spasm. Which of the following is/are true? (2012.SUR.MCQ.9)
A) Commonest cause is accidental removal of parathyroid glands
B) Correction of Mg is indicated in this patient
C) parathyroid hormone levels should be assessed before the treatment
D) Treatment with oral calcium is sufficient in this instance
E) Recovery occurs within 6 months

5. 60yr old man who underwent hernial repair, with a mesh is readmitted with fever. He is a diabetic patient &
on oral hypoglycaemic drugs. On examination there was erythema around the surgical scar. What is the most
appropriate next step in management? (2008.SUR.SBR.07)
A) Change to insulin
B) Send blood culture
C) Start iv antibiotics
D) Remove sutures
E) Remove mesh

4
6. 30 year previously healthy male presented with episodes of headaches, generalized malaise, sweating &
palpitations. His BP is 220/110 during one episode. What is the most appropriate investigation?
(2008.SUR.SBR.15)
A) CT abdomen
B) S. Calcium level
C) USS abdomen
D) Urinary catecholamine
E) S cholesterol
7. 6 hours after undergoing a thyroidectomy, 36 years old female pt vomited and having stridor cause for the
stridor? (2008.SUR.SBR.16)
A) Laryngeal oedema
B) Billy haemorrhage
C) Diaphragmatic breathing
D) Laryngeal oedema
E) Laryngeal spasm
8. A 40yr old female was detected to have a solitary nodule in the R/ lobe of the thyroid which appeared to be
hyperactive. She was treated with carbimazole and propranolol for 4 months. Now she is euthyroid. What is
the next best step in the management? (2009.SUR.SBR.36)
A) Total thyroidectomy
B) Subtotal thyroidectomy/lobectomy
C) Right hemi thyroidectomy
D) Treat medically for 2 years
E) .
9. A 28 years old women presents with 4 cm size solitary thyroid nodule in the right lobe. FNAC reviled follicular
proliferation (Thy3). What is the most appropriate next step in the management? (2010.SUR.SBR.31)
A) Core biopsy of the nodule
B) Radioisotope scan
C) Repeat FNAC
D) Hemi thyroidectomy
E) Total thyroidectomy
10. A 53-year-old woman presents with muscle pain, malaise, constipation and back ache. Laboratory
investigations revealed hypercalcemia. What is the appropriate management for her? (2010.SUR.SBR.46)
A) Administration of thiazide diuretics
B) Calcium resonium
C) IV Bisphosphonate
D) IV Normal saline
E) Neck exploration Para thyroidectomy
11. 20-year-old girl presented with excessive sweating palpitations and anxiety. On examination diffuse goitre,
tremors. Lid lag and periorbital oedema is present. What is the most likely condition toxic adenoma?
(2011.SUR.SBR.28)
A) De Quervain thyroiditis
B) Graves’ disease
C) Toxic MNG
D) Hashimotos thyroiditis
E) .

5
12. 55-year-old women are being investigated for upper cervical lymphadenopathy. FNAC of the lymph node
shows deposits of squamous cell carcinoma. She also has thyroid goitre with a dominant nodule on the right
lobe. What is the next step in management? (2011.SUR.SBR.32)
A) Excision of the lymph node
B) Fibroptic-naso-laryngoscope (FONL)
C) FNAC of solitary nodule
D) Tonsillar biopsy
E) UGIE
13. 58-year-old female underwent renal transplant 7 weeks back presented with polyuria, polydipsia abdominal
pain. Her serum corrected calcium level was 2.8 mmol/ (2.2-2.6 mmol/L) phosphate 0,69 mmol/ (1.12 to 1.45
mmol/L). Most probable diagnosis (2011.SUR.SBR.33)
A) Multiple myeloma
B) Paget's disease
C) Primary hyperparathyroidism
D) Secondary hyperparathyroidism
E) Tertiary hyperparathyroidism
14. 70-year-old woman presenting with right sided solitary thyroid nodule which turned out to be a follicular lesion
on FNAC. Rest of the thyroid gland was normal on USS neck. Most appropriate next step of management?
(2012.SUR.SBR.33)
A) Isotope scan of the thyroid
B) Total thyroidectomy
C) Right side hemi thyroidectomy
D) Serial USS monitoring
E) Thyroglobulin levels
15. 68-year-old diabetes patient was found unresponsive in ward with no breathing movements, he was having
sinus bradycardia. What is the most important next step in management? (2012.SUR.SBR.49)
A) High flow O2
B) Measure CBS
C) Check carotid pulse
D) Intubate and ventilate
E) .

Paediatrics – E&M
1. 11 Yr old boy presented with short stature to paediatric clinic. His height for age Is < 250. Height velocity is
normal. His bone age & height age is equal, but less than chronological age What is the probable diagnosis?
(2008.PAED.MCQ.08)
A) Chronic malnutrition
B) Constitutionally small.
C) Familial
D) Hypothyroidism
E) Silver Russell dwarfism.
2. Causes for oral pigmentation, (2009.PAED.MCQ.08)
A) Addison's disease
B) Pellagra
C) Peutz-Jeghers syndrome
D) Crohn's disease
E) Fanconi anaemia

6
3. 9 yr old girl with hypothyroidism is on thyroxine 50 microgram/day. She is having constipation. Her
grandmother worried that her school performance is poor she is bullied at school for short stature. Her T4 is
2.6 (2.1-2 8) and TSH is 15 (0.5-5.0). What is the next step of her management? (2009.PAED.SBR.39)
A) Assess bone age
B) Assess compliance
C) Do antithyroid antibody
D) Look for malabsorption
E) Repeat T4, TSH
4. A 6-year-old boy was referred by school medical officer due to short stature and limb deformities. O/E there
are Harrison sulcus, genu varus deformity and mild pallor. No organomegaly (2009.PAED.SBR.48)
• Serum Ca - 8 (9-10.5)
• Serum phosphates - 6.2 (2.5-4.5)
• ALP - 1800 (<400)
What is the likely condition?
A) Blount disease
B) Nutritional rickets
C) Osteoporosis
D) Renal osteodystrophy
E) Vitamin D resistant rickets
5. 4-year-old boy with bowlegs. X rays show features of rickets. Calcium level 2.05, Phosphate -2.42, alkaline
phosphatase - 1850. Next step of management? (2010.PAED.SBR.45)
A) Do USS abdomen
B) Treat as nutritional rickets.
C) Arrange an orthopaedic referral.
D) Do a blood gas analysis.
E) Request serum creatinine levels.
6. Growth Hormone secretion is stimulated by (2011.PAED.MCQ.18)
A) Sleep
B) Exercise
C) Insulin
D) Glucagon
E) Psychosocial deprivation
7. Baby boy born at term presented on day 6 with poor breast feeding and weight loss of weight. Thyroid
screening test results - TSH 50. What is the next step of management? (2011.PAED.SBR.26)
A) Admit to the ward for further evaluation
B) Start thyroxine
C) Arrange USS of neck
D) Repeat TSH and T4
E) Reassure the mother and child
8. A 6-day old term infant with a birth weight of 3kg is admitted with a complain of poor breast feeding. His
weight is 2.8kg. Heel prick TSH is 60 IU/L What is the next step in the management? (2011.PAED.SBR.47)
A) Start thyroxine immediately
B) Perform venous tsh and t4
C) Uss thyroid
D) Check serum free t4 level.
E) Optimize breast feeding.
9. Congenital hypothyroidism (2012.PAED.MCQ.19)
A) Has a strong association with trisomy 21
B) Commonly due to dyshormonogenesis.
7
C) Screening done by TSH level after birth
D) Normal weight at birth
E) Causes unconjugated hyperbilirubinemia
10. 14-year-old girl present to paediatrics clinic due to short stature. She is the shortest one in the class since
preschool. Her learning is average. On examination she is on prepubertal stage. Which of the investigation
most likely to come to diagnosis.? (2012.PAED.SBR.33)
A) CT brain B. FSH
B) Karyotype D. Free T4
C) .
D) .
E) E. USS of thyroid

8
2. IPD Module - Final MBBS Common MCQs
Medicine – IPD
1. 65 yr old man presented with fever, headache and drowsiness for 2 days. He has history of recurrent ear
Infection. What is the most appropriate investigation to come into diagnosis? (2008.MED.SBR.1)
A) Ear swab and culture
B) Blood culture
C) EEG
D) CT brain
E) Lumbar puncture
2. 25-year-old female presented with 3-day history of high fever and R/s pleuritic type chest pain. She has
produced rusty sputum. FBC- Hb. PLT - NL WBC- 19,000 (N-85%). ESR-100 What is the most likely diagnosis?
(2008.MED.SBR.8)
A) Tuberculosis
B) Streptococcus pneumonia
C) Klebsiella pneumonia
D) Aspergillosis
E) Staphylococcal pneumonia
3. 68-year-old male presented with history of TB (completed Treatment with bronchiectasis, this time presented
with fever and cough. Temp - 38.3 C. SpO2 - 88%. what is the best empirical antibiotic for this man?
(2008.MED.SBR.9)
A) Benzyl penicillin
B) Oseltamivir
C) Clarithromycin
D) Tetracycline
E) Ceftazidime
4. 21-year-old boy from Kataragama presented with fever, myalgia, headache and tinnitus for2 days, ONE-
conjunctival injection, maculopapular rash, cervical lymphadenopathy and hepatomegaly was present. PLT -
low. AST, ALT- elevated. What is the most appropriate antibiotic? (2008.MED.SBR.22)
A) Benzyl penicillin
B) Ciprofloxacin
C) Clarithromycin
D) Doxycycline
E) Metronidazole
5. 24-year-old male presented with headache, myalgia, arthralgia, Intermittent fever with chills for 1 week. He
has returned from India 2 weeks back. O/E - mild hepatomegaly with no LN enlargement. FBC-Hb-10.8 g/dl
WCC- NL, PLT - 120,000 (150,000-450,000). What is the most appropriate Initial investigation to come into a
diagnosis? (2008.MED.SBR.23)
A) Blood culture
B) HIV antibody level
C) Urine microscopy
D) Thick blood film for malaria parasite
E) Standard agglutination test (SAT)
6. 27 yr old male was bitten by an unknown snake at 8 pm in the night. His R/ leg is swollen where the bitten
mark is present. He complains of pain in the leg. He is conscious and rational. PR - 90, BP - 110/70 mm Hg.
What is most appropriate the first step in management (2008.MED.SBR.26)
A) Administer antivenom immediately
9
B) NSAIDs for pain
C) Administer tetanus toxoid
D) Treat with oral cloxacillin
E) 20min WBCT
7. Correctly matched microorganism with medication (2009.MED.MCQ.14)
A) Methicillin sensitive Staphylococcus - cloxacillin
B) Pseudomonas aeruginosa - co amoxiclav
C) Mycoplasma pneumoniae - clarithromycin
D) Plasmodium vivax - chloroquine
E) Chlamydia trachomatis – doxycycline
8. Regarding malaria in Sri Lanka (2009.MED.MCQ.15)
A) Is currently an endemic disease in Sri Lanka
B) Vivax malaria is known as tertian malaria
C) Falciparum malaria can cause nephritic syndrome
D) Pathology of cerebral malaria is encephalopathy
E) Artemisinin is the most effective drug
9. Which of the following cause a characteristic rash in palms and soles? (2009.MED.MCQ.16)
A) Secondary syphilis
B) Herpes simplex virus
C) Condylomata accuminata
D) Keratoderma blennorrhagica
E) Arsenic poisoning
10. A 28-year-old prisoner was brought to the medical ward with an acute fever episode. His CSF analysis findings
were, Polymorphs 2, Lymphocytes 48, Protein 458 mg/dl, Sugar 28 mg/dl, RBS 200 mg/dl/ What is the most
probable diagnosis? (2009.MED.SBR.24)
A) Partially treated meningitis
B) Tuberculous meningitis
C) Viral meningoencephalitis
D) Leptospirosis
E) Viral encephalitis
11. A 3-year medical student had an accidental prick injury to the hand while taking a blood sample of a patient,
suspected with Hep B infection. What is/are the most appropriate investigations that should be performed
subsequently? (2009.MED.SBR.33)
A) HBsAg of the patient
B) HBsAg of the student and HIV antibody of the patient
C) HBsAb of the student and HIV antibody of the patient
D) HBsAg and HIV antibody of the patient
E) HBsAg and HIV antibody in both
12. An 80-year-old male presented with 4 weeks history of fever. During last 2 years he had similar lower urinary
tract symptoms. Symptoms disappear after treating with norfloxacilin but fever persists. He has hypertension,
diabetes and prostatism and is on metformin and prazosin. He has mild pallor and a splenomegaly. A diagnosis
of infective endocarditis is made. What is the most likely causative organism for infective endocarditis in this
patient? (2009.MED.SBR.41)
A) Enterococci
B) Gram negative organism
C) Salmonella
D) Staphylococci
10
E) Viridans Streptococci
13. Which of the following is the best for preventing hospital acquired infections? (2009.MED.SBR.42)
A) Give prophylactic antibiotics
B) Vaccination of the hospital workers
C) Proper hand care of hospital workers
D) Early discharge
E) Isolation of the patients
14. A 55-year-old male presented to the medical ward after being bitten by an unidentified snake. He was having
right leg swelling up to the knee joint and blackish discoloration at ankle joint with an oozing puncture mark.
No other physical signs present Clotting test is normal. What would be the immediate decision?
(2009.MED.SBR.47)
A) Antivenom and surgical referral
B) Surgical referral
C) Limb elevation & antibiotic
D) Antibiotics, antivenom and surgical referral
E) Antibiotics and antivenom
15. Regarding diagnosis of pulmonary tuberculosis, (2010.MED.MCQ.5)
A) Positive smear confirms the diagnosis
B) Silver staining of acid-fast bacilli
C) Culture report for AFB takes 6 days
D) Treatment is with IV cefotaxime
E) Positive gamma interferon test is diagnostic
16. Which of the following have vaccines? (2010.MED.MCQ.14)
A) Hepatitis A
B) Typhoid
C) Leptospirosis
D) Hepatitis C
E) Haemophilus influenza infection
17. Which of the following can be used to suggest dengue fever in first day of fever? (2010.MED.MCQ.20)
A) Low platelet count
B) High haematocrit
C) NS1 antigen
D) Dengue DNA PCR
E) Dengue IgM antibody levels
18. What are the features to detect entering the dengue critical phase? (2010.MED.MCQ.20)
A) Platelet count
B) Haematocrit
C) Ca2+ levels
D) Urine output
E) Pulse pressure
19. A 30-year-old male presented with cough, fever and pleuritic type chest pain. Examination is unremarkable
except left lower lobe consolidation which is confirmed by chest x-ray. What is the best treatment option?
(2010.MED.SBR.30)
A) Start IV clarithromycin
B) Start oral co-amoxiclav
C) Start oral amoxicillin and clarithromycin
11
D) Admit to ICU
E) .
20. 44) 47 years old retired government officer presented. Fever for 8 days and confusion for last 2 days. At the
beginning of the fever, it was associated with anorexia nausea and lethargy. He is a diagnosed patient with
Diabetes mellitus and hypertension and on HCT and metformin. He visited to London and Thailand 5 years
back. and India 2 months ago. On examination, he is drowsy, pale, little icteric and has splenomegaly. Other
examinations are normal. What is the diagnosis? (2010.MED.SBR.44)
A) Typhoid.
B) Dengue.
C) Viral encephalitis.
D) Malaria.
E) HIV seroconversion illness
21. A 30 years old farmer admitted following snake bite. There are no bleeding manifestations. No local
implantation, but he is drowsy, GCS- 14/15, BP- 120/80 mmHg, PR- 80 bpm. Antivenom was given successfully
and uncomplicatedly After one hour patient become drowsier. Respiratory rate is 6/min, BP- 80/60 mmHg.
What is the most appropriate next step of management? (2010.MED.SBR.48)
A) Normal saline rapid bolus.
B) Continuous Positive Airway Pressure.
C) Repeat antivenom.
D) IM adrenaline
E) Intubation & ventilation
22. A CSF report shows increased protein increased polymorphonuclear leukocytes and Possibilities include,
(2011.MED.MCQ.2)
A) TB meningitis
B) Meningococcal meningitis
C) Herpes simplex meningitis
D) Cryptococcal meningitis
E) Cerebral malaria
23. Regarding legionella pneumophila (2011.MED.MCQ.6)
A) Causing diarrhoea
B) Lymphocytosis
C) Treated with ceftriaxone
D) Causing hyponatremia
E) Present in urine
24. Regarding sepsis (2011.MED.MCQ.14)
A) Is a common cause of distributive shock
B) If hypotensive, start ionotrope immediately
C) Dobutamine is a choice of inotrope in sepsis
D) Extremes of age is a risk factor
E) Antibiotics given only after blood culture and ABST result
25. Regarding complications of leptospirosis: (2011.MED.MCQ.15)
A) Pyogenic meningitis
B) Hemorrhagic pneumonitis
C) Myocarditis
D) Pyelonephritis
E) Acalculous cholecystitis

12
26. 25 years old lady presents with fever for 4 days with arthralgia and myalgia. On examination BP 120/80
Investigations were as follows WBC 10.5 (Neutrophil 60%), Hb -.. (low) AST- 140, ALT- 135. UFR, Red cells +.
What is the diagnosis? (2011.MED.SBR.36)
A) Dengue
B) Leptospirosis
C) Hanta
D) Typhoid
E) .
27. 23-year-old women presented with fever severe headache she has rash in the face neck and body, her blood
pressure is 80/50 mmHg she has mild neck stiffness What is the most likely diagnosis (2011.MED.SBR.37)
A) DHF
B) Hantavirus infection
C) Meningococcal infection
D) Rocky mountain spotted fever
E) Rubella infection
28. 52-year-old farmer presents with unknown snake bite while going to the paddy field. He has progressive
slurring of speech and bilateral ptosis. There's a fang mark on his left ankle. His urine output is satisfactory.
Urinalysis showed 12-20 on moderate field. What can be the snake? (2011.MED.SBR.40)
A) Common krait
B) Cobra
C) Ceylon krait
D) Hump nosed viper
E) Russell's viper
29. Middle age man presented with one hour after a snake bite. He has Difficulty in breathing and saturation is
normal. Also Having past history of 2 episodes of anaphylaxis. On examination patient is andous, tachycardia,
tachypnoeic, no local swelling or fang marks, but ptosis is there, WBCT<20minutes what is the best
management option? (2011.MED.SBR.41)
A) Give full dose of antivenom
B) Observe without giving antivenom
C) Start ventilation
D) Give antivenom with neostigmine
E) Give half dose of antivenom
30. 68 years old male patient diagnosed with type 2 DM presented with painful, red, swollen, hard, tender lesion
in the calf of right leg, what is organism responsible for this presentation? (2011.MED.SBR.42)
A) HSV
B) Staph aureus
C) Staph epidermidis
D) Streptococcus pyogenes
E) Strep pneumonia
31. 30 years old male presented with an unknown snake bite. His night leg is swollen and also, he is having signs
and symptoms of acute renal failure. Whole blood clotting time is normal. What is the most
(2011.MED.SBR.44)
A) Cobra
B) Russell's viper
C) Hump nosed viper
D) Common krait
E) Saw scaled viper

13
32. Regarding erysipelas (2012.MED.MCQ.7)
A) Edges are discrete and merge into the skin
B) Caused mainly by group A beta hemolytic streptococci
C) Affect dermis and superficial subcutaneous tissue
D) Systemic spread can cause multi organ dysfunction
E) Penicillins are the treatment of choice
33. Regarding scrub typhus (2012.MED.MCQ.11)
A) Causes meningoencephalitis
B) Causative organism is Rickettsia prowazekii
C) Presence of eschar is useful in the diagnosis
D) Weekly doxycycline therapy can prevent the disease
E) Causes thrombocytopenia
34. Regarding tuberculosis (2012.MED.MCQ.12)
A) People vaccinated with BCG rarely get TB
B) Isolation of bacteria in sputum is essential for diagnosis
C) .
D) If spine is involved, duration of therapy 9 months
E) It's a notifiable disease in Sri Lanka
35. Regarding hepatitis virus serology (2012.MED.MCQ.19)
A) A positive Hepatitis B Surface antigen indicates active infection
B) IgM antibodies to hepatitis B core antigen is positive in acute infection
C) Hepatitis B surface antigen and anti HBs antibodies can co-exist
D) Presence of Hepatitis B e antigen is indicative of infectivity
E) Anti Hbs antibody in blood shows presence of immunity
36. A 36-year-old male, with a history of an unprotected sexual act 4 weeks ago, presents to STD clinic with a
painless ulcer on penis. Previously well. What’s the Possible lesion? (2012.MED.SBR.3)
A) Chancroid
B) Genital herpes
C) Gonorrhea
D) Syphilis
E) Chlamydia
37. A 22-year-old male returned from India 1 month back. Now has intermittent fever with chills and rigors,
headache, arthralgia and myalgia for 1 week. On examination mild hepatomegaly with no lymph node
enlargement. FBC: WBC normal, Hb – 10.8, platelets – 120,000. what is the most appropriate initial
investigation to come to a diagnosis? (2012.MED.SBR.7)
A) Urine microscopy
B) HIV antibody
C) MAT for leptospirosis
D) Blood culture
E) Malaria blood film
38. 45-year-old IV drug user presenting with intermittent fever and a murmur on left lower sternal edge.
Diagnosed to have infective endocarditis. Most probable causative organism? (2012.MED.SBR.17)
A) Streptococcus epidermidis
B) Streptococcus bovis
C) Streptococcus viridans
D) Pseudomonas aeruginosa
E) Staphylococcus aureus
14
39. 18-year-old man presented following a hump nosed viper bite. No systemic envenomation features noted.
Only two fang marks are present. What is the next step in management? (2012.MED.SBR.24)
A) Arrange WBCT
B) Arrange renal function tests
C) Start antivenom serum
D) Monitor for 24 hours
E) Haemodialysis
40. 45-year-old male presented with low grade fever for 5 days associated with headache and myalgia. There has
been a dry cough for the past few days. On examination he is mildly icteric and has bilateral crepitations. Chest
x ray reveals bilateral diffuse opacities over the lower lobes of lungs. Most possible causative organism?
(2012.MED.SBR.30)
A) Streptococcus pneumoniae
B) Streptococcus epidermidis
C) Legionella pneumophila
D) Mycoplasma pneumoniae
E) Staphylococcus aureus

Surgery – IPD
1. A 7yr old boy was taken to surgery casualty ward with a knee joint swelling with pain. WBC 22000, with
neutrophil predominance, ESR 106mm/1st hour, CRP 9.6mg/dl. What is the management? (2008.SUR.SBR.24)

A) Start IV antibiotics
B) IV broad spectrum antibiotics
C) Joint aspiration
D) Open surgical drainage
E) Splinting

2. Tests to diagnose H.Pylori (2009.SUR.MCQ. 4)

A) Serology
B) Faecal antigen test
C) Nitrogen breath test
D) Rapid urease test
E) Histology

3. Middle ear infections (2009.SUR.MCQ. 16)

A) Vertigo is a feature
B) Common in children
C) Majority is haematogenous
D) Pain is a prominent feature in chronic otitis media
E) Is a cause of meningitis

4. Viral conjunctivitis (2009.SUR.MCQ. 18)

A) Causes pain
B) Causes halos around light
C) Does not impair the visual acuity
D) Has gritty sensations
E) Causes small irregular pupils

5. 72-year-old diabetic man, on oral hypoglycaemic drugs, has high fever for 2 days with right knee joint swelling.
He had extremely limited joint movement and USS shows joint effusion. What is the most appropriate
management? (2011.SUR.SBR. 30)

15
A) Joint aspiration
B) IV antibiotics
C) Arthrotomy
D) MRI
E) Above knee POP cast

6. 18-year-old man presented with redness and itching of the eyes. He also has stickiness of the eyes. He has
dysuria and pain in the left knee joint. On examination eyes are diffusely red and injected. Possible diagnosis
would be, (2011.SUR.SBR.40)

A) Adenovirus
B) Chlamydia
C) Gonorrhoea
D) Staphylococcus
E) Streptococcus

Paediatrics – IPD
1. What agents need prophylaxis for meningitis following a close contact? (2009.PAED.MCQ.06)
A) Streptococcus pneumoniae
B) Haemophilus influenzae
C) GBS
D) E. coli
E) Neisseria meningitidis
2. National immunization programme (2009.PAED.MCQ.19)
A) Oral polio is important to produce herd immunity
B) Coverage is 80%
C) Adverse effects are repeated to the Family Health Bureau
D) MMR is given at 1 year
E) JE is contraindicated in cerebral palsy
3. 2-year-old boy presented with high fever and bloody diarrhoea with abdominal pain on examination there
was generalized abdominal tenderness. Which of the following are possible causative organisms?
(2010.PAED.MCQ.08)
A) Campylobacter
B) Clostridium botulinum
C) Enterotoxigenic E. coli
D) Giardia lamblia
E) Shigella dysenteriae
4. Regarding intrauterine infections, (2010.PAED.MCQ.11)
A) HIV transmission directly correlated to maternal viral load
B) Zika virus causes microcephaly
C) .
D) .
E) .
5. Regarding the immunization programme in Sri Lanka, (2010.PAED.MCQ.19)
A) Rubella vaccine not indicated when child already got the disease.
B) OPV is given at 5 years.
C) When a vaccine lapses by 3 months the schedule should start from the beginning.
D) IPV is a live attenuated vaccine
E) JE is given at 12 months.
6. Strategies to promote rational antibiotic use are (2011.PAED.MCQ.11)

16
A) Use antibiotics for all patients with viral infections to prevent secondary bacterial Infection
B) Use literature provided by drug industry
C) Educate the population
D) Prescribe from the essential drug list
E) Having a low threshold to use empirical broad-spectrum agents
7. Regarding Adverse Events Following Immunization (2011.PAED.MCQ.16)
A) Thrombocytopenia following MMR
B) Reported to regional epidemiologist
C) Reporting by the community health team is acceptable
D) Hyporesponsive hypotensive state known to occur following pentavalent vaccine
E) Lymphadenitis following BCG is seen up to 6 months
8. 2 months old Infant with scabies, what is the best treatment? (2008.PAED.SBR.15)
A) Ivermectin
B) 2% sulphur ointment
C) Benzyl benzoate
D) Permethrin
E) Crotamiton
9. A 12-year-old child presents with history of fever and headache for 7 days and altered bowel habits for 3 days.
Which of the following clinical sign best suggests the diagnosis of typhoid fever? (2009.PAED.SBR.35)
A) Doughy abdomen
B) Coated tongue
C) Hepatomegaly
D) Splenomegaly
E) Monoarthropathy
10. A 9 yr old girl was bitten by an unknown snake while walking by the paddy field on a rainy evening. She was
brought to the rural hospital within 1/2 hr. She complains of severe pain at the bite site and WBCT is < 20 min.
Most likely snake is (2009.PAED.SBR.37)
A) Cobra
B) Greer Pit Viper
C) Hump Nosed Viper
D) Common Krait
E) Saw Scaled Viper
11. A child has presented with high fever, cough and cold for 4 days. Following admission, he developed a
maculopapular rash behind his right ear. What is the most likely diagnosis? (2010.PAED.SBR.35)
A) Infectious episcleritis
B) Measles
C) Parvo virus infection
D) .
E) .
12. Fever for 4 days cough and cold rash on the day of the admission maculopapular rash starting behind the ear.
What is the most likely aetiology? (2010.PAED.SBR.47)
A) Parvovirus
B) Roseola infantum
C) Measles
D) Coxsachie
E) .
13. A 9 yr old child developed nausea, abdominal and cramping watery diarrhoea. He is mildly dehydrated, and
abdomen is tender. He had eaten Chinese fried rice the previous night. What is most likely organism to cause
this illness? (2011.PAED.SBR.38)
A) Bacillus cereus
17
B) Vibrio parahaemolyticus
C) Salmonella typhimurium
D) Shigella flexneri
E) Campylobacter Jejuni
14. 6 months old baby, presented with irritability, fever and poor feeding. Irritable when handling and there is
neck retractability O/E 2 vesicles noted on the upper chest. LP was done. RBC-220, Protein - 1.2g/dl (0.4-0.8),
CSF WBC elevated with 90% lymphocytosis. CSF sugar is marginally reduced than RBS. What is the most likely
diagnosis? (2011.PAED.SBR.39)
A) Complex febrile convulsions
B) Non accidental injury
C) Toxic shock syndrome
D) Partially treated bacterial meningitis
E) Varicella encephalitis
15. A 9 yr old boy presents with 3day hx of fever, abdominal pain, vomiting, dark urine and has reduced urine
output. This has occurred after family trip. O/E he was drowsy, icteric with conjunctival suffusion. Abdominal
examination revealed a 2cm hepatomegaly without any associated splenomegaly. What is the initial
investigation? (2011.PAED.SBR.40)
A) Hepatitis A IgM levels
B) Liver function test
C) Leptospira antibody test
D) USS abdomen
E) .

18
3. CVS Module - Final MBBS Common MCQs
Medicine – CVS
1. Drugs with survival benefit in cardiac failure (2012.MED.MCQ.01)
A) Digoxin
B) Ivabradine
C) Frusemide
D) Carvedilol
E) Spironolactone
2. In the management of cardiac failure, influencing renin angiotensin aldosterone system, (2012.MED.MCQ.04)
A) Is done using ramipril
B) .
C) Improves re-modelling of heart
D) Should be considered in all patients
E) Is indefinitely used in patients with left ventricular failure
3. Advantages of LMWH over unfractionated heparin (2012.MED.MCQ.05)
A) No need of monitoring
B) Effect is easily reversible
C) Due to its long half-life, it doesn't have to be given as an infusion
D) Less heparin induced thrombocytopenia
E) Can be managed as outpatient
4. Regarding giant cell arteritis (2012.MED.MCQ.06)
A) Is a large vessel vasculitis
B) Visual loss is a complication
C) Is treated with high dose steroids
D) Is associated with polymyalgia rheumatica
E) ESR is < 50mm/ 1st hour in majority
5. Postural hypotension is a feature of, (2012.MED.MCQ.13)
A) Addison’s disease
B) Prazosin therapy
C) Diuretic therapy
D) Pheochromocytoma
E) Steroid therapy
6. T/F regarding AF (2011.MED.MCQ.03)
A) Metoprolol is contraindicated
B) CHA2DS2VASC is used to decide on anticoagulant treatment
C) DC cardioversion is aimed at rate control
D) Thyrotoxicosis is a known cause
E) It causes acute ischemia to the lower limb
7. Which of the following is are true /false regarding Rheumatic fever? (2011.MED.MCQ.04)
A) Caused by group A strep. Infection
B) When it involves heart, myocardium is frequently affected.
C) Chorea is a common feature.
D) Frequently affect small and large joints.
E) Erythema marginatum is a feature.
8. About cardiac cycle (2011.MED.MCQ.18)
A) Ventricular filling is predominantly a passive process
B) Blood supply to the heart occurs during systole

19
C) Tricuspid and mitral valve closes during early diastole
D) Pulmonary valve closes before aortic valve during systole
E) Atria contracts during late diastole
9. 45yrs old presented with sob for 3hrs.pr 140bpm.bp 85/70mmhg.heart sounds normal lungs clear, sat 99%,
ECG rate 138 bpm, narrow QRS absent p wave. Management? (2011.MED.MCQ.25)
A) Carotid massage
B) IV amiodarone
C) DC cardioversion
D) IV adenosine
E) IV verapamil
10. Increased JVP, with normal waveform (2010.MED.MCQ.03)
A) Obstructed SVC
B) Complete Heart block
C) IV fluid overload
D) Cardiac failure
E) Tricuspid regurgitation
11. Amiodarone (2010.MED.MCQ.22)
A) Can be used to treat SVT
B) It is known to cause hypothyroidism
C) It is known to cause hypoparathyroidism
D) .
E) .
12. On examination BP 120/50 mmHg recorded in the right upper arm. What are the possibilities
(2009.MED.MCQ.03)
A) AR
B) MR
C) AS
D) PDA
E) VSD
13. Regarding arctic stenosis (2009.MED.MCQ.04)
A) Can present with angina
B) ECG may show left ventricular hypertrophy
C) In severe disease there can be narrow pulse pressure
D) Has a thrusting apex
E) Has a loud 2t heart sound.
14. A 50-year-old male from UK visiting Sri Lanka for a short vacation presented with acute onset difficulty of
breathing, cough and left sided chest pain. Well controlled asthma on combined inhalers, smoking 10 pack
years. On examination, no fever, Pulse rate was 120 and Blood pressure 112/72 mmHg, Respiratory
examination was unremarkable. SpO2 was 88%. ECG - Sinus tachycardia with V1-V4 leads T inversions. Full
Blood Count normal. Most probable diagnosis? (2012.MED.SBR.08)
A) Acute exacerbation of asthma
B) Acute exacerbation of COPD
C) Myocarditis
D) Acute coronary syndrome
E) Pulmonary embolism
15. 25-year old male returned from holiday came with a left sided chest pain for 1-day duration. He had flu for 3
days. The pain was episodic and ceases on expiration. ECG was done. There were ST elevations in anterior and
lateral leads. CRP=30, WBC=11, troponin +. What is the most likely diagnosis? (2012.MED.SBR.13)
A) Acute coronary syndrome
B) Acute pericarditis
20
C) Pneumothorax
D) Pleural effusion
E) .
16. 45-year-old IV drug user presenting with intermittent fever and a murmur on left lower sternal edge.
Diagnosed to have infective endocarditis. Most probable causative organism? (2012.MED.SBR.17)
A) Streptococcus epidermidis
B) Streptococcus bovis
C) Streptococcus viridans
D) Pseudomonas aeruginosa
E) Staphylococcus aureus
17. 50-year-old patient presenting with central chest pain which is relieved on expiration. CVS and RS examination
normal. ECG – prominent R wave and ST depression in V1 and V2. Most probable diagnosis?
(2012.MED.SBR.21)
A) Brugada syndrome
B) NSTEMI
C) Posterolateral MI
D) True posterior MI
E) Myocarditis
18. 50 years old male, smoker, developed right ventricular and inferior STEMI. His PR 60 bpm, BP 70/50 mmHg
What is the most appropriate next step to improve BP? (2011.MED.SBR.23)
A) Dopamine infusion
B) Dobutamine Infusion
C) 0.9% normal saline fluid bolus
D) Noradrenaline infusion
E) Temporary pacing
19. 35 years British woman presented to the emergency department. She had 10 hrs flight from London Colombo
1 week ago. She complains Dyspnoea, cough, chest pain during deep breaths. She was healthy prior to the
trip. Her past medical history was uneventful apart from hx of two miscarriage. Most appropriate investigation
in arriving at the diagnosis is (2011.MED.SBR.25)
A) D-dimers
B) CT- pulmonary angiogram
C) HRCT chest
D) Echocardiogram E
E) PET-CT chest
20. 40y old male travel guide, presented with progressive SOB for 3 months. Diagnosed DM for 4 years. On
metformin and gliclazide is a teetotaller. On examination mild icterus, ankle oedema, shrunken Iver, mild
splenomegaly, bilateral fine crepitations. Investigations- hb-10 mcv-84, plt-04, s. Alb- 15, AST-64, alt-84, direct
bilirubin 68, alp-110. CXR upper lobe diversion, cardiomegaly What is the diagnosis (2011.MED.SBR.27)
A) Cardiac cirrhosis
B) Haemochromatosis
C) Hep B
D) NASH
E) Wilson’s
21. A 65-year old man was diagnosed to have proximal DVT following a total hip replacement surgery was started
with anticoagulant and PT INR is maintained at 2.5. What is the most appropriate next step in the
management? (2011.MED.SBR.34)
A) Insert an IVC filter
B) Continue anticoagulant indefinitely
C) Continue anticoagulant for 3 months and follow up with thrombophilia screening
D) Continue anticoagulant for 2 weeks and convert to aspirin
E) Continue anticoagulant for 3 months.
21
22. A 47-year female presented to the clinic with persistent hypertension. Her last blood pressure recorded was
160/90mmHg. She is asymptomatic and recently lost her job. Investigations revealed Na+ 146, K+- 4.5, Serum
urea4mmol/L and RBS 7.1mmol/L. What is the most appropriate investigation? (2010.MED.SBR.26)
A) 2D echocardiography
B) 24 hours ambulatory blood pressure monitoring
C) Transabdominal scan
D) .
E) .
23. 60-year-old male patient with left ventricular systolic dysfunction presented with SOB on climbing stairs, but
normal at rest. Commenced on Ramipril and Frusemide. Which drug would improve prognosis?
(2010.MED.SBR.27)
A) Bisoprolol
B) Digoxin
C) Amlodipine
D) GTN
E) Amiodarone
24. Previously well 60-year-old gentleman constricting type chest pain. ECG ST depressions in anterior leads.
Troponin negative and treated with heparin. What is the best combination of drugs for him at discharge?
(2010.MED.SBR.28)
A) Aspirin, ACEI, Furosemide, beta blocker, Statin
B) Aspirin, clopidogrel, ACEI, beta blocker, Statin
C) Aspirin, clopidogrel, nitrates, beta blocker, Statin
D) Aspirin, clopidogrel, ACEI, nitrates, K+ blocker
E) Aspirin, clopidogrel, nitrates, Ca+ blocker, statin
25. A 55-year-old male who is a diagnosed patient with rheumatic heart disease, presented with drowsiness and
faintishness. On examination he has irregularly irregular pulse, PR-160 bpm, and BP 70/50 mmHg. What is the
most appropriate management for this patient? (2009.MED.SBR.25)
A) Amiodarone
B) IV dobutamine
C) DC cardioversion
D) N/S intravenous infusion
E) IV digoxin
26. A 60-year-old previously well male complained of progressive breathlessness, abdominal discomfort and
swelling of the legs. On examination there was gross ascites, elevated JVP, bibasal crepitations and an enlarged
liver. BP 180/80 mmHg, RR - 25/min, PR-36 BPM. On ECG heart rate was 96 bpm. What is the best management
option? (2009.MED.SBR.26)
A) Digoxin
B) Digoxin + Furosemide
C) Furosemide
D) Abdominal paracentesis
E) Transthoracic pacemaker
27. A 55-year-old male with history of diabetes mellitus and hypertension presented with retrosternal chest pain.
ECG showed 4mm ST elevations in lead V1 to V6. Streptokinase was given after excluding contraindications.
After 2 hours, pain persists, and ECG shows 3mm ST-elevations. What is the best management option?
(2009.MED.SBR.27)
A) Urgent coronary artery bypass grafting
B) Rescue percutaneous coronary interventions
C) Repeat thrombolysis with alteplase
D) Subcutaneous enoxaparin
E) IV GTN

22
28. A 35-year-old male presents with bilateral ankle swelling to his routine medical check-up. BP 160/105 mmHg,
PR-78 bpm, UFR- protein ++, RBC- 2 hpf, serum creatinine-increased, blood urea-increased, Nat- normal, K+-
normal, GFR-40, USS-increased cortical echogenicity, normal size kidney. What will be the best drug?
(2009.MED.SBR.34)
A) Methyldopa
B) Nifedipine
C) Captopril
D) Prazosin
E) Sildenafil
29. An 80-year-old male presented with 4 weeks history of fever. During last 2 years he had similar lower urinary
tract symptoms. Symptoms disappear after treating with norfloxacilin but fever persists. He has hypertension,
diabetes and prostatism and is on metformin and prazosin. He has mild pallor and a splenomegaly. A diagnosis
of infective endocarditis is made. What is the most likely causative organism for infective endocarditis in this
patient? (2009.MED.SBR.41)
A) Enterococci
B) Gram negative organism
C) Salmonella
D) Staphylococci
E) Viridans Streptococci
30. 28yr old man with known valvular heart disease presented with fever for 2 weeks duration and SOB. O/E -
temperature -38.3C, harsh pansystolic murmur and mild bibasal crepitations. Blood cultures were taken. What
is the most appropriate next step in management, (2008.MED.SBR.04)?
A) ECG
B) IV antibiotics
C) Throat swab
D) Frusemide
E) 2D – Echo
31. 25 yr primi-gravida mother in T3 of pregnancy, admitted to emergency medical clinic with progressive
worsening of SOB for 3 days. O/E-BP-90/60 mm HR. PR-100 with fine basal crepitations. Oxygen saturation on
room air - 90%. What is the most appropriate Investigation? (2008.MED.SBR.05)
A) Troponin I
B) Chest x-ray
C) ABG
D) Transthoracic Echocardiogram
E) FBC
32. 43 yr old female presented with severe headache. O/E-BP - 230/120mm Hg and papilledema. What is the best
treatment option? (2008.MED.SBR.06)
A) Sublingual GTN
B) Sublingual nifedipine
C) Prazosin And atenolol
D) IV GTN
E) Frusemide infusion
Surgery – CVS
1. 30-year-old previously healthy male presented with episodes of headaches, generalized malaise, sweating &
palpitations. His BP is 220/110 during one episode. What is the most appropriate investigation?
(2008.SUR.SBR.15)
A) CT abdomen
B) S.calcium level
C) USS abdomen
D) Urinary catecholamine

23
E) S cholesterol
2. WOF is the most likely clinical finding in class 2 hemorrhagic shock. (2008.SUR.SBR.23)
A) HRC 100 bpm
B) Normal systolic blood pressure
C) Low diastolic blood pressure
D) UOP > 30ml/hr
E) Confusion
3. 56 years old Elderly male, smoker, presented with intermittent claudication without any evidence of tissue
loss. He does not have DM, HT and dyslipidemia What is the best management option? (2008.SUR.SBR.26)
A) Stop smoking, antiplatelet, vasodilators, statins
B) Stop smoking, antiplatelet, vasodilators, stenting
C) Stop smoking, walking exercise, antiplatelet therapy, statin therapy
D) Stop smoking, antiplatelet, vasodilators,
E) Walking exercise, antiplatelet, vasodilators and statins
4. Patient came to surgical ward 8 hours after right hemi colectomy was on epidural analgesia, found to have
pain, low blood pressure and dyspnoea. What is the most appropriate explanation for above condition?
(2008.SUR.SBR.30)
A) Clot dislodgment, epidural analgesia, slip ligature
B) Clot dislodgment, slip ligature, DVT
C) Clot dislodgment, Sepsis, slip ligature
D) Clot dislodgment, epidural analgesia, slip ligature
E) .
5. Regarding the occlusive arterial disease of lower limb, (2009.SUR.MCQ.10)
A) ABPI of 1.0 is abnormal
B) Critically ischemic limb cannot be saved
C) Rest pain is best managed with anticoagulants
D) Short segment occlusion is best treated with angioplasty
E) Distal vascular disease is a contraindication for bypass surgery
6. Preoperative ECG monitoring can be used to, (2009.SUR.MCQ.13)
A) Assess cardiac output
B) Diagnose volume status
C) Diagnose myocardial depression
D) Detect MI
E) Identify cardiac arrhythmias
7. A 28-year old woman presents with saphenofemoral and long saphenous reflux causing varicose veins and
pigmentation in the gaiter area What is the most appropriate management? (2009.SUR.SBR.31)
A) Physical exercise and compression stockings
B) Saphenofemoral ligation and sclerotherapy
C) Saphenofemoral ligation, stripping & stab avulsion
D) Sclerosant injection to varicosities
E) Stab avulsion of varicosities
8. A 68-year-old patient was brought back to the ward following an uncomplicated abdominal aortic aneurysm
repair. He developed significant pain postoperatively. He was seen by the anesthetic team 15 min ago and an
epidural top up was given. Nurse calls you to review as blood pressure drops to 90/50 and pulse rate 60. He
feels dizzy but is not in pain. His urine output is adequate postoperatively and CVP is 12 mmHg. What is the
most likely diagnosis of this patient? (2009.SUR.SBR.47)
A) Cardiogenic shock
B) Haemorrhage
C) Sympathetic blockade
D) Pulmonary embolism
24
E) Sepsis
9. A 19-year-old male was admitted to the emergency department with a crush injury to the pelvis. Clinical
diagnosis of class 2 hemorrhagic shock was made. What is the most appropriate fluid resuscitation regime he
should receive? (2009.SUR.SBR.16)
A) Rapid infusion of 250ml Hartmann if MAP drops below 60 mmHg
B) Rapid infusion of 1000ml Hartmann
C) Rapid infusion of 250ml hypotonic saline
D) Rapid infusion of 1000 ml hetastarch
E) Rapid infusion of 4 units of O negative blood
10. 75 years old patient treated for hypertension and ischemic heart disease is scheduled to undergo TURP under
spinal anesthesia. Which drug should be discontinued at least one week before? (2010.SUR.SBR.21)
A) Aspirin
B) Stencil
C) Clopidogrel
D) Warfarin
E) Heparin
11. 60-year-old male with patellar fracture immobilized for 1 week presented with calf pain. Duplex scan showed
thrombus in common femoral artery. Most appropriate management is (2010.SUR.SBR.36)
A) Heparin bolus plus infusion
B) Heparin infusion plus stocking
C) Heparin infusion plus warfarin
D) LMWH plus stocking
E) LMWH plus warfarin
12. 38-year-old known patient with rheumatic heart disease presented with pain and swelling of right calf. On
examination pulses were absent beyond the femoral level. What is the most appropriate definitive
management? (2010.SUR.SBR.42)
A) Heparin bolus followed by infusion
B) Iliofemoral bypass surgery
C) Trans femoral embolectomy
D) Digital subtraction angiogram and SK.
E) Arterial duplex scan
13. A 26-year-old man admitted following a road traffic accident. He is dyspnoeic. Neck veins are engorged.
Trachea midline. Lungs normal. PR 136 bpm, BP 80/50 mmHg, RR 35/min. What is the most likely condition?
(2010.SUR.SBR.43)
A) Cardiac tamponade
B) Flail chest
C) Massive haemothorax
D) Pulmonary contusion
E) .
14. T/F regarding abdominal aortic aneurysm (2011.SUR.MCQ.20)
A) Back ache is a known symptom
B) Commonly suprarenal
C) Most are asymptomatic
D) .
E) .
15. 50-year-old male with ischemic heart disease is to undergo emergency laparotomy. His weight is 60 kg and
pre surgical haemoglobin level is 9 g/dl. The expected amount of blood loss during first hour is 1100ml. What
is the best fluid to give? (2011.SUR.SBR.43)
A) 0.9% saline
B) 5% albumin
25
C) Hetastarch
D) Packed red cells
E) Hartmann’s
16. 68 years old male patient who is diagnosed to have diabetes mellitus and hypertension presented with
intermittent claudication of his right leg. Claudication distance is 250m. On examination popliteal and dorsalis
pedis pulses were absent. He is on aspirin and atorvastatin and his diabetic and hypertension control is good.
What would be the best management option for him? (2011.SUR.SBR.48)
A) Angioplasty
B) Bypass surgery
C) Add anticoagulant
D) Dietary advices
E) Exercise programmes
17. 50-year-old retired schoolteacher with history of smoking for 20 years presented with calf claudication after
walking 200 meters. On examination there were absent popliteal and pedal pulses. (2012.SUR. MCQ.16)
A) Arterial imaging is needed to decide further management
B) Expected ABPI 0.3-0.5
C) Revascularization is needed
D) Risk of cardiovascular event is 2-3 times higher than a normal individual
E) Caused by occlusion of superficial femoral artery
18. 53-year-old man on post op 5th day following knee replacement surgery develop pulmonary embolism. Which
is the most appropriate management? (2012.SUR. SBR.40)
A) Dual antiplatelet therapy
B) Placement of IVC filter
C) Thrombectomy
D) Therapeutic dose of anticoagulation
E) Thrombolysis
Paediatrics – CVS
1. 6-month-old child found to have central cyanosis and systolic murmur best heard in pulmonary area. No
cardiomegaly. G&D age appropriate. Rest of the examination normal. Diagnosis? (2008.PAED.SBR)
A) TAPVD
B) TGA
C) Tricuspid atresia
D) TOF
E) ASD
2. A child presented for routine immunization is having a history of defaulted Rx for a congenital heart disease.
MOH wants to refer the child to the paediatric cardiologist. What is the best indicator for referral?
(2008.PAED.SBR)
A) Loud second heart sound,
B) Harsh pansystolic murmur.
C) Thrill in lower sternal edge
D) Poor oral hygiene
E) Ejection systolic murmur in pulmonary area.
3. A 3 days old child presents with SOB. saturation of PaO2 in room air is 80%, What are the possibilities?
(2009.PAED.MCQ)
A) Persistent pulmonary hypertension
B) Coarctation of aorta
C) Eisenmenger syndrome
D) Tetralogy of Fallot
E) Transposition of great vessels

26
4. A 3-year-old child with moderately large VSD presented to hospital for the corrective surgery. His surgery was
postponed several times previously What is the sign that indicate worst prognosis? (2009.PAED.SBR)
A) Tender hepatomegaly
B) Gallop rhythm
C) Loud second heart sound
D) Heaving at 5th intercostal space
E) Severe wasting
5. Aortic stenosis(2010.PAED.MCQ)
A) Presenting with chest pain in children
B) Carotid thrill present
C) Ejection systolic murmur best heard at upper left sternal edge
D) Causes biventricular hypertrophy
E) Second heart sound is loud
6. True regarding Kawasaki disease(2010.PAED.MCQ)
A) Kawasaki is diagnosed by a diagnosed criterion
B) Causes purulent conjunctivitis
C) prednisolone is a treatment option
D) Peeling off the skin is a late feature
E) Viral infection is an aetiological factor
7. 12-year-old girl with past history of rheumatic heart disease presented with fever, right knee joint pain and
swelling for 3 days. Pan-systolic murmur and early diastolic murmur were detected. HR 120 bpm, ESR 112, and
WBC 2000 (N50%). 3 pus cells and 2 red cells in the UFR. ASOT was 400 and ECHO is pending. (2010.PAED.SRB)
A) IE
B) Reactivation of rheumatic fever
C) JIA
D) Post streptococcal arthritis
E) SLE
8. 5-year-old boy was diagnosed to have a short systolic murmur at left upper sternal edge, which changed in
intensity when changing posture. Second heart sound was normal. No carotid thrill or radiation. What is the
most likely cause? (2010.PAED.SRB)
A) Pulmonary stenosis
B) ASD
C) Innocent murmur
D) Tetralogy of Fallot
E) Aortic stenosis
9. Features of uncomplicated PDA include: (2011.PAED.MCQ)
A) Bounding pulse
B) Loud P2
C) Right ventricular hypertrophy
D) Radio femoral delay
E) Wide pulse pressure
10. SVT in an infant (2011.PAED.MCQ)
A) No p waves ECG
B) Treated with adenosine
C) Treated with ice packs on the face
D) Diagnosed when heart rate is more than 160bpm
E) Presents with excessive crying
11. 4-year-old boy presented to the emergency department with sudden onset unusual posture and movements.
He is alert. He has a tilted head and writhing movements of upper limbs. What is the most appropriate Initial
management? (2011.PAED.SBR)
27
A) ASD
B) Atrioventricular malformation
C) Coarctation of the aorta
D) Patent ductus arteriosus
E) Ventricular Septal Defect
12. 8 yr old girl, referred to cardiology from school medical Inspection due to a soft systolic murmur at upper
sternal edge. The murmur changes with posture Pansystolic at apex. She engages in sports at school. What is
the most appropriate management? (2011.PAED.SRB)
A) Admit & evaluate
B) Arrange routine ECG
C) Arrange routine echocardiogram
D) Reassure the mother and the child
E) Review in 3 months
13. Carditis in acute Rheumatic fever is evidenced by(2012.PAED.MCQ)
A) Diastolic murmur
B) Dyspnoea
C) High ESR
D) Hypertension
E) An irregular pulse

28
4. RS Module - Final MBBS Common MCQs
Medicine – RS
1. Regarding GBS (2009.MED.MCQ.01)
A) Type 1 respiratory failure a feature
B) Global reflexes are typical.
C) Distal demyelination is a feature
D) Elevated CSF protein is a feature
E) Treated with IV methylprednisolone.
2. Which is/are correct (2009.MED.MCQ.05)
A) Tidal percussion is positive in a pneumothorax
B) Vocal resonance is reduced in a pleural effusion,
C) Cracked pot sign is seen in large lung cavities
D) Coin sign is seen in a pneumothorax
E) High pitched bronchial breathing is heard over a consolidation
3. Exudative pleural effusions are seen in (2009.MED.MCQ.6)
A) Hypothyroidism
B) Meig’s syndrome
C) Nephrotic syndrome
D) Pancreatitis
E) Pulmonary infarction
4. Regarding diagnosis of pulmonary tuberculosis, (2010.MED.MCQ.05)
A) Positive smear confirms the diagnosis
B) Silver staining of acid-fast bacilli
C) Culture report for AFB takes 6 days
D) Treatment is with IV cefotaxime
E) Positive gamma interferon test is diagnostic
5. A patient presenting with acute exacerbation of asthma can be discharged if (2010.MED.MCQ.06)
A) Stable on oral medication without nebulization during past 48 hours
B) No nocturnal wheezing
C) Normal chest x-ray
D) He is afebrile
E) .
6. Regarding asthma (2011.MED.MCQ.05)
A) Chronic inflammation with predominant neutrophils
B) Nocturnal variation present
C) Bronchodilators beta Agonist effective than ant muscarinic drugs
D) In acute exacerbation usually pco2 is high &
E) In management step 2, add on long acting beta 2 agonist
7. Regarding legionella pneumophila (2011.MED.MCQ.06)
A) Causing diarrhea
B) Lymphocytosis
C) Treated with ceftriaxone

29
D) Causing hyponatremia
E) Present in urine
8. Regarding pneumonia (2012.MED.MCQ.08)
A) .
B) Erythromycin is an alternative for doxycycline in atypical pneumonia
C) Treatment with ciprofloxacin delays the diagnosis of tuberculosis
D) Cough persisting without fever is an indication to change the antibiotics
E) Chest radiography features resolve before the resolution of clinical symptoms
9. Prolonged use of inhaled corticosteroids cause, (2012.MED.MCQ.09)
A) Hoarseness of voice
B) Fetal growth restriction
C) Oropharyngeal candidiasis
D) Diabetes mellitus
E) Sleep disturbances
10. Regarding tuberculosis (2012.MED.MCQ.12)
A) People vaccinated with BCG rarely get TB
B) Isolation of bacteria in sputum is essential for diagnosis
C) .
D) If spine is involved, duration of therapy 9 months
E) It's a notifiable disease in Sri Lanka
11. A 60-year-old previously well male complained of progressive breathlessness, abdominal discomfort and
swelling of the legs. On examination there was gross ascites, elevated JVP, bibasal crepitation and an enlarged
liver. BP 180/80 mmHg, RR - 25/min, PR – 36 BPM. On ECG heart rate was 96 bpm. What is the best
management option? (2009.MED.SBR.26)
A) Digoxin
B) Digoxin + Furosemide
C) Furosemide
D) Abdominal paracentesis
E) Transthoracic pacemaker
12. 68-year-old man presented with 3-month history of hemoptysis and cough. He has LOW and LOA O/E trachea
deviated to L/S, L/upper zone - bronchial breathing and lower zone crepitation. What is the most likely
diagnosis? (2008.MED.SBR.7)
A) Acute collapse of left upper lobe
B) Consolidation of left upper lobe
C) Emphysematous bulla in R/S
D) Fibrosis of L/lung
E) R/S pneumothorax
13. 25-year-old female presented with 3-day history of high fever and R/s pleuritic type chest pain. She has
produced rusty sputum. FBC- Hb. PLT - NL WBC- 19,000 (N-85%). ESR-100 What is the most likely diagnosis?
(2008.MED.SBR.8)
A) Tuberculosis
B) Streptococcus pneumonia
C) Klebsiella pneumonia
D) Aspergillosis

30
E) Staphylococcal pneumonia
14. 68-year-old male presented with past history of TB (completed Treatment with bronchiectasis, this time
presented with fever and cough. Temp - 38.3 C. SpO2 - 88%. what is the best empirical antibiotic for this man?
(2008.MED.SBR.09)
A) Benzyl penicillin
B) Oseltamivir
C) Clarithromycin
D) Tetracycline
E) Ceftazidime
15. A chronic smoker developed cough and progressive SOB over 10 years. He presented with exacerbation of
symptoms for 1 week with productive cough. Physician ordered O2 via a face mask. What is the percentage,
which you would give to the face mask? (2009.MED.SBR.28)
A) 45%BY
B) 35%
C) 60%
D) 80%
E) 100%
16. A 23-year-old man presented with mild fever for 2 weeks. CXR shows a right sided pleural effusion. Full blood
count was done. (2009.MED.SBR.29)

• WBC-8000/mm3
• Neutrophils-60%
• Lymphocytes-40%
• Pleural fluid aspiration shows predominant lymphocytosis.
What is the most likely diagnosis?
A) TB
B) Connective tissue disease
C) Lymphoma
D) Empyema
E) Parapneumonic effusion
17. A 65-year-old male with cough, cold and SOB for 3 years, presented with an acute exacerbation. He was
treated with salbutamol + 0, nebulization. After 4 hours he developed confusion and restlessness. What is the
most appropriate investigation to find out the? cause for his acute confusional state? (2009.MED.SBR.30)
A) FBC
B) CRP
C) CXR
D) CT Brain
E) ABG
18. A 20-year-old girl presented with acute exacerbation of bronchial asthma. Oxygen was given and was
nebulized with salbutamol. But no improvement. Next step in management? (2009.MED.SBR.46)
A) Repeat salbutamol
B) CXR
C) CPAP
D) IV aminophylline
E) IV MgSO4
31
19. 35 years British woman presented to the emergency department. She had 10 hrs. flight from London Colombo
1 week ago. She complains Dyspnea, cough, chest pain during deep breaths, she was healthy prior to the trip.
Her past medical history was uneventful apart from hx of two miscarriage. Most appropriate investigation in
arriving at the diagnosis is, (2011.MED.SBR.25)
A) D-dimers
B) CT- pulmonary angiogram
C) HRCT chest
D) Echocardiogram E
E) PET-CT chest
20. 55 years old male has presented with progressive dyspnea, intermittent cough and on examination there is
clubbing, central cyanosis and bilateral fine crepts. What is the diagnosis? (2011.MED.SBR.26)
A) Allergic Broncho pulmonary Aspergillosis.
B) Bronchiectasis.
C) Chronic bronchitis
D) Asthma
E) Idiopathic pulmonary fibrosis.
21. 40y old male travel guide, presented with progressive SOB for 3 months. Diagnosed DM for 4 years. On
metformin and gliclazide is a teetotaler. On examination mild icterus, ankle edema, shrunken liver, mild
splenomegaly, bilateral fine crepitation. Investigations- hb-10 mcv-84, plt-04, s. Alb- 15, AST-64, alt-84, direct
bilirubin 68, alp-110. CXR upper lobe diversion, cardiomegaly. What is the diagnosis? (2011.MED.SBR.27)
A) Cardiac cirrhosis
B) Haemochromatosis
C) Hep B
D) NASH
E) Wilsons
22. A 50-year-old male from UK visiting Sri Lanka for a short vacation presented with acute onset difficulty of
breathing, cough and left sided chest pain. Well controlled asthma on combined inhalers, smoking 10 pack
years. On examination, no fever, Pulse rate was 120 and Blood pressure 112/72 mmHg, Respiratory
examination was unremarkable. SpO2 was 88%. ECG -Sinus tachycardia with V1-V4 leads T inversions. Full
Blood Count normal. Most probable diagnosis? (2012.MED.SBR.08)
A) Acute exacerbation of asthma
B) Acute exacerbation of COPD
C) Myocarditis
D) Acute coronary syndrome
E) Pulmonary embolism
23. 25-year-old male returned from holiday came with a left sided chest pain for 1-day duration. He had flu for 3
days. The pain was episodic and ceases on expiration. ECG was done. There were ST elevations in anterior and
lateral leads. CRP=30, WBC=11, troponin +. What is the most likely diagnosis? (2012.MED.SBR.13)
A) Acute coronary syndrome
B) Acute pericarditis
C) Pneumothorax
D) Pleural effusion
E) .
24. A patient presenting 4 hours after ingestion of an organophosphate. BP – 80/50, PR – 56, RR – 23, SpO2 – 88%.
On auscultation crackles were heard. Next most appropriate step of management? (2012.MED.SBR.20)
32
A) 250ml 0.9% NaCl
B) Nebulize with salbutamol
C) Oxygen via venturi mask
D) Atropine 3mg IV bolus
E) Pralidoxime 1g IV over 30 minutes

Surgery – RS
1. 25-year-old male is admitted to ER after falling from a tree. He is dyspnoea, tachypnoea and tachycardic. On
examination his blood pressure is 80/45. R/side chest is hyper-resonant and there is reduced air entry on same
side. What is the most appropriate next step in management? (2008.SUR.SBR.04)
A) IC tube insertion
B) Wide bore needle insertion
C) Observe and monitor
D) .
E) .
2. 23 years boy admitted to the causality ward with a run over injury. He is conscious and rational; Chest X ray
shows air fluid level of the left lower hemithorax. After NG tube Insertion, it curved upward to the left lower
thorax What is the most appropriate management? (2008.SUR.SBR.05)
A) Laparotomy
B) Thoracotomy
C) IC Tube insertion
D) Observe and monitor
E) Peritoneal lavage
3. 6 hours after undergoing a thyroidectomy, 36 years old female pt vomited and having stridor. Cause for the
stridor? (2008.SUR.SBR.16)
A) Laryngeal oedema
B) Billy haemorrhage
C) Diaphragmatic breathing
D) Laryngeal oedema
E) Laryngeal spasm
4. A polytraumatic patient 25-year-old male admitted to ETU. He is dyspnoeic, GCS 9/15, brought to hospital. He
is on 50% Oxygen. His ABG results were, pH 7.25, PCO2 90 mmHg, pO2-60 mmHg, HCO3 30, BE 3 mmol/l. What
is the most appropriate management? (2010.SUR.SBR.33)
A) CPAP
B) Endotracheal intubation and ventilation
C) IV 8,4% NaHCO3
D) IV mannitol infusion
E) .
5. A heavy smoker presents with hoarseness of voice for 4 weeks duration. On clinical examination, no thyroid
enlargement and no palpable cervical lymph nodes. What is the best investigation to diagnose the condition?
(2010.SUR.SBR.34)
A) Chest x-ray
B) Laryngoscopy
C) Bronchoscopy
D) CT of the chest
E) US of neck
6. A 26-year-old man admitted following a road traffic accident. He is dyspnoeic. Neck veins are engorged.
Trachea midline. Lungs normal. PR 136 bpm, BP 80/50 mmHg, RR 35/min. What is the most likely condition?
(2010.SUR.SBR.43)

33
A) Cardiac tamponade
B) Flail chest
C) Massive haemothorax
D) Pulmonary contusion
E) .
7. 28-year-old man was admitted to the emergency unit following a fall from height with blunt trauma to the
right chest. PR-120, blood pressure -70/50, Spo2-84%. Hyper resonance over right lung fields with reduced
breath sounds. What is the immediate management? (2011.SUR.SBR.29)
A) FAST scan
B) Wait till ICU available
C) Chest tube
D) Needle thoracostomy
E) Urgent way
8. A 30-year-old man had sustained a flail chest after a blunt injury to chest following RTA. On examination he is
mildly dyspnoeic and maintaining normal Spo2 levels with 02 via nasal cannula. Chest x-ray had found to have
fractured ribs. No other injuries detected. What is the most appropriate management of this patient?
A) Insert an IC drain (2011.SUR.SBR.39)
B) Give analgesia and chest physiotherapy
C) Plan a surgery to correct rib fracture
D) Apply chest corset and Immobilize
E) Endotracheal tube insertion and ventilation
9. Chest injuries that respond well to mechanical ventilation, (2012.SUR.SBR.39)
A) Cardiac tamponade
B) Flail chest
C) Open pneumothorax
D) Pulmonary contusion
E) Tension pneumothorax
10. Indications for thoracotomy in a patient with blunt trauma to the chest (2010.SUR.MCQ.05)
A) 200ml /hour for 4 hours
B) Persistent cardiac tamponade
C) Massive unilateral pulmonary contusion with hypoxia
D) Haemothorax of 1500ml of volume at IC tube insertion
E) Flail chest
11. A 70yr old man presents enlarged hard lymph node in anterior triangle of neck. Biopsy revealed deposit of
squamous cell carcinoma. What are the investigations need to find out primary site?
A) Nasal endoscopy (2010.SUR.MCQ.19)
B) Upper Gi endoscopy
C) CECT thorax
D) Bronchoscopy
E) Serum calcitonin
12. Chest injuries that respond well to mechanical ventilation, (2012.SUR.MCQ.11)
A) Cardiac tamponade
B) Flail chest
C) Open pneumothorax
D) Pulmonary contusion
E) Tension pneumothorax
13. 55-year-old smoker presented with cough, recent weight loss, partial ptosis and left supraclavicular LN
enlargement. What is most the most suggestive chest X-ray feature of malignancy? (2012.SUR.SBR.46)
A) As
34
B) Elevated right hemidiaphragm
C) Widening of superior mediastinum
D) Presence of lung cavitation
E) Left upper lobe collapse
14. Previously healthy 18 months old child stays in a separate room from family members and experiences sudden
onset cough which settles after few minutes. Then child appears normal but has drooling of saliva and refusal
to feed. What’s the most possible diagnosis? (2012.SUR.SBR.47)
A) Epiglottitis
B) Foreign body in piriform fossa
C) Foreign body in airway
D) Foreign body
E) .

Paediatric – RS
1. 1-hour old baby is in respiratory distress. On auscultation heart sounds were heard on Right side. Abdomen is
scaphoid. Spo2 87%. What is the management? (2008.PAED.MCQ.1)
A) O2 via face mask.
B) Bag and mask ventilation
C) Intubation and ventilation
D) Needle thoracentesis
E) Reassure
2. 5 yr old girl with poorly controlled bronchial asthma, on beclomethasone 100 micro grams bd &
bronchodilators via DP inhaler has poorly controlled asthma. She has a cat at home. What would you do next
to improve her control? (2008.PAED.MCQ.4)
A) Check inhaler technique.
B) Increase beclomethasone dose.
C) Avoid contact with cat.
D) Combine with long acting beta 2 agonist.
E) Change to a MDI with spacer.
3. A 3yr old girl with sudden onset difficulty in breathing is rushed to the ETU. She was playing in the garden with
beads prior to this incident. She was noted to be coughing vigorously on arrival. What is the immediate
management step? (2008.PAED.MCQ.5)
A) Do a blind sweep of the mouth.
B) Encourage coughing
C) Give S chest thrusts.
D) Give 5 back blows.
E) Give high flow 02
4. A 3 days old child presents with SOB. saturation of PaO2 in room air is 80%, What are the possibilities?
(2009.PAED.MCQ.3)
A) Persistent pulmonary hypertension
B) Coarctation of aorta
C) Eisenmenger syndrome
D) Tetralogy of Fallot
E) Transposition of great vessels
5. Features of acute life-threatening asthma (2009.PAED.MCQ.4)
A) Heart rate > 120bpm
B) Cyanosis
C) Pulsus paradoxus
35
D) .
E) .
6. Direct tissue involvement in Mycobacterium tuberculosis is seen in (2009.PAED.MCQ.11)
A) Adenoma sebaceum
B) Erythema nodosum
C) Lupus vulgaris
D) Phlyctenular conjunctivitis
E) Scrofuloderma
7. Side effects of aminophylline (2009.PAED.MCQ.12)
A) Convulsions
B) SVT
C) Hypokalemia
D) Hyperglycaemia
E) Vomiting
8. A child with pneumonia should be admitted to hospital if (2011.PAED.MCQ.1)
A) Age <6 months
B) Infected by an atypical organism
C) If there is no response to oral antibiotics therapy after 48hrs
D) Features of multiple lobe involvement
E) He/she is immunocompromised
9. Regarding pneumonia (2012.PAED.MCQ.3)
A) Mycoplasma pneumonia cause extra pulmonary manifestations.
B) Virus is the commonest aetiology for children more than 5years
C) Staphylococcus pneumonia is associated with metastatic abscess
D) Strep pneumoniae leads to invasive disease
E) H. Influenzae infection commonly leads to pneumatocele
10. Neonate at term becomes apnoeic PR 100 bpm. What is the immediate action? (2008.PAED.SRB.22)
A) Transfer to ICU
B) Oxygen via head box
C) Give inflation breaths
D) Chest compressions
E) Intubate and ventilate
11. You were called to see a baby who was born 1hour ago by normal vaginal delivery at term. The baby was in
respiratory distress. His respiratory rate was 70 and had nasal flaring grunting and marked intercostal
recessions. Heart sounds were heard on the right side, Abdomen was scaphoid, Sp02 was 91% on room air.
What is the next best step in the management? (2009.PAED.SRB.21)
A) Commence broad spectrum IV antibiotics
B) Insert an NG tube
C) Give oxygen via nasal prongs
D) Insert an oropharyngeal airway
E) Intubation and ventilation
12. You are attending a baby delivered at 38 weeks by LSCS due to fetal distress. Liquor is not meconium stained.
Baby is floppy and having poor respiratory effort and bradycardia. What is the next step in management?
(2009.PAED.SRB.22)
A) Give inflation breaths
B) Give ventilation breaths
36
C) Give adrenaline
D) Give chest compressions
E) Intubation
13. A 12-year-old boy was brought from an estate sector hospital. He had a cough for 2 weeks despite of 1-week
antibiotic treatment. On examination he had a moderate pleural effusion. Pleural aspiration was done. Protein
60(20-40) Glucose 10 (30-50) Neutrophils 10% Lymphocytes 88% Which investigation is needed to confirm the
diagnosis (2009.PAED.SRB.25)
A) Adenosine deaminase
B) Mantoux test
C) Pleural aspiration culture for Mycobacterium tuberculosis
D) Pleural fluid direct smear
E) Chest x ray
14. A 10-year-old boy presented with cough, high fever for 10 days with recent onset retrosternal chest pain. He
is ill and has increased work of breathing He has an inspiratory stridor. Chest xray is unremarkable Most likely
diagnosis? (2009.PAED.SRB.26)
A) Bacterial tracheitis
B) Croup
C) Epiglottitis
D) Retropharyngeal abscess
E) Spasmodic croup
15. A 10-year-old presented with a 1-week history of cough and fever. Examination was not significant L 88% N
10% Hb 9.2g/dl Most likely diagnosis? (2009.PAED.SRB.36)
A) Pertussis b
B) Mycoplasma pneumonia
C) Lobar pneumonia
D) Tuberculosis
E) IMN
16. A 3 yr old boy who had difficulty in breathing was admitted to OPD. Al emergency treatment unit initial
evaluation was done with nebulization of salbutamol. Child did not respond. His saturation remains 89% with
high flow 02 after back to back nebulization. Next step? (2009.PAED.SRB.44)
A) IV Salbutamol
B) CXR
C) Admit to ICU
D) Repeat salbutamol nebulization
E) IV Mg SO4
17. 6-year-old boy admitted with low grade fever and cough for 2 days duration. Cough was dry and paroxysmal
and associated with wheezing. Also complained of headache and body aches. CXR-B/L Hilar shadows, WBC -
7600, N% 54, ESR-30, CRP- 12. What is the most likely aetiological factor? (2010.PAED.SRB.25)
A) Haemophilus influenzae
B) Mycobacterium tuberculosis
C) Mycoplasma pneumoniae
D) Pneumocystis jirovecii
E) Streptococcus pneumoniae
18. A child has presented with high fever, cough and cold for 4 days. Following admission, he developed a
maculopapular rash behind his right ear. What is the most likely diagnosis? (2010.PAED.SRB.35)
A) Infectious episcleritis
B) Measles
37
C) Parvo virus infection
D) .
E) .
19. A 6-year-old boy admitted with acute severe asthma. For the last month he has had two nebulization at OPD
visits. He is on beclomethasone 400 mcg bd. He has frequent night symptoms. What would you give him in
discharge? (2010.PAED.SRB.36)
A) Beclomethasone 800 mcg with spacer
B) Beclomethasone 800 mcg with spacer and mask
C) Oral prednisone
D) Salmeterol and fluticasone MDI with spacer
E) Beclomethasone DPI
20. 18-month-old baby presented with a 2 days' history of coryzal symptoms, fever & cough. On examination
found to have a stridor. What is the immediate management option? (2010.PAED.SRB.40)
A) Nebulize with adrenalin
B) Nebulizer with Budesonide
C) Nebulizer with ipratropium Bromide.
D) Humidified oxygen
E) Nebulize with Salbutamol
21. 6-month-old child who was diagnosed to have TOF presented with dyspnoea, wheezing, rhinorrhoea. CXR
shows B/L patchy consolidations. He was not improved despite treatment with IV Cefotaxime and IV
Gentamicin. Next best treatment, (2010.PAED.SRB.41)
A) Intubate and ventilate
B) IV Morphine
C) Sputum culture
D) Treat with Oseltamivir
E) Urgent BT shunt
22. 3-year-old previously treated for right lower lobe pneumonia 3 months back. X-ray not resolved. Now
complaints of persistent cough. What is the best investigation for the diagnosis? (2010.PAED.SRB.46)
A) 24hr PH
B) Sputum culture & ABST
C) Mantoux
D) Immunoglobulin testing
E) Sweat chloride
23. 3kg term baby born by a spontaneous NVD was noted to have a RR of 80 breaths/min after 3 hrs. He also had
chest Indrawing & grunting. Membranes were ruptured 24 hrs before delivery. Mother was healthy. She
attended antenatal care regularly and had an uncomplicated pregnancy with normal USS. Probable diagnosis?
(2011.PAED.SRB.21)
A) Diaphragmatic hernia
B) Congenital pneumonia
C) Meconium aspiration xd
D) Surfactant deficit lung disease
E) TTN
F) Place head in neutral position
24. 1-hour old child presented with apnoea and cyanosis. Child is pink while crying and other examination findings
are normal. Septic screening ABG & cranial USS are normal. Baby was found to have cyanosed with spO2 of
66% whilst sleeping. He starts to cry & become pink when oxygen is administered. What is the most likely
cause for the above presentation? (2011.PAED.SRB.22)

38
A) Choanal atresia
B) Congenital heart disease
C) GORD
D) Neonatal seizures
E) Tracheoesophageal fistula
25. A 3-year-old episodic Wheezer who is not on long term inhalers presented with recurrent wheezing attacks.
He has developed day and night symptoms at least 3 days per week. What is the best next step of
management? (2011.PAED.SRB.27)
A) Ask to take salbutamol inhalers where necessary
B) Combined metered dose Inhalers
C) Commence leukotriene modifiers
D) Montelukast
E) Review with symptom diary
26. A 10-year-old boy presented following an acute episode of asthma which needed IV salbutamol for
management. He experiences frequent night symptoms and needed 4 nebulizations in last 6 months. What is
your management at discharge? (2011.PAED.SRB.28)
A) Advice to use salbutamol inhaler when needed
B) Start salmeterol + fluticasone MDI
C) Start a mast cell stabilizer
D) Oral prednisolone for 5 days
E) .
27. 10 Yr acute ex of asthma didn't improve with nebulization. Needed IV salbutamol. Past 6 months 4 wheezing
attacks requiring nebulization Cough early in morning on most of days. Most appropriate discharge plan is
(2011.PAED.SRB.29)
A) .
B) .
C) .
D) .
E) .
28. A 6 months old baby presents to the emergency department following a prolonged seizure. His heart rate is
55 bpm and his respiratory rate is 40/min. He also made some gurgling sounds while breathing Which of the
following is the next Immediate step in management? (2011.PAED.SRB.43)
A) Commence bag and mask ventilation
B) Oxygen therapy via nasal prongs
C) Place in neutral position
D) Suck out secretions
E) .
29. Baby born by NVD was pale and floppy at birth. No respiratory effort. HR 20/min. Five inflation breath given
with adequate chest expansion. Baby was reassessed and there was only occasional gasping, HR 40/min. What
is the most appropriate next step in management? (2012.PAED.SRB.21)
A) Give 15 ventilation breaths
B) IV adrenaline infusion
C) Ventilation with bag and face mask.
D) Give 5 inflation breaths with jaw thrust
E) Intubate and ventilate

39
30. 28 months old premature baby whose birth weight is 1.3kg ventilated due to respiratory distress extubated
after 3 days. Next day his BP was 80/40 heart was 130bpm. What is the most appropriate management option?
(2012.PAED.SRB.24)
A) Take blood culture and start IV antibiotics.
B) Intravenous prostaglandin infusion
C) Oral paracetamol
D) Repeat dose of surfactant
E) .
31. 2 days old boy admitted to a peripheral hospital with poor feeding. O/E he is dyspneic. RR 80 per min. PR
170bpm. Blood pressure in upper limb 80/55 mmHg. Lower limb – 55/30 mmHg. SpO2 - 94%. What is the
immediate management of this child? (2012.PAED.SRB.25)
A) Prepare for intubation and ventilation
B) High flow oxygen via nasal prongs
C) Give 10ml/kg 0.9% normal saline bolus
D) IV prostaglandin infusion
E) Transfer to cardiac specialist unit for surgery
32. Child presented with a stridor while crying since 3 weeks of birth. His feeding is satisfactory. No other
abnormalities detected on examination except for an inspiratory stridor. Most appropriate management
option (2012.PAED.SRB.26)
A) Make arrangements for micro laryngoscopy
B) Nebulise with Budesonide
C) Reassure the mother
D) Nebulise with Dexamethasone
E) Arrange lateral neck X-ray
33. 7 years old girl presented with an acute attack of wheezing. Nebulized 2 time with Salbutamol within last 1
hour in OPD. On admission RR- 50 per min, HR – 110, air entry equal, vesicular breathing with prolonged
expiration, SpO2 – 94%. Next step of management? (2012.PAED.SRB.27)
A) Commence MgSO4 infusion
B) Commence Aminophylline
C) Oxygen via face mask
D) Nebulise with Ipratropium bromide
E) Repeat Salbutamol nebulization
34. 2-month old baby presents with spasmodic cough and post-tussive vomiting for 1 week. On examination baby
is not tachypnoeic. CXR is normal FBC WBC 18×109/L Hb 11g/dl Plt 190×109/L CRP 14mg/l Most likely diagnosis
(2012.PAED.SRB.35)
A) Bronchiolitis
B) Infantile wheezing
C) Gastro oesophageal reflux
D) Whooping cough
E) Viral pneumonia

40
5. NGD Module - Final MBBS Common MCQs
Paediatrics – Nutrition
1. 8.5 yr old girl from Medawachchiya. Mother complains she is clumsy. During the latter part of the day she falls
knocking on furniture and objects fall from her hand. Has a greyish lesion on her conjunctiva. What is the
management? (2008.PAED.MCQ.06)
A) Dietary modifications
B) Estimate retinol levels in blood
C) Give Vitamin A mega dose
D) Reassure
E) MRI scan of her brain
2. Causes for oral pigmentation, (2009.PAED.MCQ.08)
A) Addison's disease
B) Pellagra
C) Peutz-Jeghers syndrome
D) Crohn's disease
E) Fanconi anaemia
3. Which of the following is/are feature/s of kwashiorkor? (2009.PAED.MCQ.10)
A) Flaky paint dermatosis
B) Hypokalaemia
C) Increased appetite
D) Oedema
E) Apathy
4. Anorexia nervosa (2009.PAED.MCQ.17)
A) hypokalaemia is a feature
B) Girls are affected predominantly
C) Affected individuals like to cock
D) Basal metabolic rate is low
E) Vomiting is a feature
5. An 18-month-old girl presents with an LRTI. Nutritional assessment shows weight for height. below-3SD. Most
appropriate next step? (2009.PAED.SBR.33)
A) Start BP 100
B) IV 10% glucose
C) Liquid diet with milk
D) Multivitamin drops
E) Thriposha
6. A 6-year-old boy was referred by school medical officer due to short stature and limb deformities. O/E there
are Harrison sulcus, genu varus deformity and mild pallor. No organomegaly Serum Ca - 8 (9-10.5) Serum
phosphates - 6.2 (2.5-4.5) ALP - 1800 (<400) What is the likely condition? (2009.PAED.SBR.48)
A) Blount disease
B) Nutritional rickets
C) Osteoporosis
D) Renal osteodystrophy
E) Vitamin D resistant rickets

41
7. In comparison with a term baby, a preterm baby (2010.PAED.MCQ.02)
A) Has a higher risk of developing intracranial haemorrhages
B) Is at a higher risk of getting iron deficiency
C) Is more prone to get hypothermia
D) Needs more ml per kilogram of fluid
E) is at a higher risk of developing osteopenia
8. Features of iron deficiency anaemia, (2010.PAED.MCQ.09)
A) Cognitive impairment is associated
B) .
C) .
D) .
E) .
9. Compared to cow's milk, breast milk has (2010.PAED.MCQ.10)
A) More iron
B) More protein
C) High whey: casein ratio
D) Low sodium
E) Low pufa
10. Regarding Severe Acute Malnutrition (2011.PAED.MCQ.10)
A) Hypothermia is a known feature
B) Therapy starts with f100
C) Weight for height is <-3SD
D) Oedema is due to heart failure
E) Haemolytic uremic syndrome HUS Is a life-threatening condition
11. 15-year-old girl was referred from the school medical inspection for further evaluation of obesity. Her weight
is 60 kg which is >94th centile and her height is 140 cm which is <3 rd centile. She is frequently bullied by her
schoolmates due to being fat and slow in her activities. What is the initial management? (2011.PAED.SBR.37)
A) Commence on metformin
B) Dietary advice on reducing oil and sweets
C) .
D) .
E) .
12. Regarding breast milk (2012.PAED.MCQ.10)
A) Sterile solution
B) Production is stimulated by posterior pituitary gland
C) Composition changes with the period of gestation at birth
D) Is a rich source of Vitamin K
E) Has a protective effect against autoimmune disease
13. A mother came with a 3 days old baby, complaining of unable to breastfeed her baby and worrying due to
reduced UOP of baby. On admission both her nipples were red and cracked. What is the most suitable step
according to her situation? (2012.PAED.SBR.35)
A) Advice on starting formula feeding
B) Apply a nipple shield and continue breast feeding
C) Commence IV fluids
42
D) Give antibiotics to the mother
E) Express breast milk and do cup feeding
14. A 2-year-old girl with cough and fever for 5 days, her mother complaints that she has poor appetite and on
examination the child is lethargic, pale, and weight below -3SD and Hb 9 g/dl. What is the most appropriate
nutritional support? (2012.PAED.SBR.36)
A) Add high protein diet
B) BP 100
C) F 75
D) Triposha
E) .

Paediatrics – Growth
1. Complications of prematurity (2009.PAED.MCQ.01)
A) Bronchopulmonary dysplasia
B) Meconium aspiration syndrome.
C) Intraventricular haemorrhage.
D) Erb's palsy
E) Paraventricular haematoma
2. Regarding constitutional delay of growth (2009.PAED.MCQ.07)
A) Bone age is retarded
B) Hypothyroidism is a causative factor
C) Puberty is delayed
D) Final adult height is less than expected
E) Overweight is a factor
3. Regarding anthropometric measures (2012.PAED.MCQ.07)
A) Upper body: lower body ratio - 1.3:1 is normal in a 7-year-old child
B) Shakir tape is used for measure mid arm circumference
C) Waist circumference more than 90th centile according to the age and sex is a feature of metabolic
syndrome
D) Height is measured 6 monthly
E) OFC measurement doesn’t have a value in patients with craniosynostosis
4. 11-Year-old boy presented with short stature to paediatric clinic. His height for age Is < 250. Height velocity is
normal. His bone age & height age is equal, but less than chronological age What is the probable diagnosis?
(2008.PAED.SBR.08)
A) Chronic malnutrition
B) Constitutionally small
C) Familial
D) Hypothyroidism
E) Silver Russell dwarfism.
5. A child presented to the paediatric clinic with height below 3SD, weight median to -150, weight for height
above median, Parents are normal in height. Most likely diagnosis? (2009.PAED.SBR.31)
A) Constitutional delay
B) Malabsorption
C) Prolonged breastfeeding
D) GH deficiency
E) Inadequate diet

43
6. 9 yr old girl with hypothyroidism is on thyroxine 50 microgram/day. She is having constipation. Her
grandmother worried that her school performance is poor she is bullied at school for short stature. Her T4 is
2.6 (2.1-2 8) and TSH is 15 (0.5-5.0). What is the next step of her management? (2009.PAED.SBR.39)
A) Assess bone age
B) Assess compliance
C) Do antithyroid antibody
D) Look for malabsorption
E) Repeat T4, TSH
7. In a temporary settlement camp for displaced persons the MOH of the area is expected to assess the <5-year
population, for the assessment of nutritional status. What is the best anthropometric measure for this
screening? (2010.PAED.SBR.30)
A) Chest circumference
B) Height or length
C) Mid arm circumference
D) Skin fold thickness
E) Weight
8. 4-year-old boy with bowlegs. X rays show features of rickets. Calcium level 2.05, Phosphate -2.42, alkaline
phosphatase - 1850. Next step of management? (2010.PAED.SBR.45)
A) Do USS abdomen
B) Treat as nutritional rickets.
C) Arrange an orthopaedic referral.
D) Do a blood gas analysis.
E) Request serum creatinine levels.
9. 10yr old boy with short stature, no dysmorphic features, upper segment to lower Segment body ratio is
appropriate to the age. What is the next step in management? (2011.PAED.SBR.34)
A) GH assay
B) Mid parental height
C) Assess bone age
D) Request for serum IGF level
E) Request thyroid function test
10. 14-year-old girl present to paediatrics clinic due to short stature. She is the shortest one in the class since
preschool. Her learning is average. On examination she is on prepubertal stage. Which of the investigation
most likely to come to diagnosis.? (2012.PAED.SBR.34)
A) CT brain
B) FSH
C) Karyotype
D) Free T4
E) USS of thyroid
11. A 13-year-old boy with weight for height more than 3SD, and height for age less than 2SD. Delayed bone age
presented with BP 130/80. What is the most likely diagnosis? (2012.PAED.SBR.45)
A) Cushing’s syndrome
B) Growth hormone deficiency
C) .
D) .
E) .

44
Paediatrics – Development
1. A child presents with speech delay. He plays alone. He has only 5 functional words. Rest of the development
milestones are age appropriate. His mother complains that he gets angry easily without a reason and the
tantrums which last a long time. What is the most appropriate management? (2009.PAED.SBR.46)
A) Treat as MR
B) Refer to a specialist
C) Explain to parents that he will not be able to perform normal activities in future
D) Reassure and evaluate after one month
E) Advice to admit the child to a childcare centre
2. Regarding child development (2010.PAED.MCQ.07)
A) Prematurity needs correction up to 2 years
B) Gross motor development occurs in cranio caudal direction
C) Development delay in more than 2 domains is known as global development delay
D) CHDR is an effective tool in assessing development
E) Development level of -1SD defines development delay for that domain
3. T/F regarding intelligence quotient (IQ), (2010.PAED.MCQ.18)
A) Determined by an intelligence test.
B) Mean IQ has changed over time.
C) IQ is an objective measurement.
D) Majority have an IQ between 85 - 115.
E) A person with IQ of 140 considered as a genius.
4. An 18-month-old child can, (2011.PAED.MCQ.07)
A) Build a tower of 2-3 blocks
B) Help with dressing
C) Kick a ball
D) Walk unaided
E) .
5. Bed wetting in children (2012.PAED.MCQ.5)
A) Should be assessed if persisting beyond 5 year of age
B) Is associated with chronic constipation
C) Is precipitated by stressful life event
D) Responds to behavioural therapy effectively
E) Best treated with Desmopressin
6. 16-month-old Infant was bought to the clinic who is unable to walk. He had his head control at 3 months of
age. Sat with support by 10 months. Now he can sit and move around in seated position. He can catch the
pencil and scribble and speak only 5 words. What is the most appropriate management? (2008.PAED.SBR.09)
A) Do an USS of hip
B) Commence physiotherapy
C) Perform nerve conduction test
D) Perform thyroid function test
E) Reassure the mother that child has no neurological abnormality
7. 4yr old girl was taken to the paediatric clinic by her mother, complaining of speech delay. She was otherwise
healthy. Mother noted Indifference when talking with her. She was always trying to be alone, not interested
in playing with her siblings or peers. At clinic it was noticed that she has a downward gaze and was looking at
her fingers and playing with them. What can be the condition the child has? (2008.PAED.SBR.24)
A) Autism spectrum disorder.
B) Degenerative brain disorder.
C) Mental retardation
D) Tics

45
E) ADHD
8. 15-month-old boy presented with delay in walking. He was born in an uncomplicated breech delivery and had
achieved initial milestones. He can stand up with support but cannot walk with support. He bottom shuffles.
He can speak meaningful words and do circular scribbling. He shows interactive play. What is the most
appropriate management? (2011.PAED.SBR.35)
A) Explain and reassure the parents and review in three months’ time
B) Do a nerve conduction study
C) Do brain imaging studies
D) Refer for orthopaedic opinion
E) Start him on physiotherapy
9. 3 years old preschool child was brought by her parents who are concerned about her speech. She has 8-10
words with meaning. She can run & climb steps unaided. She is a well grown active child with good
interactions. The most appropriate step in mx is (2011.PAED.SBR.48)
A) Refer for audiometry assessment
B) Refer for formal development assessment
C) Refer to psychologist
D) Refer to neurologist
E) Refer to occupational therapist
10. 15-month old boy delivered by breech extraction. He is a bottom shuffler. Can stand alone but can't walk
without support. Other developments are normal. What is the most appropriate next management option?
(2012.PAED.SBR.34)
A) Arrange neuroimaging
B) Reassure the mother
C) Review at 18 months of age
D) Refer him to orthopaedic opinion
E) Refer him to for physiotherapy
11. A 4-year-old girl was presented by her parents and complain that the child likes to play alone and doesn’t
interact with peers. He has abnormal hand movements that disappear while he is sleeping. On examination
he has variable tone and involuntary movements in his limbs. The most probable diagnosis is
(2012.PAED.SBR.49)
A) Athetoid cerebral palsy
B) ADHD
C) Autism spectrum disorder
D) Emotional deprivation
E) Hearing impairment

46
6. GIT Module - Final MBBS Common MCQs
Medicine – GIT
1. 68-year-old patient is on warfarin presented with hematemesis. His INR is 9. What is the most appropriate
management? (2008.MED.SBR.10)
A) Cryoprecipitate
B) FFP
C) IV vitamin K
D) Omeprazole
E) Endoscopic banding
2. 28-year-old male presented with watery diarrhea for 6 weeks duration with loss of weight & LOA, abdominal
pain and low-grade fever, O/E there was a right iliac fossa mass. What is the most suitable diagnosis,
(2008.MED.SBR.11)?
A) Coeliac disease
B) Giardiasis
C) Intestinal TB
D) Irritable bowel syndrome.
E) Ulcerative colitis
3. 54-year-old known patient with alcoholic cirrhosis, presented with altered sleep pattern, ascites and ankle
oedema. O/E his abdomen is tensed and tender. FBC - Hb-11g/dl, WBC -15,000(N-80%), PLT 150,000. Ascitic
fluid-450/mm3 of neutrophils. What is the best management for this patient? (2008.MED.SBR.12)
A) Cefotaxime
B) Frusemide
C) Salt poor plasma
D) Large volume paracentesis
E) Low salt diet
4. Which of the following investigations are correctly matched with their cause for chronic liver cell disease?
(2009.MED.MCQ.07)
A) Primary biliary cirrhosis - anti ds DNA
B) Auto immune hepatitis - anti smooth muscle antibody
C) Wilson's disease - 24-hour urinary copper excretion
D) Hepatitis B - HBsAg
E) Hemochromatosis -24-hour hemosiderin excretion
5. Causes of cirrhosis, (2009.MED.MCQ.08)
A) Hepatitis E
B) After acute poisoning of methyl alcohol
C) Asymptomatic gall stones without common bile duct obstruction
D) NASH
E) Portal vein thrombosis
6. Metabolic acidosis is seen in (2009.MED.MCQ.10)
A) Severe vomiting
B) Severe diarrhea.
C) Ketosis
D) Diuretic abuse
E) Treatment with acetazolamide
7. A 50-year-old patient with cirrhosis presented with variceal bleeding, He was managed with octreotide and
banding What is the most likely measure to prevent further variceal bleeding? (2009.MED.SBR.31)
A) Interval band ligation
B) Oral proton pump inhibitors
47
C) Interval sclerotherapy
D) Oral nitrates
E) Oral beta blockers
8. What is the best management option for hepatorenal syndrome? (2009.MED.SBR.32)
A) Reduce diuretic therapy
B) IV fluid resuscitation with normal saline
C) IV terlipressin
D) Repeated large volume paracentesis
E) IV ceftriaxone
9. Regarding fat malabsorption, (2010.MED.MCQ.07)
A) Vitamin deficiency is uncommon
B) Occurs in bacterial overgrowth
C) Occurs in obstructive jaundice
D) Diagnosed when faecal fat content > 7 g in 24 hours
E) 90% of exocrine pancreas function should be before fat malabsorption occurs
10. Regarding inflammatory bowel disease, (2010.MED.MCQ.08)
A) Crohn's disease is commonly seen in rectum
B) Ulcerative colitis causes transmural ulceration of the bowel wall
C) Intravenous steroids are indicated in acute severe ulcerative colitis
D) Colectomy will cure Crohn's disease
E) Granuloma on histology is characteristic of Crohn's disease
11. 46 years old man with an alcoholic cirrhosis presented with progressive abdominal and ankle oedema while
he is taking spironolactone 100mg and frusemide 40mg. On examination, he is icteric, not pale, pitting ankle
oedema up to knee, no flapping tremors. WBC 12x10/L, Neutrophil 85%, CRP 24, WBC on ascitic fluid 450.
What is the first management option? (2010.MED.SBR.33)
A) Salt free albumin.
B) IV ceftriaxone.
C) Increase dose of diuretics.
D) Large volume paracentesis.
E) Low salt diet.
12. 30 years old female presents with excessive pruritus and abdominal distension. On examination there is free
fluids in abdomen, and she is deeply icteric. Her Hb 9g/dl, MCV 104, WBC 3000/mm, PLT-110x10'/mm3, INR
3, ALT 159.What is the best next investigation to diagnose her condition? (2010.MED.SBR.34)
A) Upper Gi endoscopy.
B) Serum bilirubin.
C) Serum ferritin.
D) Serum copper.
E) Ultrasound abdomen.
13. Regarding hepatic encephalopathy (2011.MED.MCQ.07)
A) Increase serum Ammonium level required for diagnosis
B) Infection is a precipitating factor
C) Strait protein restriction is needed until full recovery of consciousness
D) Lactulose not given in unconscious patients
E) Rifaximin is a secondary prophylaxis
14. 40y old male travel guide, presented with progressive SOB for 3 months. Diagnosed DM for 4 years. On
metformin and gliclazide is a teetotaler. On examination mild icterus, ankle oedema, shrunken Iiver, mild
splenomegaly, bilateral fine crepitations. Investigations- hb-10 mcv-84, plt-04, serum Alb- 15, AST-64, alt-84,
direct bilirubin 68, alp-110. CXR upper lobe diversion, cardiomegaly What is the diagnosis? (2011.MED.SBR.27)
A) Cardiac cirrhosis
48
B) Haemochromatosis
C) Hep B
D) NASH
E) Wilsons
15. 50-year-old lady presented with diarrhoea for 6-month duration. Also complaining of difficulty in getting up
from squatting position. She feels difficulty in walking at night. On examination she is pale, tenderness over
the thigh, multiple bruises. What is the most appropriate investigation to arrive at diagnosis?
(2011.MED.SBR.28)
A) CECT abdomen
B) Endoscopy and biopsy
C) FBC
D) Stool full report.
E) USS abdomen
16. Regarding ascites (2012.MED.MCQ.17)
A) More than 500ml causes shifting dullness
B) More than 100ml can be seen in USS
C) Polymorphs 250/μl is diagnostic of bacterial peritonitis
D) Adenosine deaminase will increase in TB peritonitis
E) Portal hypertension has a SAAG score more than 1.1
17. Regarding gastric mucosa (2012.MED.MCQ.18)
A) Secreted by G cells of the stomach
B) Acetylcholine stimulates gastrin secretion
C) Ranitidine blocks the action of H+ / K+ ATPase
D) Helicobacter pylori stimulates gastrin secretion
E) Gastric acid secretion is inhibited by prostaglandin E2
18. Regarding hepatitis virus serology (2012.MED.MCQ.19)
A) A positive Hepatitis B Surface antigen indicates active infection
B) IgM antibodies to hepatitis B core antigen is positive in acute infection
C) Hepatitis B surface antigen and anti HBs antibodies can co-exist
D) Presence of Hepatitis B e antigen is indicative of infectivity
E) Anti Hbs antibody in blood shows presence of immunity
19. A 35-year-old man with chronic bloody diarrhea. Which of the following favor the diagnosis of Crohn’s disease
over ulcerative colitis? (2012.MED.SBR.10)
A) Pancolitis
B) Perianal abscess
C) Past history of perianal fistula
D) Elevated fecal calprotectin
E) Positive family history
20. 35-year-old female presents with icterus and generalized pruritus. Her INR is 2.5, total bilirubin 2.5 and ALP
800. USS is suggestive of CLCD. Best investigation to check for etiology? (2012.MED.SRB.14)
A) anti-mitochondrial antibody
B) hepatitis B surface antibody
C) Hepatitis C antibody
D) Serum Ceruloplasmin level
E) Anti- smooth muscle antibody
Surgery – GIT
1. Pancreatic Carcinoma (2009.SUR.MCQ.03)
A) Chronic pancreatitis is a risk factor
B) Adeno carcinoma is the commonest type
49
C) Head of the pancreas is the commonest site
D) Survival rate is 50%
E) .
2. Tests to diagnose H.pylori (2009.SUR.MCQ.04)
A) Serology
B) Faecal antigen test
C) Nitrogen breath test
D) Rapid urease test
E) Histology
3. Conditions associated with multiple perianal sinuses and fistulas, (2009.SUR.MCQ.07)
A) Actinomycosis
B) Crohn's disease
C) Anal fissures
D) Hidradenitis suppurativa
E) TB
4. Regarding gallstones, (2009.SUR.MCQ.08)
A) Calcium bilirubinate is the predominant constituent of the mixed stones
B) Pigment stones are caused due to haemolytic anaemia
C) Majority of stones are radiolucent
D) Majority of stones are pigmented stones Majority are of mixed type
E) .
5. Regarding acute pancreatitis, (2009.SUR.MCQ.09)
A) Glasgow Irwin score is used to indicate for ICU admission
B) .
C) .
D) .
E) .
6. A 50-year-old female presented with massive bleeding per rectum. What are the possible causes
(2009.SUR.MCQ.12)?
A) Diverticular disease of colon
B) Duodenal ulcer
C) Carcinoma of the caecum
D) Angiodysplasia of the colon
E) ileocecal intussusceptions
7. Regarding salivary glands, (2009.SUR.MCQ.15)
A) Stones are commonest in the submandibular gland
B) Marginal mandibular nerve can get damaged in submandibular sialadenectomy
C) Acute parotitis is commonly caused by Staphylococcus aureus
D) Commonest tumor is Warthin's tumor
E) Young women commonly get adenolymphoma of parotid
8. Which of the following are used to stage oesophageal carcinoma? (2010.SUR.MCQ.01)
A) CECT abdomen and thorax
B) Endoscopic ultrasound
C) MRI of abdomen and thorax
D) PET CT
E) Upper GI endoscopy
9. 30-year-old man presents with altered bowel habits associated with blood and mucus. Which of the following
features favour a diagnosis of Crohn's disease over Ulcerative colitis? (2010.SUR.MCQ.02)
A) Episodes of intestinal obstruction
50
B) Polyarthralgia
C) Recurrent abdominal pain
D) Anal fissures
E) Recurrent UTI
10. Premalignant lesions of colon are, (2010.SUR.MCQ.03)
A) Crohn's disease.
B) Diverticular disease
C) Ulcerative colitis.
D) Peutz Jegher syndrome.
E) Serrated adenoma
11. Acute anal fissure, (2010.SUR.MCQ.04)
A) best diagnosed by proctoscopy
B) lateral internal sphincterotomy is performed to treat intractable fissures
C) local application Nitroglycerine gel is used in treatment
D) Majority recurs following conservative management
E) Mostly located at 6 O' clock position of the anal canal
12. Regarding acute cholecystitis (2010.SUR.MCQ.05)
A) Cholecystectomy not done in the first 48 hours
B) jaundice is a characteristic feature.
C) May occur even in absence of gallstones
D) Pericolic fluid in ultrasound scan confirm the diagnosis
E) ERCP done
13. Regarding carcinoma of the pancreas, (2010.SUR.MCQ.06)
A) Carcinoma in the body pancreas presents later than the head of pancreas cancer
B) Endoscopy US used in staging
C) Ductal adenocarcinoma is more common in the head of pancreas than the body of pancreas
D) Histological diagnosis is mandatory prior to surgical resection.
E) Pancreatic body tumours treated with Whipple's procedure
14. Investigations in acute abdominal pain, (2010.SUR.MCQ.13)
A) A supine abdominal x-ray to diagnose acute intestinal obstruction
B) Erect chest x-ray to see intraperitoneal gas leakage
C) Contrast CT abdomen should be done in a patient suspected to have a ureteric colic.
D) Ultrasonography is the 1st line imaging modality in a patient with suspected rupture of aortic aneurism.
E) X-ray KUB should be done immediately in a patient with a ureteric colic.
15. 58 years old male is planned to undergo an inguinal hernia repair. What factors would prevent him
undergoing a day surgery? (2010.SUR.MCQ.15)
A) Uncontrolled DM
B) Obesity BMI > 30 kg/m?
C) History of smoking.
D) Live alone at home.
E) Body mass index >30kg/m?
16. A 70yr old man presents enlarged hard lymph node in anterior triangle of neck. Biopsy revealed deposit of
squamous cell carcinoma. What are the investigations need to find out primary site? (2010.SUR.MCQ.19)
A) Nasal endoscopy
B) Upper GI endoscopy
C) CECT thorax
D) Bronchoscopy
E) Serum calcitonin
17. 69-year-old male comes with progressive dyspnoea and has undergone Barium swallow due to difficulties at

51
the upper GI endoscopy. Which of the following finding would likely to indicate carcinoma of the
oesophagus? (2010.SUR.MCQ.20)
A) Narrowing of the oesophagus with bird beak appearance
B) Dilation of the upper oesophagus
C) Persistent filling defect in oesophagus
D) Air-fluid levels in upper oesophagus
E) Aspirated contrast in trachea-oesophageal tract
18. Which the following structures contribute to lower esophageal sphincter function (2011.SUR.MCQ.01)
A) Angle of His
B) Coeliac plexus
C) Intra abdominal pressure
D) Left crus of diaphragm
E) mucosal arrangement of at the lower end of oesophagus
19. 45yr lady complains of PR bleeding, mucus diarrhoea and sense of incomplete evacuation. Possible causes
(2011.SUR.MCQ.02)
A) Ca rectum
B) Fissure in ano
C) Infectious proctitis
D) Solitary rectal ulcer
E) Villous adenoma
20. Regarding appendicitis (2011.SUR.MCQ.03)
A) Commonest among children less than five years
B) Lymphoid tissue proliferation is a cause
C) In elderly caecal carcinoma should be considered.
D) Non operative management is preferred in appendicular mass
E) Most tender area is McBurney’s point
21. Neoadjuvant chemotherapy in colonic carcinoma (2011.SUR.MCQ.04)
A) Reduce the metastatic disease
B) Reduce the local recurrence
C) Down staging the tumour
D) Preservation of sphincter
E) Increases the risk of anastomotic leakage
22. Regarding gallstones disease (2011.SUR.MCQ.05)
A) Pigmented stones are due to parasites
B) Higher in nulliparous than in multiparous
C) Common among patient with cirrhosis
D) Ileal resection is a predisposing factor
E) Majority is asymptomatic
23. A 45 years old man presented with epigastric pain after meals for 6 months duration. Imaging reveals
pancreatic calcification and there is no duct dilatation. Which of the following management which relive his
pain. (2011.SUR.MCQ.06)
A) Celiac plexus block.
B) Distal pancreatectomy
C) Endoscopic pancreaticograpy and main duct dilatation.
D) Oral tramadol.
E) Pancreatic enzyme supplementation
24. 40yrs old presented with paraumbilical hernia. what are the indications for early surgery? (2011.SUR.MCQ.16)
A) Longer duration of hernia
B) Irreducibility

52
C) Skin ulceration over the hernia
D) Tenderness over the hernia
E) Components being bowel as opposed to the omentum
25. Which of the following surgeries are matched with the optimum anaesthetic procedure? Which of the
following modes of anesthesia are correctly matched with the respective surgical intervention?
(2011.SUR.MCQ.18)
A) Evacuation of a thrombosed external pile - Local anaesthesia
B) Incision and drainage of paronychia - Brachial plexus block
C) Laparoscopic cholecystectomy - High spinal anaesthesia
D) Lateral sphincterotomy - Spinal anaesthesia
E) Relocating a dislocated shoulder - IV sedation
26. 50yrs old man with a history of fever, vomiting and abdominal pain for 2 days duration present with pulse
110 blood pressure70/50 mmHg, tense tender abdomen and gas under the diaphragm in X-ray. Low blood
pressure despite adequate fluid resuscitation. Which of the following are T/F? (2011.SUR.MCQ.19)
A) corticosteroids have no place in initial management
B) low blood pressure possibly due to reduced systemic vascular resistance
C) noradrenalin is the vasopressor of choice
D) septic shock is likely
E) WBC count of 3500 /mm3 exclude diagnosis of sepsis
27. Regarding peptic ulcer disease, (2012.SUR.MCQ.03)
A) Occurs more commonly in antrum than body of the stomach
B) Unlikely to recur
C) Cause of microcytic anemia
D) Invasion of pre-pyloric vein lead to massive hemorrhage
E) Can occur in Meckel’s diverticulum
28. 45-year-old woman presenting with 2 weeks history of increased frequency of bowel motion, abdominal
pain, PR bleeding and she is a diagnosed patient with ulcerative colitis. True regarding this exacerbation,
(2012.SUR.MCQ.04)
A) Can be managed as outpatient
B) Needs urgent laparotomy
C) Needs immediate colonoscopy
D) Responds to steroid therapy
E) Sulfasalazine effective as the initial treatment
29. Regarding incisional hernia (2012.SUR.MCQ.05)
A) Rectus closure with absorbable sutures reduces the risk
B) Higher rate of strangulation than femoral hernia
C) Prophylactic mesh reduces its risk
D) Obesity increases the incidence.
E) .
30. Regarding neoplasms of Liver, (2012.SUR.MCQ.06)
A) Hepatic adenoma is a complication of prolonged OCP use
B) Cirrhosis is a risk factor for hepatocellular carcinoma
C) Metastatic colorectal cancer can be cured by completely by resection
D) Commonest is the haemangioma
E) Transplantation is its curative treatment for hepatocellular carcinoma
31. Regarding chronic pancreatitis (2012.SUR.MCQ.07)
A) Splenic artery thrombosis is a complication
B) Gallstones are the commonest causative factor in Sri Lanka
C) Intraductal calcification can be seen on abdominal X ray
D) Hypoglycemia is a feature
53
E) .
32. In a 50-year-old male presenting with a large parotid tumor, (2012.SUR.MCQ.10)
A) Sialogram is indicated to exclude calculus
B) Core biopsy is mandatory prior to surgery
C) Ipsilateral facial nerve palsy is a feature of compression
D) Pleomorphic adenoma will be the most likely diagnosis
E) Ultrasound scan indicated
33. 16yr old patient with DM & HTN, on atenolol undergone APR for colorectal carcinoma. What is the best post
op pain management? (2008.SUR.SBR.01)
A) IV morphine infusion
B) Diclofenac Sodium suppository
C) Pethidine IM
D) Epidural analgesia with 5% Bupivacaine
E) Oral Tramadol
34. 25 years old patient with chronic pancreatitis and multiple pancreatic calcification on abdominal X-ray with
pancreatic duct dilation presented with acute abdominal pain. What is the most appropriate investigation for
the diagnosis of chronic pancreatitis of this patient? (2008.SUR.SBR.02)
A) S.Lipase
B) S.Amylase
C) ERCP
D) MRCP
E) .
35. 70 years old male presented with dysphagia and left supraclavicular lymphadenopathy. Biopsy of lymph node
shows squamous cell carcinoma. What is the Best management Option? (2008.SUR.SBR.06)
A) Oesophagectomy
B) Radiotherapy
C) Self-retaining stenting
D) Gastrostomy
E) NG tube insertion
36. 60yr old man who underwent hernial repair, with a mesh is readmitted with fever. He is a diabetic patient &
on oral hypoglycemic drugs. On examination there was erythema around the surgical scar. What is the most
appropriate next step in management? (2008.SUR.SBR.07)
A) Change to insulin
B) Send blood culture
C) Start iv antibiotics
D) Remove sutures
E) Remove mesh
37. Regarding a 70 years old male patient who has undergone an oesophagectomy recently what is the most
appropriate route to start feeding? (2008.SUR.SBR.08)
A) Oral
B) NG tube
C) Gastrostomy
D) Jejunostomy
E) Parenteral nutrition
38. 70 yr old man presents with loose stools for 4 months, also has sense of incomplete evacuation. Sometimes
blood and mucus, DRY- blood in stools. What is the most likely diagnosis? (2008.SUR.SBR.09)
A) Ca of rectum
B) Sigmoid Ca
C) Solitary rectal Ulcer
D) Ca Caecum

54
E) Ulcerative colitis
39. 15 yr old boy presents with recurrent bouts of heavy fresh PR bleeding for 6 months. Most probable diagnosis?
(2008.SUR.SBR.10)
A) Angiodysplasia
B) Ulcerative colitis
C) Diverticular disease of sigmoid colon
D) Ileocolic intussusception
E) Meckel’s diverticulum
40. 45yr old female with severe acute gallstone pancreatitis, presented to surgical casualty ward. What would
be the benefit with an endoscopic retrograde cholangio pancreatography? (2008.SUR.SBR.11)
A) Remove acute fluid collection in lesser sac
B) Remove necrotic pancreatic tissue on CECT
C) Remove the stone located in the common bile duct
D) .
E) .
41. 55yr old patient with Child A cirrhosis is found to have a solitary hepatocellular carcinoma of 4cm in diameter
in L/ lobe of the liver.WOF is the most appropriate? (2008.SUR.SBR.12)
A) Bevacizumab orally
B) Percutaneous ethanol ablation
C) Radiofrequency ablation
D) TACE (trans arterial chemo embolization)
E) Surgical resection of left lobe
42. 30yr old male patient presented with progressive jaundice and darkening of urine. USS revealed dilated
intrahepatic and common hepatic ducts. But CBD measured 7 mm.GB is not dilated and no GB calculi. what
is the most appropriate management? (2008.SUR.SBR.13)
A) Percutaneous transhepatic cholangiogram
B) Contrast enhanced CT
C) Mrcp
D) Ercp
E) Hida
43. 46-year-old male, recurrent acute pancreatitis with duct dilatation. What is the mode of management?
(2008.SUR.SBR.14)
A) Opioids
B) USS guided Coeliac plexus block
C) Stenting
D) .
E) .
44. 40 years old male patient who is diagnosed with chronic pancreatitis admitted with recurrent
abdominal pain. What is the mode of management? (2008.SUR.SBR.22)
A) Splanchnicectomy
B) Coeliac plexus block
C) ERCP and stenting
D) Pancreatico jejunostomy
E) Whipple's Procedure
45. Patient came to surgical ward 8 hours after right hemi colectomy was on epidural analgesia, found to have
pain, low blood pressure and dyspnoea. What is the most appropriate explanation for the above condition?
(2008.SUR.SBR.30)
A) Clot dislodgment, epidural analgesia, slip ligature
B) Clot dislodgment, slip ligature, DVT
C) Clot dislodgment, Sepsis, slip ligature
55
D) Clot dislodgment, epidural analgesia, slip ligature
E) .
46. 55-year-old man presented with a history of painless PR bleeding. He is having sensation of incomplete
evacuation of 6 months duration. On examination he is pale, normal DRE What is the best investigation to
diagnose? (2009.SUR.SBR.23)
A) Upper GI endoscopy
B) Colonoscopy
C) CT abdomen & pelvis
D) Stool occult blood test
E) Flexible sigmoidoscopy and biopsy
47. A young patient was brought to the ward after a major abdominal surgery. Which of the following is the best
for his fluid management? (2009.SUR.SBR.24)
A) 5% dextrose
B) 0.9% NaCL
C) 0.45% NaCl
D) Hartmann solution
E) Darrow's solution
48. A 50-year-old male presented with 2 weeks history of dysphagia for solids. He was found to have a
malignancy at the lower 1/3 of the oesophagus. CT chest & abdomen revealed multiple metastasis in the
lungs. What is the best option of management? (2009.SUR.SBR.27)
A) Radiotherapy
B) Chemotherapy
C) LASER therapy
D) Stenting
E) Oesophagectomy
49. 30-year-old male presented with fresh, painless PR bleeding of 2 weeks duration. He had constipation of 1
weeks duration. DRE was unremarkable What is the most likely diagnosis? (2009.SUR.SBR.28)
A) Anal fissure
B) Rectal CA
C) Diverticulosis of the sigmoid colon
D) Haemorrhoids
E) Ulcerative colitis
50. A 50-year-old otherwise healthy male presented with recurrent, vague upper abdominal pain which was
worsening after meals. He had lost 5 kg during last 2 months, USS showed multiple stones in gallbladder with
mild thickening of the gallbladder wall. Extrahepatic ducts looked normal. No percholedocal fluid. What is
the next appropriate step of management? (2009.SUR.SBR.29)
A) ERCP
B) ESWL
C) Laparoscopic cholecystectomy
D) MRCP
E) UGIE
51. An 18-year-old presented with vomiting after eating. Investigations confirmed achalasia of the Cardia. What is
the most appropriate investigation that confirms the diagnosis? (2009.SUR.SBR.32)
A) Upper GI endoscopy
B) Endoscopic biopsy of lower oesophagus
C) 24-hour pH measurement of oesophagus
D) Esophageal manometry
E) Barium swallow
52. A 45-year-old healthy male presented with recurrent vomiting He was found to have distal gastric carcinoma,
with direct infiltration to pancreatic bed and metastasis to coeliac lymph nodes and left lobe of the liver.

56
What is the most appropriate management? (2009.SUR.SBR.33)
A) Distal partial gastrectomy and left lobe resection and lymph node removal.
B) NG decompression followed by feeding jejunostomy
C) NG tube insertion followed by neo adjuvant chemo radiotherapy
D) Palliative distal gastrectomy and gastrojejunostomy
E) Palliative gastrojejunostomy
53. A 30-year-old man presented with pain in the perineal area and fever, Pain increases with Coughing and
straining. He does not allow DRE due to severe pain. Most likely diagnosis is, (2009.SUR.SBR.34)
A) Acute anal fissure
B) Fistula in ano
C) Grade III hemorrhoids
D) Intersphincteric abscess
E) Perianal haematoma
54. A 34-year-old female presented with acute cholecystitis. She was treated with Co amoxiclav 1.2g IV. After 48
hours of treatment, she develops swinging fever with severe right hypochondrial tenderness. Most
appropriate next investigation is, (2009.SUR.SBR.35)
A) CT scan
B) ERCP
C) MRCP
D) FBC
E) USS of abdomen
55. A 40yr old female presented with an irreducible paraumbilical hernia for 2 years. She gets Intermittent pain.
No other complications. What is most likely to contain in the hernial sac? (2009.SUR.SBR.37)
A) Peritoneal fluid
B) Omentum
C) Jejunum
D) Pre-peritoneal fat
E) Transverse colon
56. A 56 years old female presented with severe colicky epigastric pain for 3 hours. It did not radiate but increases
with fatty meals. She does not have fever or jaundice. What is the most likely diagnosis? (2009.SUR.SBR.39)
A) Acute pancreatitis
B) Biliary colic
C) Acute cholangitis
D) Acute cholecystitis
E) PUD
57. A 60-year-old man presented with an irreducible inguinoscrotal hernia for 6 months. He has mild discomfort.
What is the most appropriate management? (2009.SUR.SBR.41)
A) Assess for early elective Surgery
B) Bed rest, analgesia, ice pack to scrotum
C) Gentle manual reduction
D) Abdominal x-ray to exclude obstruction
E) USS abdomen to diagnose strangulation
58. A 60 yr old male underwent UGIE for epigastric pain and LOA for 2 months. UGIE showed an ulcer in the
antrum with elevated margins. Biopsy was taken and it came as not malignant. What is the best option of
treatment? (2009.SUR.SBR.42)
A) H.pylori eradication Rx
B) Prepare him for gastrectomy
C) Reassure the patient
D) Repeat UGIE & biopsy
E) Start PPI

57
59. A 4-month-old baby underwent I&D for abscess in the buttock under GA. 7 hours after the Surgery, the child
was wide awake & started to cry. Mother wanted to feed the child. What is the most appropriate management
in this situation? (2009.SUR.SBR.43)
A) Slow breastfeeding
B) Start on IV fluids
C) Listen to bowel sounds to decide on feeding
D) Breast feed after 6 hours
E) Wet the mouth with water
60. 50-year-old man with no co morbidities presented with right sided obstructive lesion confirmed to be recto
sigmoid adenocarcinoma. CT scan showed a focal lesion in the left lobe of the liver. CT chest is normal. Most
appropriate management? (2010.SUR.SBR.22)
A) Defunctioning colostomy and palliative chemotherapy
B) Hartmann's procedure and palliative chemotherapy
C) Hartmann's procedure and liver resection
D) Stenting and neoadjuvant chemotherapy
E) Neoadjuvant chemotherapy and anterior resection
61. A 60 years old female presented with abdominal pain for 4 days. She is afebrile and having left iliac fossa
tenderness. CRP - 150, WBC - 20 000/mm with neutrophilia. What is the most appropriate next investigation?
(2010.SUR.SBR.23)
A) USS abdomen.
B) Barium enema.
C) Colonoscopy
D) Sigmoidoscopy
E) .
62. 47-year-old woman was presented with anal pain and spotting of blood on defecation. On DRE an ulcer was
detected. Other physical examination was normal. Biopsy showed squamous cell CA. What is the most
appropriate curative management? (2010.SUR.SBR.24)
A) Abdominoperineal Resection
B) Anterior Resection
C) Chemo-radiotherapy
D) Primary radiation therapy
E) Trans-anal excision
63. A 42 years old man is coming with alternative bowel habits. Patient has undergone a lower GI endoscopy and
revealed malignant growth of 4 cm x 5 cm size. Before the surgery decided to give neoadjuvant chemotherapy.
What is the main objective of that? (2010.SUR.SBR.25)
A) Increase survival.
B) Reduce the local recurrence.
C) Reduce liver metastasis.
D) Reduce metachronous tumour
E) .
64. 21-year-old woman presented to the surgery clinic with pain and swelling over the right submandibular
region which is worse soon after meals. Most appropriate next step in the management (2010.SUR.SBR.26)
A) Core biopsy
B) FNAC of nodule.
C) Submandibular sialadenectomy.
D) X-ray closed mouth occlusive view
E) CT-neck
65. 34-year-old woman presented with right upper quadrant pain and diagnosed to have gall stones. On US scan
there were multiple gall stones in the gall bladder. She was given a date for interval cholecystectomy and
discharged. 4 days later she presented with fever jaundice and abdominal pain. What is the best next step of
investigation? (2010.SUR.SBR.27)
58
A) ERCP
B) CECT abdomen
C) USS abdomen
D) Urgent laparoscopic cholecystectomy and on table cholangiogram
E) Urgent open cholecystectomy and on table cholangiogram
66. 65-year-old male patient was found to be having a solitary mass lesion in the right lobe of the liver. What is
the most appropriate investigation to diagnose? (2010.SUR.SBR.28)
A) FNAC
B) CECT Liver
C) USS abdomen
D) Liver biopsy
E) Laparoscopy
67. 55-year-old woman presented to the surgical emergency at night with painful tender swelling of left groin,
associated with vomiting of 6 hours. Clinical diagnosis of obstructed femoral hernia is made. What is the most
appropriate management option? (2010.SUR.SBR.40)
A) Arrange an USS to confirm the diagnosis
B) Arrange for emergency explorative laparotomy and repair
C) Foot end elevation, ice pack over the swelling and wait for spontaneous reduction
D) Give IM Pethidine + antiemetics & monitor for 2 hours
E) .
68. A newborn on day 5 of birth had bile stained vomiting. His vitals, hydration status and abdomen was normal.
Which of the following is the most likely cause? (2010.SUR.SBR.41)
A) Pyloric stenosis
B) Duodenal atresia
C) Intestinal malrotation
D) Hirschsprung disease
E) Gastro oesophageal reflux disease
69. 54-year-old chronic smoker & betel chewing person presented with dysphagia and LOA. Most probable
diagnosis? (2010.SUR.SBR.49)
A) CA in cervical oesophagus
B) CA in thoracic oesophagus
C) CA in antrum
D) CA in pylorus
E) CA in body of the stomach
70. A 70-year-old man complains of progressive dysphagia for 8 weeks. Upper GI Endoscopy revealed malignant
looking lesion at 30cm. What is the most appropriate Investigation to assess local invasion
(2011.SUR.SBR.21)?
A) Barium swallow
B) CXR
C) Endoscopic US
D) MRI
E) PET scan
71. A 60 yr old man with esophageal varices presents with haematemesis. He is unconscious. His blood pressure
is 80/50mmHg & pulse rate is 124 bpm. What is the most appropriate next step in the management?
(2011.SUR.SBR.22)
A) IV Octreotide
B) Insertion of wide calibre NG
C) Insertion of Sengstaken Blakemore tube
D) Intubation with cuffed ETT
E) UGIE & banding of varices

59
72. A 70-year-old man who is on treatment for Atrial Fibrillation presented with acute onset of severe generalized
abdominal pain. Few abdominal signs on examination. He was found to have increased inflammatory markers
and increased lactate levels. What is the most likely diagnosis? (2011.SUR.SBR.23)
A) Acute appendicitis
B) Acute ischemic colitis
C) Acute pancreatitis
D) Diverticular disease
E) Perforated duodenal ulcer
73. 68 years old male presented with absolute constipation and abdominal distension for 3 days duration. He
doesn't have any previous episodes. He has tachycardia with mild abdominal tenderness without guarding
and rigidity. Supine abdominal X-ray showed bowel distension with coffee bean appearance. What is the
most appropriate management (2011.SUR.SBR.24)?
A) Emergency laparotomy
B) Monitoring for 48 hrs with NG suction
C) Phosphate enema
D) Sigmoid colectomy
E) Sigmoid decompression with flatus tube
74. Treatment options for anal fissures are (2011.SUR.SBR.25)
A) Botox
B) Colonoscopy
C) Lateral anal sphincterotomy
D) Manual anal dilatation.
E) Topical GTN
75. 55-year-old women are being investigated for upper cervical lymphadenopathy. FNAC of the lymph node
shows deposits of squamous cell carcinoma. She also has thyroid goitre with a dominant nodule on the right
lobe. What is the next step in management? (2011.SUR.SBR.32)
A) Excision of the lymph node
B) Fibroptic-naso-laryngoscope (FONL)
C) FNAC of solitary nodule
D) Tonsillar biopsy
E) UGIE
76. 48-year-old man coming with an irreducible painful hernia associated with abdominal pain. On examination
abdomen is tender. What is the immediate management? (2011.SUR.SBR.36)
A) Explore hernial sac
B) Emergency laparotomy
C) Ice packs and elevate the limb
D) Inguinal herniotomy
E) .
77. A 40-year-old known diabetic patient underwent a laparoscopic Cholecystectomy. She was admitted with fever.
On examination PR 100, RR 16, blood pressure 120/70, tenderness in right hypochondrium and there was no
jaundice. What is the Best management option? (2011.SUR.SBR.42)
A) MRCP
B) Urgent ERCP
C) Diagnostic laparotomy
D) USS abdomen
E) CECT abdomen
78. A 70-year-old patient admitted to the ICU following laparotomy for intestinal obstruction. On admission his
arterial blood gas was; FiO2 - 40%, pH- 7.25,7.35,7.45 PO2 - 95,95,100, PCO2-17 (35-45), HCO3--17 (22-26),
Base excess -15 (+1.-2) What is the acid base abnormality found? (2011.SUR.SBR.44)
A) Mixed acidosis

60
B) Primary metabolic acidosis
C) Primary metabolic acidosis with partial respiratory compensation
D) Primary respiratory acidosis with partial metabolic compensation
E) Uncompensated respiratory alkalosis
79. Lingual nerve is damaged during submandibular sialadenectomy. What is the most likely effect on tongue
(2011.SUR.SBR.45)?
A) Loss of general +taste sensation in ipsilateral anterior 2/3 of tongue
B) Loss of general sensation in ipsilateral tongue + Ipsilateral taste in anterior 2/3 of tongue
C) Loss of ipsilateral general sensation in anterior 2/3 of tongue
D) Loss of general sensation ipsilateral tongue
E) Loss of taste sensation in ipsilateral anterior 2/3 of tongue
80. 45-year-old male presents with chronic diarrhoea with numbness of extremities and fatigue. Investigations
review macrocytic anaemia with CRP 4. What is the most likely site of pathology? (2011.SUR.SBR.46)
A) Stomach
B) Ascending colon
C) Jejunum
D) ileum
E) Transverse colon
81. 9-month-old baby boy presented with right sided inguinal hernia. What is the appropriate management?
(2011.SUR.SBR.47)
A) Herniotomy
B) Herniotomy with bassini repair
C) Laparoscopic mesh repair
D) Lichtenstein mesh repair
E) Observe for 6 months
82. 50 kg male awaiting colonic surgery. Which of the following is most likely to reduce risk of surgical site
infections? (2012.SUR.SBR.20)
A) Mechanical bowel preparation
B) Preoperative CHO loading
C) Preoperative nutrition
D) Shaving the site
E) Prophylactic antibiotics
83. 60-year-old cirrhotic patient comes with haematemesis, he is conscious, rational, PR 90, BP100/80.
What is the best management option? (2012.SUR.SBR.24)
A) IV access, massive transfusion
B) IV access, octreotide infusion, urgent endoscopy
C) IV access and give normal saline bolus
D) IV access, Grouping and cross matching, early endoscopy
E) Intubation with Sengstaken tube to arrest bleeding
84. 60-year-old lady with past history of dyspepsia for 2 years and treated for pulmonary TB 10 years back
presented with LOA and anemia for 2 weeks. UGIE shows ulcer with everted edges and necrotic material
with contact bleeding. What is the most likely diagnosis? (2012.SUR.SBR.25)
A) Benign peptic ulcer
B) Gastric cancer
C) GIST
D) Primary gastric lymphoma
E) TB ulcer
85. 40 years old female patient is presenting with abdominal pain and bilious vomiting. She has undergone a
surgery for small bowel perforation 5 years back. She has gall stone disease. What could be the reason for
this time presentation? (2012.SUR.SBR.26)

61
A) Adhesions
B) Carcinoma of the colon
C) Gall stone ileus
D) Inguinal hernia
E) Paralytic ileus
86. 64yr lady Who is having history of altered bowel habits presents with sudden onset vomiting, lower
abdominal pain and abdominal distention. On examination there is a left iliac fossa lump. She had similar
episode 1yr back. Probable diagnosis? (2012.SUR.SBR.27)
A) Left tubo-ovarian abscess
B) Diverticular mass
C) Fulminant ulcerative colitis
D) Locally advanced left colonic cancer
E) Acute pyelonephritis
87. 25-year-old male presented with painless fresh bleeding per rectum for 2 weeks duration. What is the most
likely diagnosis? (2012.SUR.SBR.28)
A) Anal carcinoma
B) Anal fissure
C) Anal polyp
D) Haemorrhoids
E) Rectal carcinoma
88. 11 months old child with tender and tense right inguinal lump. On examination the child is irritable,
and abdomen is distended. What is the best management? (2012.SUR.SBR.29)
A) Hydration and pain relief
B) Manual reduction and inward observation
C) Manual reduction and routine surgery
D) Resuscitation and emergency surgery
E) USS and IV antibiotics
89. 50-year-old female patient presented with deep jaundice with colicky abdominal pain and fever for 3 days
duration, she has undergone a laparoscopic cholecystectomy for symptomatic gall stones two weeks ago.
What is the most likely diagnosis? (2012.SUR.SBR.31)
A) Benign biliary stricture
B) Bile duct injury
C) Gall stone pancreatitis
D) Retained stone in the bile duct
E) Local bile collection
90. Male presentation with colicky abdominal pain and absolute constipation for 4 days. which x-ray sign
indicate immediate laparotomy? (2012.SUR.SBR.43)
A) Absent rectal gas
B) Dilated sigmoid colon
C) Small bowel loop dilation
D) Dilation of caecum to 12 cm
E) Distension of transverse colon to 6cm
91. Previously healthy 18 months old child stays in a separate room from family members and experiences
sudden onset cough which settles after few minutes. Then child appears normal but has drooling of saliva
and refusal to feed. What’s the most possible diagnosis? (2012.SUR.SBR.47)
A) Epiglottitis
B) Foreign body in piriform fossa
C) Foreign body in airway
D) Foreign body in esophagus
E) GORD

62
Paediatrics – GIT
1. 6 months old baby chronic diarrhoea, perianal and peri oral erythematous ulcers & peeling off skin, what is
the management? (2008.PAED.MCQ.11)
A) Antidiarrheal
B) Steroid cream
C) Zinc supplementation
D) NSAIDs
E) Stop breast feeding
2. 2yr old child admitted to the hospital with a history of kerosene oil ingestion % hour ago. On admission she is
active and alert. On examination lungs are clear. Next step in management? (2008.PAED.MCQ.14)
A) Gastric lavage
B) Discharge
C) Induce vomiting
D) Keep under observation
E) Perform urgent chest x-ray.
3. 6 weeks old baby bought to the emergency unit by mother due to forceful vomiting after feeding. He feeds
hungrily and is alert. There is a fullness in right upper quadrant. What is the diagnosis? (2008.PAED.MCQ.17)
A) Pyloric stenosis
B) Raised ICP
C) NEC
D) Duodenal atresia
E) GORD
4. 2yr old child presented with watery diarrhoea and vomiting. Child is drowsy and reluctant to drink water. What
is the most appropriate first step in management? (2008.PAED.MCQ.18)
A) Manage with ORS
B) 0.9% NaCl 100 ml/kg within 4hours
C) 0.9% NaCl 100 ml/kg within 24 hours
D) 0.9% NaCl 20 ml/kg within 24 hours
E) Fluid resuscitation via NG tube
5. Intussusception (2009.PAED.MCQ.14)
A) Commonly seen in neonates
B) Associated with an abdominal mass
C) Diagnosed by technetium studies
D) Can effectively reduce by N/S reduction
E) Can cause bowel necrosis
6. Causes for oral pigmentation, (2009.PAED.MCQ.08)
A) Addison's disease
B) Pellagra
C) Peutz-Jeghers syndrome
D) Crohn's disease
E) Fanconi anaemia
7. Anorexia nervosa (2009.PAED.MCQ.17)
A) Hypokalaemia is a feature
B) Girls are affected predominantly
C) Affected individuals like to cock
D) Basal metabolic rate is low
E) Vomiting is a feature

63
8. 2-year-old boy presented with high fever and bloody diarrhoea with abdominal pain on examination there
was generalized abdominal tenderness. Which of the following are possible causative organisms?
(2010.PAED.MCQ.08)
A) Campylobacter
B) Clostridium botulinum
C) Enterotoxigenic E. coli
D) Giardia lamblia
E) Shigella dysenteriae
9. Regarding duodenal atresia (2010.PAED.MCQ.14)
A) Bile stained vomitus is a feature
B) Causes hypokalaemia as a complication
C) Abdominal x-ray shows multiple air-fluid level
D) Presents within 48 hours of life
E) Usually resolves spontaneously
10. Regarding Hirschsprung disease (2011.PAED.MCQ.06)
A) Known cause for constipation
B) Has autosomal dominant inheritance
C) Associated with Down's syndrome
D) Complicated with E coli sepsis
E) Loaded rectum in digital rectal examination
11. Regarding Intussusception (2011.PAED.MCQ.13)
A) It's common in the first 3 months of age
B) lleo-lleal is the commonest subtype
C) Presents with blood-stained vomiting
D) Ultrasound is diagnostic
E) Saline reduction is used as a therapy
12. GORD in preterm babies, (2012.PAED.MCQ.06)
A) Causes apnoeic attacks
B) May present with arching of the body
C) Diagnosed clinically
D) Resolves spontaneously
E) Treated with fundoplication in resistant cases
13. Congenital diaphragmatic hernia (2012.PAED.MCQ.13)
A) Commonly occurs in the left side
B) Presents with respiratory distress at birth
C) Resuscitated with mask and bag ventilation
D) Needs NG decompression
E) Needs immediate surgical correction
14. A 10-month-old well fed baby presented with yellowish discoloration of palms and He was well looking, sclera
clear. Most appropriate management? (2008.PAED.SBR.26)
A) Expose baby to sunlight
B) Take a careful dietary history and make changes
C) Phototherapy
D) Stop breast feeding & increase solids
E) Serum bilirubin levels
15. A 2 weeks old baby was referred by a midwife for jaundice. Mother complains of inadequate breast milk. Baby
was born at term by NVD. Birth weight - 2.8kg. Immediate postnatal period was uncomplicated on examination
baby is active, well perfused, mildly jaundiced, body weight 2.7kg, what is the next step in management?
(2009.PAED.SBR.32)

64
A) Admit for phototherapy
B) Perform urine culture
C) Reassure and send home
D) Assess and support breastfeeding.
E) Commence formula feeding
16. 3-month-old baby on exclusive breastfeeding presented with vomiting and diarrhoea for 3 days. He is not
dehydrated. Well looking. Perianal excoriations present. Stools were positive for reducing substances. What
advice would you give you to the mother? (2010.PAED.SBR.32)
A) Continue breastfeeding
B) Prescribe lactose free formula
C) Check stools for reducing substances
D) Evaluate for cow milk protein allergy
E) Reassure parents 33
17. A 5-day old baby presented with bilious vomiting on day 5. Baby had passed meconium on birth. There was
no abdominal distension. CRFT <2 sec. Baby was pink and well. What is the diagnosis? (2010.PAED.SBR.38)
A) Duodenal atresia
B) Necrotizing enterocolitis
C) Jejunal atresia
D) Septicaemia
E) Volvulus
18. 3-month-old baby is brought with gradual worsening of jaundice, dark urine and pale stools. Child had a birth
weight 2.6 kg with current weight 3.2kg. He is not on medications Examination reveals 1cm hepatomegaly
with mild palpable spleen. USS shows distended gallbladder contractile with meals. S.total bilirubin 155 umol/
direct bilirubin 70 umol/I AST 65 ALT 85 what is the most likely cause for the jaundice? (2011.PAED.SBR.32)
A) Biliary atresia
B) Choledochal cyst
C) Congenital hypothyroidism
D) Dubin Johnson syndrome
E) Neonatal hepatitis syndrome
19. 11-year-old girl presented with episodes of severe abdominal pain lasting 3 to 4h. She was pale and ill during
the episodes. She had also passed mucus loose stools. On examination the abdomen was tender. ALT 58, AST
49, ESR 85mm/1st h. What is the most likely investigation to arrive at a diagnosis? (2011.PAED.SBR.33)
A) CECT abdomen
B) Rectal biopsy
C) Ultrasound abdomen
D) .
E) .
20. A 9-yr. old child developed nausea, abdominal and cramping watery diarrhoea. He is mildly dehydrated, and
abdomen is tender. He had eaten Chinese fried rice the previous night. What is most likely organism to cause
this illness? (2011.PAED.SBR.38)
A) Bacillus cereus
B) Vibrio parahaemolyticus
C) Salmonella typhimurium
D) Shigella flexneri
E) Campylobacter jejuni
21. 3-year-old presents with 3 days of diarrhoea with white colour urine. No other complications
(2011.PAED.SBR.45)
A) Packed cells 10 ml/kg
B) 0.9% NACL 10ml/kg infusion over ihr
C) Administer dextran 10 ml/kg bolus
D) 0.9% normal saline 20m/kg
65
E) Hartman's 10 ml/kg
22. 5 months old adopted child presented with watery diarrhoea for 3 weeks’ duration following one episode of
acute gastroenteritis. Examination was normal except anal excoriation. What is the next management?
(2012.PAED.SBR.31)
A) Change another brand of formula milk
B) Commence soya-based formula
C) Continue oral antibiotics
D) Commence complementary feeding
E) Commence zinc supplements
23. 13 years old girl presented with recurrent abdominal pain for last 4 years. What is the most appropriate feature
suggestive of a functional abdominal pain? (2012.PAED.SBR.32)
A) Presence of blood and mucus diarrhoea
B) Intensity increases with acute stressful events
C) Pain localized to periumbilical region
D) Pain worsens at night
E) Pain relived with defecation
24. 8 days old baby presented with dark urine and pale stools and he is having low grade fever, loss of appetite
for 5 days’ duration with mild icterus and pallor. On examination liver is palpated 2cm and spleen is 4cm
enlarged. Pulse rate 110 beats/min and other parameters are normal. What is the most probable diagnosis?
(2012.PAED.SBR.40)
A) Acute viral hepatitis
B) Autoimmune hepatitis
C) G6PD deficiency
D) Congenital infection
E) Biliary atresia
25. 8-year-old child diagnosed with CAH presents with repetitive vomiting. Severely dehydrated and tachypnoeic.
BP 70/50. Most important initial step in management, (2012.PAED.SBR.43)
A) IV dextrose
B) IV 0.9% saline 20ml/kg
C) Double dose IV fludrocortisone
D) 10% dextran bolus
E) IV hydrocortisone
26. 2 years old boy presented with one episode of vomiting and fresh blood with stool. Had history of mucoid
stool. On examination child ill, tender abdomen present. What is the diagnosis? (2012.PAED.SBR.47)
A) Intussusception
B) Meckel’s diverticulum
C) Perforated appendix
D) Intestinal malrotation
E) Volvulus

66
7. BLS Module - Final MBBS Common MCQs
Medicine – BLS
1. 65-year-old male presented with pallor and lethargy. O/E-he is pale, generalized lymphadenopathy, 3cm
hepatomegaly and 4cm splenomegaly. Ix-Hb-9g/dl, WBC-40,000, Plt - 100,000. Lymphocytes 90%. What is the
most likely diagnosis? (2008.MED.SBR.19)
A) Non-Hodgkin Lymphoma
B) CLL
C) CML
D) Myelofibrosis
E) .
2. 23-year-old boy presented with anaemia O/E - he is pale, no organomegaly Investigations revealed - Hb -
9.5g/dl, MCV-60, MCHC-16, WBC -6,000, PLT - 200,000. S. ferritin – 275(40-200) What is the most likely
diagnosis, (2008.MED.SBR.20)
A) Thalassemia Trait
B) Beta thalassemia Major
C) Iron deficiency anaemia
D) Pernicious anaemia
E) Myelodysplastic syndrome
3. Regarding warm autoimmune haemolytic anaemia (2009.MED.MCQ.12)
A) Haemolysis occurs in peripheries
B) Associated with IgM antibodies
C) Caused by Mycoplasma pneumoniae
D) is usually Coombs positive
E) Can be caused by methyldopa treatment
4. A 25-year-old male presented with hypochromic microcytic anaemia and blood picture showed target cells
Serum iron and ferritin levels are in upper normal level. HbA1c level elevated. What is the diagnosis?
(2009.MED.SBR.39)
A) Hereditary spherocytosis
B) Sideroblastic anaemia
C) Beta thalassaemia trait
D) Normocytic normochromic anaemia
E) Alpha thalassaemia minor
5. A 20-year-old female is tired and having SOB. She had fever and cough 5 days ago and was treated with
azithromycin 500mg daily. Hb-8.3g/dl WBC-5000 PLT-180000 MCV-99 MCH-28 Monospot test-negative
Reticulocyte count-7%Blood film- Red cell agglutination What is the diagnosis? (2009.MED.SBR.40)
A) G6PD deficiency
B) Mycoplasma pneumonia
C) DIC
D) IMN
E) B12 deficiency
6. What are the causes for megaloblastic anaemia? (2010.MED.MCQ.12)
A) Rheumatoid arthritis
B) Chronic alcoholism

67
C) Pernicious anaemia
D) Hypothyroidism
E) Liver cell disease
7. 20 years old female presented with pallor and jaundice. Her Hb is 7 g/dl, WBC is 6000/mm, PLT- 250x 10/m3
(2010.MED.SBR.41)
A) Blood picture, Reticulocyte count, Serum bilirubin, Urine hemosiderin.
B) Direct coombs test, Reticulocyte count.
C) Bilirubin, Urine hemosiderin, Haptoglobin, Osmotic fragility test
D) Reticulocyte count, Bilirubin, Blood picture, Direct Coombs test,
E) Reticulocyte count, Bilirubin, Haptoglobin, Hb electrophoresis
8. 30-year-old asymptomatic female presenting with low platelet count (110), and 3 cm splenomegaly. She has
no other symptoms. No other abnormalities in examination and full blood count. What is the most likely
cause? (2010.MED.SBR.42)
A) Thrombotic thrombocytopenic purpura
B) Immune thrombocytopenic purpura
C) Multiple myeloma
D) Cirrhosis
E) .
9. Features of chronic extra vascular haemolysis (2011.MED.MCQ.11)
A) Increased urine hemosiderin
B) Increased conjugated bilirubin
C) Reticulocytosis
D) Hypocellular bone marrow
E) Pigment stones
10. A 65-year-old man was diagnosed to have proximal DVT following a total hip replacement surgery was started
with anticoagulant and PT INR is maintained at 2.5. What is the most appropriate next step in the
management? (2011.MED.SBR.34)
A) Insert an IVC filter
B) Continue anticoagulant indefinitely
C) Continue anticoagulant for 3 months and follow up with thrombophilia screening
D) Continue anticoagulant for 2 weeks and convert to aspirin
E) Continue anticoagulant for 3 months.
11. Advantages of LMWH over unfractionated heparin (2012.MED.MCQ.05)
A) No need of monitoring
B) Effect is easily reversible
C) Due to its long half-life, it does not have to be given as an infusion
D) Less heparin induced thrombocytopenia
E) Can be managed as outpatient

Surgery – BLS
1. 45-year-old male presents with chronic diarrhoea with numbness of extremities and fatigue. Investigations
review macrocytic anaemia with CRP 4. What is the most likely site of pathology? (2011.SUR.SBR.46)
A) Stomach
B) Ascending colon
C) Jejunum
68
D) Ileum
E) Transverse colon
2. 58-year-old female underwent renal transplant 7 weeks back presented with polyuria, polydipsia abdominal
pain. Her serum corrected calcium level was 2.8 mmol/ (2.2-2.6 mmol/L) phosphate 0,69 mmol/ (1.12 to 1.45
mmol/L). Most probable diagnosis (2011.SUR.SBR.33)
A) Multiple myeloma
B) Paget's disease
C) Primary hyperparathyroidism
D) Secondary hyperparathyroidism
E) Tertiary hyperparathyroidism
3. What is the most likely malignancy that can occur in a patient who underwent Kidney Transplant due to End
Stage Renal Failure? (2011.SUR.SBR.49)
A) Hepatocellular carcinoma
B) Hodgkin Lymphoma
C) Kaposi's sarcoma
D) Nasopharyngeal carcinoma
E) Squamous cell carcinoma

Paediatrics – BLS
1. 8-year-old a febrile boy presented with 2 weeks history of ecchymotic patches. No mucosal bleeding or
lymphadenopathy. Hb-12, WBC 10500, N 58% L 40%, PLT 24000. Management? (2008.PAED.MCQ.07)
A) Manage as an outpatient.
B) Start with immunoglobulin followed by steroids
C) Give only immunoglobulin
D) Admit, observe as an Inward patient for new bleeding and repeat plt count
E) Give prednisolone following a bone marrow study
2. 1yr old boy presented with fever and erythematous palms and soles. He has also cervical lymph nodes. Fever
didn't respond to simple antipyretics. He has mild redness on eyes. No organomegaly. WBC-28300, N=75%,
plt-increased, ESR=78mm/1" hr, Blood cultures no growth. What is the most probable diagnosis?
(2008.PAED.SBR.10)
A) JIA
B) Dengue
C) HSP
D) Kawasaki
E) SLE
3. 5-year-old boy on maintenance therapy for ALL presenting with a fever. His absolute neutrophil count is 500.
Blood culture has been taken. What is the next step? (2008.PAED.SBR.12)
A) Buffy coat transfusion
B) Commence broad spectrum IV antibiotics
C) Manage fever with antipyretics
D) Transfer the child to center of oncology unit.
E) Manage according to blood culture
4. 12-year-old girl with fever, malaise and lethargy for 10 days. Splenomegaly and lymphadenopathy. Blood
picture shows atypical lymphocytes. Diagnosis? (2008.PAED.SBR.13)
A) ALL
B) EBV
69
C) CMV
D) Kawasaki disease
E) Streptococcal tonsillitis
5. Girl with hair loss, oral ulcers, small joint arthritis. What is the best test to diagnose her condition?
(2008.PAED.SBR.23)
A) ANA
B) Antibodies to double stranded DNA
C) ASOT
D) ESR
E) Rheumatoid factor
6. A 10-month-old well fed baby presented with yellowish discoloration of palms and He was well looking, sclera
clear. Most appropriate management? (2008.PAED.SBR.26)
A) Expose baby to sunlight
B) Take a careful dietary history and make changes
C) Phototherapy
D) Stop breast feeding & increase solids
E) Serum bilirubin levels
7. Regarding hereditary spherocytosis (2009.PAED.MCQ.09)
A) Presents as neonatal jaundice
B) Causes aplastic crisis
C) Splenectomy is a treatment option
D) X linked recessive
E) Pigment stone can be formed
8. Regarding ALL (2009.PAED.MCQ.13)
A) Common in girl
B) Common in Down Syndrome
C) Can be excluded by a normal blood picture
D) Chromosome t (9:22) indicates poor prognosis\Hypodiploidy is a feature of good prognosis
E) .
9. National immunization programme (2009.PAED.MCQ.19)
A) Oral Pulio is important to produce herd immunity
B) Coverage is 80%
C) Adverse effects are reported to the Family Health Bureau
D) MMR is given at 1 year
E) JE is contraindicated in cerebral palsy
10. A 12-month-old boy p/c with anemia. Parents are consanguineous. When he is 6 months old, he was pale and
was given iron treatment. Now, on examination, he is pale and icteric. There is hepatosplenomegaly on
abdominal examination. Hb-7.8g/dl, MCV 60(76-96) MCH-24 RDW-12% What is the investigation performed
to confirm the diagnosis? (2009.PAED.SBR.34)
A) Bone marrow aspiration
B) Hb electrophoresis
C) HPLC
D) Stool occult blood
E) Serum ferritin

70
11. A 4 yr old boy presented with fever for 4 weeks O/E bilateral lymph node enlargement was noted. Non-
hodgkin’s lymphoma is suspected. Most appropriate investigation? (2009.PAED.SBR.42)
A) Bone marrow biopsy
B) Lymph node biopsy
C) USS of lymph node
D) Serum LDH level
E) Blood picture
12. A 10 yr old girl was diagnosed to have polyarticular JIA. She is having severe pain with morning stiffness even
though she is on high dose ibuprofen What is the most appropriate management? (2009.PAED.SBR.45)
A) Change NSAID to Diclofenac sodium
B) Start antibiotics
C) Intra articular steroids
D) Oral prednisolone
E) Methotrexate
13. Increased risk of leukaemia in children (2010.PAED.MCQ.13)
A) Chemotherapy
B) Down syndrome
C) Immunodeficiency
D) Fanconi anaemia
E) .
14. True regarding Kawasaki disease (2010.PAED.MCQ.15)
A) Kawasaki is diagnosed by a diagnosed criterion
B) Causes purulent conjunctivitis
C) prednisolone is a treatment option
D) Peeling off the skin is a late feature
E) Viral infection is an aetiological factor
15. Regarding the immunization programme in Sri Lanka, (2010.PAED.MCQ.19)
A) Rubella vaccine not indicated when child already got the disease.
B) OPV is given at 5 years.
C) When a vaccine lapses by 3 months the schedule should start from the beginning.
D) IPV is a live attenuated vaccine
E) JE is given at 12 months.
16. A term baby of O+ was born with a birth weight of 2.5 kg to a mother of A+ and was found to be pale. His
haemoglobin was 10 g/dl, reticulocyte count 5% and PCV 34. Which of the following is the most likely
diagnosis? (2010.PAED.SBR.33)
A) ABO incompatibility
B) Foeto-maternal haemorrhage
C) Iron deficiency anaemia
D) Physiological anaemia
E) G6PD deficiency
17. A 4-week-old baby presented with poor feeding and poor weight gain. Birth weight was 3.1 kg and current
weight is 2.9 kg. Examination is normal. Investigations results are given below. What is the next best
investigation? WBC -8.8 103 N - 48% L-47% Na - 121 mmol/L K-5.3 mmol/L Urea - 4 mmol/L
(2010.PAED.SBR.37)

71
A) ABG
B) Estimation of anion gap
C) Random blood sugar
D) .
E) .
18. 20-month-old boy presented with fever and irritability for 3 days. He is unable to move his R/leg. O/E there is
R/knee joint swelling and tenderness. What is the most appropriate investigation before management?
(2010.PAED.SBR.42)
A) Blood culture and ABST.
B) FBC.
C) ESR.
D) USS knee joint.
E) X-Ray knee joint
19. Haemophilia A (2011.PAED.MCQ.08)
A) Mucosal bleeding
B) Measure creatinine kinase level
C) .
D) .
E) .
20. Causes of monoarthritis in a 4-year old (2011.PAED.MCQ.14)
A) Psoriatic arthritis
B) Acute Lymphoblastic Leukaemia
C) Gout
D) Haemophilia A
E) Von Willebrand disease
21. Regarding Adverse Events Following Immunization (2011.PAED.MCQ.16)
A) Thrombocytopenia following MMR
B) Reported to regional epidemiologist
C) Reporting by the community health team is acceptable
D) Hyporesponsive hypotensive state known to occur following pentavalent vaccine
E) Lymphadenitis following BCG is seen up to 6 months
22. 3-day old baby weighing 2.9kg is noted of having yellowish discolouration in face. Birth weight was 3kg Baby
is well and examination is unremarkable. Mother's blood group is A positive. What is the immediate
management option? (2011.PAED.SBR.20)
A) Give advice on breast feeding
B) Reassure the mother and discharge
C) Send blood for investigation
D) Start intravenous fluid
E) Start phototherapy
23. 40-Year-old child has presented with left knee joint swelling and gum bleeding. He has a History of easy
bruising. FBC is normal. APTT and bleeding time is prolonged What is the most probable diagnosis?
(2011.PAED.SBR.41)
A) Christmas disease
B) Haemophilia A

72
C) Vitamin C deficiency
D) Vitamin K deficiency
E) Von Willebrand Disease
24. 7-year old boy undergoing chemotherapy for Acute lymphoblastic leukaemia presented with Fever for 24
hours and oliguria for 18 hours. His Investigations are as follow. WBC-50x109 Neutrophil - 15 109 Serum K-6.5
(3.5-5.5) Serum Phosphate -6.5 (3.5-5.5) What is the most likely diagnosis (2011.PAED.SBR.42)
A) Drug Induced neutropenia
B) Leukemia Induced nephropathy
C) Renal vein thrombosis
D) Severe sepsis
E) Tumour lysis syndrome
25. 6-year old boy presented in a collapsed state. He had fever and coffee ground vomiting 3 days; He is currently
afebrile. Pulse rate 135. BP 70/50 mmHg. Blood investigations done in the morning were as follows (Not sure
of the values) (2011.PAED.SBR.44)
A) FBC reduced WBC count with lymphocytic predominance
B) Thrombocytopenia
C) .
D) .
E) .
26. Hypochromic microcytic anaemia seen in (2012.PAED.MCQ.08)
A) Vitamin b12 deficiency
B) Lead poisoning
C) Sideroblastic anaemia
D) Alpha thalassaemia trait
E) Haemolytic ureamic syndrome
27. Regarding EPI schedule in Sri Lanka (2012.PAED.MCQ.17)
A) BCG vaccine can protect children from TB meningitis
B) JE live vaccine is given via S/C route
C) Pentavalent vaccine contains whole cell pertussis vaccine
D) MMR is given at 9 months and 3 years
E) Pneumococcal vaccine is introduced recently
28. 10-year-old child presented with high fever & cough for 6 days, 3 years ago diagnosed ALL & completed
treatment. On examination pale, cervical lymph node enlarged & hepatomegaly 4cm. Hb - 7.8 WBC - 48
(lymphocytes 98%) Platelet 70. What's the best management? (2012.PAED.SBR.41)
A) Allopurinol
B) Admit and refer to oncology team
C) Arrange blood transfusion
D) Do an urgent bone marrow
E) IV antibiotics and blood for culture

73
8. Nephro-Urology Module - Final MBBS Common MCQs
Medicine – NU
1. A 35-year-old male presents with haematuria of 1-day duration. He has had a similar episode 1 1/2 years
back. He is on treatment for diabetes mellitus. On examination there is no oedema. Blood pressure is elevated
UFR-RBC 200, pus cells 3-4, protein nil. No RBC casts and dysmorphic red cells. Serum creatinine and blood
urea normal. Which of the following can be a cause? (2009.MED.MCQ.09)
A) Post streptococcal glomerulonephritis.
B) UTI
C) Renal cell carcinoma
D) IgA nephropathy
E) .
2. 40 years old man presented with SOB for 1 week to A & E department. Lethargy and LOA for 2 weeks. O/E-
pale, B/L ankle oedema, BP - 170/100. What are the features suggestive of CKD rather than AKI?
(2010.MED.MCQ.37)
A) Increased JVP
B) Hyperkalaemia
C) Shrunken kidney on USS
D) .
E) .
3. Regarding acute kidney injury, (2011.MED.MCQ.08)
A) NSAIDs is a major cause.
B) Cephalosporin is given for associated Infection
C) Red blood cell casts indicate renal tubular acidosis.
D) IV Furosemide may improve survival.
E) Severe acidosis is an Indication for dialysis.
4. Renal diseases correctly matched with the cause (2012.MED.MCQ.03)
A) Membranous nephropathy – bronchial CA
B) Acute interstitial nephritis – ibuprofen treatment
C) Acute tubular necrosis – low dose aspirin
D) Acute cortical necrosis – Russel’s viper
E) Chronic interstitial nephritis – metformin
5. Regarding acute kidney injury (2012.MED.MCQ.20)
A) Adequate replacement of the volume will reverse prerenal failure
B) Brown muddy granular casts- glomerular nephritis???
C) Renal biopsy is contraindicated in anuria
D) Pericarditis is a complication
E) Diuretics improve survival
6. 32-year-old male with HTN presented to ward. His father died due to SAH. UFR - Alb RBC 40-50. S.Cr- 112.
Na+ - 139, K+ - 4.2, Hb - 16 g/dl, HCT-45, WBC- 7000, pH - 7.4. ESR -5, 24-hour protein excretion - 900 mg.
What is the most likely diagnosis? (2008.MED.SBR.13)
A) Renal artery stenosis
B) Nephrotic Xd
C) Polyarteritis nodosa
D) PCKD
E) CRF
7. 16 yr old boy presented with facial puffiness, ankle oedema. Urine dipstick +++, 24 hr urine protein- 4.5g.
What is the most likely diagnosis? (2008.MED.SBR.14)
A) Ig A nephropathy

74
B) Polycystic kidney disease
C) Minimal change nephrotic Xd
D) Post infectious GN
E) RPGN
8. Which of the following is the best indicator of chronic kidney disease? (2008.MED.SBR.15)
A) Blood urea > 110
B) Bilateral shrunken kidneys
C) Creatinine clearance - low
D) Proteinuria
E) Serum K+ - 6.5 mmol/L
9. What is the best management option for hepatorenal syndrome? (2009.MED.SBR.32)
A) Reduce diuretic therapy
B) IV fluid resuscitation with normal saline
C) IV terlipressin
D) Repeated large volume paracentesis
E) IV ceftriaxone
10. A 35-year-old male presents with bilateral ankle swelling to his routine medical check-up. BP 160/105 mmHg,
PR-78 bpm, UFR- protein ++, RBC- 2 hpf, serum creatinine-increased, blood urea-increased, Na+ normal, K+-
normal, GFR-40, USS-increased cortical echogenicity, normal size kidney. What will be the best drug?
(2009.MED.SBR.34)
A) Methyldopa
B) Nifedipine
C) Captopril
D) Prazosin
E) Sildenafil
11. A 60-year-old patient presented to medical unit due to an unidentified snake bite. Then he was diagnosed to
have acute kidney injury. His urine output was 600ml/24h. What is the best management option?
(2009.MED.SBR.35)
A) Restrict protein intake up to 40g/day.
B) Restrict fluid intake to daily loss
C) Start 0.9% saline daily maintenance solution
D) Give low caloric and low protein diet
E) Give high caloric and high protein diet.
12. An 80-year-old male presented with 4 weeks history of fever. During last 2 years he had similar lower urinary
tract symptoms. Symptoms disappear after treating with norfloxacin, but fever persists. He has hypertension,
diabetes and prostatism and is on metformin and prazosin. He has mild pallor and a splenomegaly. A diagnosis
of infective endocarditis is made. What is the most likely causative organism for infective endocarditis in this
patient? (2009.MED.SBR.41)
A) Enterococci
B) Gram negative organism
C) Salmonella
D) Staphylococci
E) Viridans Streptococci
13. A 47-year female presented to the clinic with persistent hypertension. Her last blood pressure recorded was
160/90mmHg. She is asymptomatic and recently lost her job. Investigations revealed Na+ 146, K+- 4.5, Serum
urea4mmol/L and RBS 7.1mmol/L. What is the most appropriate investigation? (2010.MED.SBR.26)
A) 2D echocardiography
B) 24 hours ambulatory blood pressure monitoring
C) Transabdominal scan
D) .
E) .
75
14. 48-year-old man with PCKD investigated for headache. WOF is most useful in diagnosing cause for headache?
(2010.MED.SBR.35)
A) Radiograph of paranasal sinus.
B) Serum creatinine.
C) CT head.
D) Lumbar puncture.
E) MR angiogram.
15. 38-year-old male diagnosed with DM for 18 years presented with progressive leg swelling and SOB. Which of
the following make diabetic nephropathy the most likely diagnosis? (2010.MED.SBR.39)
A) Proteinuria 3+
B) Leukonychia
C) Creatinine level less than 2 mg/dl
D) Proliferative retinopathy
E) .
16. 18-year-old girl presenting with acute heart failure. Blood pressure 230/120 mmHg. There are fine
crepitations in both lungs on auscultation. Hb 8,5 g/dL Blood puncture shows normochromic normocytic cells.
Normal platelets. Urinalysis shows Albumins. Cr 780 mmol/L. S. Ca 1.8 What is the most likely diagnosis?
(2011.MED.SBR.29)
A) HUS
B) CKD
C) Renal artery embolism
D) PSGN
E) IgA nephropathy
17. A patient with acute kidney injury has following investigation results Serum creatinine 8,2 mg/dl. Sodium
140mmo Potassium 7.9 mmol Serum bicarbonate 16 mmol ECG shows tall T waves and broad QRS complexes.
Which of the followings are most appropriate next step in the management? (2011.MED.SBR.30)
A) IV calcium gluconate
B) IV furosemide
C) IV insulin and dextrose
D) IV bicarbonate
E) Salbutamol nebulization
18. A 40-year-old male presented with periorbital oedema for 2 days.
• O/E : BP – 160/100, RS examination – bilateral basal crepitations
• UFR findings
➢ Protein +
➢ RBC ++
➢ Granular casts
➢ Pus cells – nil
➢ Serum creatinine – 1.8
Most probable diagnosis? (2012.MED.SBR.11)
A) Acute interstitial nephritis
B) Acute glomerulonephritis
C) .
D) Nephrotic syndrome
E) Undiagnosed CKD
19. 45-year-old male presenting with first episode of UTI. Culture positive for E. coli. What is the next best
investigation? (2012.MED.SBR.15)
A) Cystoscopy
B) DMSA
C) Screening for STD
76
D) USS KUB
E) IV urogram
20. 18-year-old schoolboy present with one day history of reduced urine output. He is previously healthy and is
a long-distance runner.
• BU – 10
• Serum creatinine – 900
• Na – 132
• K–6
Possible aetiology for his renal failure? (2012.MED.SBR.19)
A) CKD
B) Dehydration leading to hypotension
C) Interstitial nephritis
D) Rhabdomyolysis
E) .
Surgery - NU
1. 70-year-old male presented with progressive difficulty in passing urine of 6 months duration which of the
following features are suggestive of carcinoma of prostate? (2009.SUR.MCQ)
A) Haematuria
B) Backache
C) Irregular hard prostate
D) Loss of weight of 5kg
E) Palpable bladder after micturition
2. An otherwise healthy adult is admitted with ureteric colic is found to have 9mm stone in the ureter at the level
of 20 sacral vertebrae. Patient is pain free after analgesia. Renal functions are normal. There is no evidence of
sepsis but there is evidence of moderate hydronephrosis on USS WOF is the best management option?
(2008.SUR.SBR)
A) Medical expulsive therapy
B) Extracorporeal shock wave lithotripsy
C) Ureteroscopy and ICSE
D) Nephrostomy
E) Open ureterectomy
3. A 76yr old male with dysuria, urine analysis, pus cells, red cells. PSA 23ng/dl. Examination normal. Most
appropriate management? (2008.SUR.SBR)
A) Prostate biopsy
B) Bilateral orchidectomy
C) Androgen antagonist treatment
D) Commencement of antibiotics after sending urine culture & ABST
E) TURP
4. 10 year old boy presents with right sided testicular pain for 4 hours duration. There is no history of fever. His
right testis is extremely tender. Left testis is normal. What is the most important next step in management?
(2009.SUR.SBR)
A) Urgent USS of testis
B) Start antibiotics
C) Scrotal support
D) Wait for 6 hours
E) Urgent exploration of right testis

77
5. A 35-year-old male presented with left sided colicky abdominal pain and in distress USS revealed a 10mm
stone in mid ureter and a dilated ureter with moderate hydronephrosis. His renal functions were normal. What
is the best management option? (2009.SUR.SBR)
A) Dormia basket extraction
B) ESWL
C) Ureterorenoscopic shockwave lithotripsy
D) Percutaneous nephrostomy
E) Medical expulsion therapy
6. Regarding BPH, (2010.SUR.MCQ)
A) Commonly convert to the malignancy.
B) 5-alpha reductase inhibitors relived symptoms within weeks
C) Can leads to bilateral hydronephrosis.
D) Laser therapy is safer than TURP for larger prostates.
E) Occurs in the central zone of the prostate.
7. 36 years old male presents with recurrent bilateral uric acid urinary calculi. Which of the following can be used
to reduce recurrent attacks? (2010.SUR.MCQ)
A) Allopurinol
B) High dose of Vitamin C
C) Hydrochlorothiazide
D) Potassium citrate
E) Sodium bicarbonate
8. 26-year-old scrotal swelling for 3 weeks. USS confirmed solid mass arising from testis. Next appropriate
management option is, (2010.SUR.SBR)
A) Testicular biopsy
B) AFP and beta hCG level
C) Inguinal orchidectomy
D) MRI scan
E) .
9. Female loin pain and haematuria diagnosed to have 1.6 cm calculus in renal pelvis. Which of the following
investigations give most information? (2010.SUR.SBR)
A) IVU
B) NCCT
C) Contrast spiral CT of KUB
D) X Ray KUB
E) .
10. A 65-year-old man with voiding LUTS presented to the surgical casualty. On examination, there is a hard
prostate on DRE. His PSA was 65ng/l. On bone scan, two lytic lesions were found in the iliac crest and lumbar
vertebra. What would be the best management for this patient? (2010.SUR.SBR)
A) Androgen ablation therapy
B) Oral bicalutamide
C) Radical prostatectomy
D) Primary radiotherapy
E) TURP
11. A 45 years old female presented with invisible (microscopic) haematuria. To evaluate her problem, following
is/are required (2011.SUR.MCQ)
A) CRP
B) Flexible Cystourethroscopy
78
C) Repeat the test if menstruation is present during the time of initial test
D) USS KUB
E) Uroflowmetry
12. Continues dribbling in a 3-year-old girl. (2011.SUR.MCQ)
A) Ectopic ureteric opening
B) Meningomyelocele
C) Posterior urethral valve
D) Recto vaginal fistula
E) Recto vesical fistula vesico-ureteric reflux
13. Best Investigation to identify ureteric stones? (2011.SUR.SBR)
A) Intravenous pyelogram
B) X-ray KUB
C) Steroids
D) X-ray Kidney Ureter Bladder (KUB)
E) NCCT 32
14. 60-year-old man who has background of diabetes nephropathy, serum creatinine normal, is planning undergo
CECT abdomen. Which of the following would the best option to minimize contrast nephropathy?
(2011.SUR.SBR)
A) Normal saline
B) Post procedure haemodialysis
C) give post-procedural Intravenous furosemide
D) Pre procedure IV NaHco3
E) Preoperative administration of NAC
15. What is the most likely malignancy that can occur in a patient who underwent Kidney Transplant due to End
Stage Renal Failure? (2011.SUR.SBR)
A) Hepatocellular carcinoma
B) Hodgkin Lymphoma
C) Kaposi's sarcoma
D) Nasopharyngeal carcinoma
E) Squamous cell carcinoma
16. 65-year-old patient presented with acute urinary retention and after catheterization 2 litres of volume was
drained via catheter. What is the management relevant for next 48 hours? (2012.SUR.MCQ)
A) Serum creatinine
B) Prostatectomy
C) PSA levels
D) Tamsulosin therapy
E) Trial without catheter
17. Regarding causes of renal cell carcinoma, (2012.SUR.MCQ)
A) Chronic haemodialysis
B) Renal papillary necrosis
C) Renal tubular acidosis
D) Vesico-ureteric reflux
E) Von Hipple Lindau syndrome
18. Regarding acute kidney injury, (2012.SUR.MCQ)
A) Always requires haemodialysis
B) Hypokalaemia
79
C) Intra operative hypotension is a risk factor
D) Irreversible
E) .
19. 69yr old female, presented with polyuria, polydipsia and abdominal pain. She has been having recurrent
urinary tract stones. What's the next Ix to diagnose her problem? (2012.SUR.SBR)
A) Metabolic screening
B) Serum calcium
C) Serum parathyroid hormone levels
D) Capillary blood sugar
E) Stone analysis f. X-ray KUB
20. Previously healthy male was diagnosed with 2cm renal stone in renal pelvis. Density is 1350HU. He has episodic
pain and haematuria. Renal functions are normal. Best management option is, (2012.SUR.SBR)
A) ESWL
B) Medical expulsion therapy
C) PCNL
D) Pyelolithotomy
E) .
21. 5-year-old boy presented with empty right hemi scrotum, on examination a normal size testis was palpable in
R/inguinal canal which could be easily manipulated to the scrotum and stays there without traction what is
the most likely diagnosis? (2012.SUR.SBR)
A) Retractile testis
B) Ectopic testis
C) Ascending testis
D) Transverse testicular ectopia
E) Undescended testes

Paediatrics – NU
1. 9 months old baby boy presented with low grade fever and haematuria for 2-day duration. On abdominal
examination there was a lump on R/side which do not crosses midline. What is the diagnosis?
(2008.PAED.MCQ.20)
A) R/ sided neuroblastoma
B) R/ sided nephroblastoma
C) R/ sided pyonephrosis
D) R/ sided urinary calculi
E) R/ sided pyelonephritis
2. Causes for haematuria (2009.PEAD.MCQ.05)
A) UTI
B) Hypocalciuria
C) HSP
D) IgA nephropathy
E) Infective endocarditis
3. Indications for renal biopsy in nephrotic syndrome (2010.PEAD.MCQ.06)
A) Macroscopic haematuria
B) Low C3 &C4
C) Age < 2 year
D) Renal failure with normovolaemia
E) Not responding to steroid for 14 days

80
4. 2-year-old child investigated for renal disease and mother is waiting for renal transplant. Maternal two uncles
also have renal problem. What are the inheritance patterns? (2010.PEAD.MCQ.16)
A) AD
B) AR
C) Mitochondrial
D) XR
E) XD
5. 50-hours old baby has not passed urine since birth. Late antenatal scan showed oligohydramnios. Baby is
active and well. What is the next step of management? (2010.PEAD.SBR.21)
A) IV N/S bolus
B) Palpate for bladder catheterize and check whether urine is coming
C) DMSA
D) .
E) .
6. Baby boy investigated with USS showing R/S dilated renal pelvis, suggesting of PUJ obstruction. What is the
best investigation to confirm the diagnosis? (2010.PEAD.SBR.28)
A) CT scan
B) DMSA
C) DTPA
D) IV pyelogram
E) MCUG
7. 7-year-old boy presented with 2-day history of painless haematuria. Mother complains that the child passes
uniformly red urine with reduced output. He also complains of headache. He had a UTI 6 months back. What
is the most appropriate next investigation? (2010.PEAD.SBR.29)
A) Check urine for deposits
B) Measure her blood pressure
C) Do X-ray of abdomen
D) USS
E) Renal biopsy
8. 2-year-old presents with a mass in right lumbar region. It is not ballotable but crosses to left side. What is the
most likely condition? (2010.PEAD.SBR.39)
A) Hepatoblastoma
B) Neuroblastoma
C) Nephroblastoma
D) Renal cell carcinoma
E) .
9. 6-year-old boy presented with 1st relapse of nephrotic syndrome. Proteinuria persisted at 3+ even after 21
days of 60mg/m2/day prednisolone, but his facial oedema and ankle swelling settled, what is the most
appropriate next step, (2012.PEAD.SBR.30)
A) Add levamisole
B) Give Cyclosporine
C) IV Hydrocortisone
D) Change to methyl prednisone
E) Continue Prednisolone

81
9. Neurology Module - Final MBBS Common MCQs
Medicine – Neurology
1. Regarding GBS (2009.MED.MCQ.01)
A) Type 1 respiratory failure a feature
B) Global areflexia is typical.
C) Distal demyelination is a feature
D) Elevated CSF protein is a feature
E) Treated with IV methylprednisolone.
2. WOF are true about myasthenia gravis (2009.MED.MCQ.02)
A) Muscle wasting is a prominent feature
B) fluctuating muscle weakness is seen
C) Heart is not affected
D) Proximal muscles are mostly affected
E) Associated with thymic hyperplasia.
3. Regarding Myasthenia gravis, (2010.MED.MCQ.01)
A) Pupillary reflex is affected
B) Touch sensation of hand is affected
C) Circulating anti cholinesterase enzyme antibodies are helpful in diagnosing MG
D) Muscle biopsy is needed for diagnosis
E) Aminoglycoside improves fatigability
4. 22-year-old girl complains of severe throbbing headache sometimes lasting for as long as 24 hours,
associated with vomiting, photophobia and phonophobia. WOTF drugs can be used to treat these acute
attacks? (2010.MED.MCQ.02)
A) Pizotifen
B) Amitriptyline
C) Paracetamol
D) Sumatriptan
E) Propranolol
5. Drugs and their side effects (2010.MED.MCQ.04)
A) Sodium valproate - hirsutism
B) Amiodarone – photosensitivity
C) Propafenone - corneal microdeposits
D) Lignocaine - convulsions
E) Digoxin – confusion
6. Structures involved in light reflex, (2010.MED.MCQ.17)
A) Occipital cortex
B) Lateral geniculate body
C) Edinger Westphal nucleus
D) Oculomotor nerve
E) Ciliary ganglion
7. Features of a large lesion in a dominant parietal lobe? (2011.MED.MCQ.01)
A) Receptive aphasia B
B) Left right orientation.
C) Behavioural abnormalities
D) Contralateral hemiparesis
E) Lower left quadranopia
8. 4 year old male was brought to the emergency department by an ambulance 1 hour of aphasia a right
hemiparesis. On examination blood pressure 178/94, pulse rate Bpm, irregular rhythm. C scar 1 hour of
82
arrival showed no early ischemic changes What is the most appropriate manager option
(2011.MED.MCQ.19)
A. Mechanical thrombectomy
B. Artena rtPA
C. IV labetalol
D. Venous rtPA
E. low dose aspirin
9. Regarding a lesion in the posterior column (2012.MED.MCQ.14)
A) .
B) It causes ipsilateral joint position sensory loss
C) Causes contralateral hemiparesis
D) Tendon reflexes are absent
E) Rhomberg test is positive
10. Regarding Guillen Barre Syndrome, (2012.MED.MCQ.16)
A) Presence of a sensory level is a characteristic feature
B) Exaggerated tendon reflexes present
C) Cerebrospinal fluid protein is increased
D) MRI scan is essential in diagnosis
E) Steroids are the mainstay of treatment
11. 65 yr old man presented with fever, headache and drowsiness for 2 days. He has history of recurrent ear
Infection. What is the most appropriate investigation to come into diagnosis? (2008.MED.SBR.01)
A) Ear swab and culture
B) Blood culture
C) EEG
D) CT brain
E) Lumbar puncture
12. 24-year-old male presented with sudden onset dysphasia and R/S hemiplegia and R/Horner's syndrome.
No cerebellar signs. What is the most likely diagnosis? (2008.MED.SBR.02)
A) Lacunar infarct of internal capsule
B) Syringomyelia
C) lateral medullary Syndrome
D) Carotid artery dissection
E) Pancoast tumour
13. 39-year-old obese female admitted with a 3-day history of rapidly developing weakness of all 4 limbs. O/E-
LMN type B/L facial N. palsy with flaccid quadriparesis. Tendon reflexes are absent and plantar response
is flexor. What is most appropriate therapeutic option (2008.MED.SBR.03)
A) IV immunoglobulin
B) Steroids
C) IV antibiotics
D) Azathioprine
E) Physiotherapy
14. A 49-year-old male was found unconscious on road. On arrival his temperature is 37°C, BP - 160/90 mmHg.
PR-90 bpm. Cardiac auscultation findings are normal. His RR-20/min, lungs clear. There is no neck stiffness.
B/L pupils are 1mm in diameter, poorly reactive to light reflex and fundus is difficult to examine B/L'Lower
limb reflexes are normal except B/L plantar responses, which are extensors. What is the most likely
diagnosis? (2009.MED.SBR.22)
A) Snake bite envenomation
B) Organophosphate poisoning
C) Pontine haemorrhage
D) Myasthenic crisis
E) Botulinum toxicity
83
15. A 72-year-old male was brought to the medical clinic by his wife. Wife complains that her husband has
forgetfulness and poor concentration during last few months. She also complains of urinary incontinence
and abnormal walk for the last few months. On examination he is having an unstable gait. His fundal
examination is normal. What is the most possible diagnosis? (2009.MED.SBR.23)
A) Lewy body dementia
B) Motor neuron disease
C) Normal pressure hydrocephalus
D) Parkinsonism
E) Alzheimer disease
16. A 59-year-old male presented with right sided spastic hemiparesis with increased reflexes, and right-side
loss of proprioception and joint position sensation and left side loss of pain sensation. There was no loss
cranial nerve palsy and facial sensation was normal. What is the most likely place of lesion?
(2010.MED.SBR.23)
A) Left side lesion to cervical cord
B) Lesion in right midbrain
C) Right side lesion to cervical cord
D) Lesion in left cerebral cortex
E) Lesion in right cerebral cortex
17. 55-year-old male presented with recurrent seizures. What is the first line investigation? (2010.MED.SBR.24)
A) FBC
B) CECT brain
C) Random blood sugar
D) Serum electrolytes
E) EEG
18. A 55-year-old presents with unconsciousness and smelling of alcohol. He has multiple bruises in various
healing stages, right sided weakness with increased reflexes and tongue bite. (2010.MED.SBR.25)
A) Left sided subdural haemorrhage
B) Left sided cerebral tumour
C) Right sided cerebral tumour
D) Right side haemorrhage in pons
E) Right sided subdural haemorrhage
19. Difficulty in walking in darkness and tendency to fall. On gliclazide and metformin.HBA10 6.5
(2011.MED.SBR.21)
A) Critical illness neuropathy
B) Diabetic neuropathy
C) Diabetic amyotrophy
D) Folate den
E) Vitamin B12 deficiency
20. 60-year-old female presented with progressive dysphagia & dysarthria for 2 months. On examination her
hand muscles are wasted. Both upper limbs & lower limbs are hypertonic & there is no sensory loss, what
is the most likely diagnosis? (2011.MED.SBR.22)
A) Brainstem stroke
B) Cervical disc prolapse
C) Motor neurone disease
D) Multiple sclerosis
E) Myotonic dystrophy
21. 56y old male. Known diabetic. Past history of rheumatic heart disease. Presented with sudden onset LL
weakness. O/E bilateral flaccid paralysis. Power 2/5. Pain sensation impaired up to umbilicus level. Intact
joint position. What is the most probable diagnosis? (2012.MED.SBR.05)

84
A) GBS
B) Transverse myelitis
C) Cauda equina syndrome
D) Anterior spinal artery occlusion
E) Diabetic amyotrophy
22. A 65 year old man presented with acute onset right sided upper limb and lower limb weakness of 2 hours
duration and he had difficulty in understanding what other people are speaking. Blood pressure was
170/110mmHg and pulse rate 88 bpm. What is the next step of management? (2012.MED.SBR.09)
A. .
B. Infusion of IV labetalol
C. IV tissue plasminogen activator
D. Start him on warfarin 5mg/day treatment
E. Treat with S/C enoxaparin
Surgery – Neurology (Neurosurgery)
1. 25-year-old drunk man was accidentally fallen and injured medial aspect of the elbow. What is the least likely
result that can be occurred? (2008.SUR.SBR.03)
A) Complete Ulnar nerve palsy
B) 1" dorsal interosseus paralysis
C) Loss of sensation on anterior aspect of the 5th finger
D) .
E) .
2. Regarding neurotrauma (2009.SUR.MCQ.17)
A) Intubation is indicated when GCS is less than 8
B) Compound depressed skull fracture is a surgical emergency
C) Head is propped up to reduce intracranial pressure
D) Lumbar puncture can be done to reduce intracranial pressure e) Brain atrophy has a high risk of SDH
E) .
3. Which of the following is/are true of manifestation of RLN palsy after total thyroidectomy?
(2011.SUR.MCQ.09)
A) Hoarseness of voice
B) Stridor
C) Snoring
D) .
E) .
4. Lingual nerve is damaged during submandibular sialadenectomy. What is the most likely effect on tongue?
A) Loss of general + taste sensation in ipsilateral anterior 2/3 of tongue
B) Loss of general sensation in ipsilateral tongue + Ipsilateral taste in anterior 2/3 of tongue
C) Loss of ipsilateral general sensation in anterior 2/3 of tongue
D) Loss of general sensation ipsilateral tongue
E) Loss of taste sensation in ipsilateral anterior 2/3 of tongue
5. A 45-year-old school physical training teacher presented with severe headache and palpitations on exertion
which was recent in onset. On examination he has BP 190/110mmHg and pulse 105/min. No papilledema or
focal neurological signs. What is the most appropriate investigation to diagnosis? (2012.SUR.SBR.21)
A) 2D echo
B) 24-hour ambulatory blood pressure monitoring
C) CT scan of the head
D) Magnetic Resonance Imaging of Head
85
E) 24-hour urinary collection for metanephrine and normetanephrine level
6. 20-year-old man presented with a wrist drop following a road traffic accident. There are no collateral injuries,
but he can't recall details of the impact of fall. On examination there was impaired sensation in the anatomical
snuff box and impaired extension of fingers at metacarpophalangeal joints. All the upper limb reflexes are
preserved. What is the anatomical site of the lesion? (2012.SUR.SBR.22)
A) Anterior interosseous nerve
B) Posterior interosseous nerve
C) Radial nerve damage at shaft of humerus
D) Radial nerve damage at the wrist
E) Radial nerve damage at the axilla

Paediatrics – Neurology
1. 4yr old boy with viral encephalitis who was on IV fluids has low urine output during last 18hrs. Bladder is not
distended. BP normal. What is the basic investigation that would indicate the cause of low urine output?
(2008.PAED.MCQ.16)
A) Blood urea
B) FBC
C) Serum calcium
D) Serum creatinine
E) Serum sodium
2. 4-year-old girl presented to the ETU with fever and headache. On admission she drowsy. BP=80/60 mmHg,
PR=146 bpm, CRAFT=4s. What is the immediate management? (2008.PAED.SBR.21)
A) Transfer to ITU
B) Resuscitate with fresh blood.
C) N/saline 20ml/kg
D) N/ saline 10ml/kg
E) Dextran
3. A 10-year-old male presented with a GTC seizure for 30 minutes Seizure was controlled after administration
of IM Midazolam He was diagnosed with epilepsy at the age of 8 year and was on sodium valproate for 2 years.
There were no seizures for last 2 years. What would be the next management option? (2009.PAED.SBR.27)
A) Start carbamazepine
B) Measure sodium valproate level
C) Urgent CT brain
D) Arrange EEG
E) Revise sodium valproate dose
4. Regarding absence seizures, (2010.PAED.MCQ.05)
A) Responds well to carbamazepine
B) Imaging of brain should be advised
C) Event precipitated by hyperventilation
D) Eye lid fluttering is associated during the event
E) .
5. 3-day old baby boy with 2.2 kg of birth weight presented with poor activity and convulsions. On examination
he was pink, peripheral circulation was good. CBS is 20mg/dl. What is the best management option?
(2010.PAED.SBR.22)
A) 10% glucose bolus + breast feeding
B) 10% glucose bolus + 10% glucose infusion
C) 5% glucose bolus + infusion
D) 50% glucose bolus + 10% glucose infusion
86
E) 10% glucose infusion + breast feeding
6. 2-year-old baby, with 4 days' history of fever, coryzal synt & convulsions. Treated as for bacterial meningitis.
After 72 hours developed increased fever spikes and drowsiness. What is the likely cause? (2010.PAED.SBR.26)
A) Subdural effusion
B) Meningo-encephalitis
C) Cannula site infection
D) SIADH
E) TB meningitis 26
7. Hydrocephalus occurs due to, (2011.PAED.MCQ.04)
A) Aqueduct stenosis
B) Dandy Walker syndrome
C) Toxoplasmosis
D) Arnold Chiari malformation
E) Interventricular haemorrhage
8. Regarding meningitis in children, (2011.PAED.MCQ.09)
A) Adjunctive steroids indicated in the neonate
B) Communicating hydrocephalus is a complication of meningitis
C) Subdural effusion is a common complication of Neisseria meningitidis
D) .
E) .
9. Features suggestive of a space occupying lesion in a patient with afebrile seizure (2011.PAED.MCQ.12)
A) Recent deterioration of school performance
B) Presence of focal neurological signs
C) Early morning headache
D) Drowsiness following convulsions
E) Seizures lasting more than 5 minutes
10. 4-year-old boy presented to the emergency department with sudden onset unusual posture and movements.
He is alert. He has a tilted head and writhing movements of upper limbs. What is the most appropriate Initial
management? (2011.PAED.SBR.24)
A) .
B) .
C) .
D) .
E) .
11. A 6 months old baby, presented with irritability, fever and poor feeding. Irritable when handling and there is
neck retractability. O/E 2 vesicles noted on the upper chest. LP was done.
• RBC-220, Protein - 1.2g/dl (0.4-0.8)
• CSF WBC elevated with 90% lymphocytosis.
• CSF sugar marginally reduced than RBS.
What is the most likely diagnosis? (2011.PAED.SBR.39)
A) Complex febrile convulsions
B) Non accidental injury
C) Toxic shock syndrome
D) Partially treated bacterial meningitis
E) Varicella encephalitis

87
12. A 6 months old baby presents to the emergency department following a prolonged seizure. His heart rate is
55 bpm and his respiratory rate is 40/min. He also made some gurgling sounds while breathing Which of the
following is the next Immediate step in management? (2011.PAED.SBR.43)
A) Commence bag and mask ventilation
B) Oxygen therapy via nasal prongs
C) Place in neutral position
D) Suck out secretions
E) .
13. 4-year-old was brought to the emergency unit with sudden onset abnormal posture and movements. On
examination the child was alert. There were uncontrolled writhing movements in upper limbs and torticollis.
His sibling is being treated for fever and vomiting. What is the most appropriate management?
(2011.PAED.SBR.46)
A) Benzhexol
B) Midazolam
C) Chloral hydrate
D) Urgent EEG
E) Urgent CT
14. Regarding Acute flaccid paralysis (2012.PAED.MCQ.04)
A) Antibody mimicry happen between bacterial antigen and myelin proteins
B) Nerve conduction is reduced in lower limbs
C) It is a notifiable condition
D) Distal muscles are affected more than the proximal muscles
E) Is known to occur following vaccination
15. Which of the following drugs are correctly matched with the disease? (2012.PAED.MCQ.11)
A) Bacterial meningitis – Ceftazidime
B) SIADH- vasopressin
C) GBS - methylprednisolone
D) Kawasaki disease – Aspirin
E) Rheumatic heart disease – benzyl penicillin
16. Regarding cerebral tumour (2012.PAED.MCQ.12)
A) They are the commonest tumour in children
B) Occur commonly in supratentorial fossa
C) Craniopharyngioma causes distant metastasis disease
D) Medulloblastoma occurs in midline in posterior fossa
E) Treatment of choice is the surgery in most cases
17. A 4-year-old boy had a quarrel with another child in the classroom and fell, following which he developed
sudden jerky movements that settled on its own. Teacher took him to hospital and mother revealed that there
was a similar episode following MMR vaccination at 3 years, most appropriate management option,
(2012.PAED.SBR.28)
A) Admit and observe for head injury
B) Contrast enhanced CT scan
C) Get an ECG
D) Get an EEG
E) Reassure the mother
18. A 9-year-old child presented with fever, headache and was drowsy developed one episode of seizure on 3rd
day of illness. CSF report as follows. What is possible the diagnosis? (2012.PAED.SBR.29)

88
• Cell count – (lymphocyte 95%)
• Proteins – mildly elevated
• Glucose – almost normal
A) Acute Disseminated meningoencephalitis
B) Autoimmune encephalitis
C) Partially treated bacterial meningitis
D) TB meningitis
E) Viral encephalitis

89
10. Special Senses Module - Final MBBS Common MCQs
Medicine – Special Senses
Ophthalmology
1. Structures involved in light reflex, (2010.MED.MCQ.17)
A) Occipital cortex
B) Lateral geniculate body
C) Edinger Westphal nucleus
D) Oculomotor nerve
E) Ciliary ganglion
Dermatology
1. A rash with polygonal purple flat-topped papules becomes plaques, which is pruritic and commonly seen on
flexor surfaces at wrist is, (2008.MED.MCQ.27)
A) Lichen planus
B) Atopic eczema
C) Lichen sclerosis
D) psoriasis
E) contact dermatitis
2. Which of the following cause a characteristic rash in palms and soles? (2009.MED.MCQ.16)
A) Secondary syphilis
B) Herpes simplex virus
C) Condylomata accuminata
D) Keratoderma blennorrhagica
E) Arsenic poisoning
3. Koebner phenomenon is seen in (2010.MED.MCQ.15)
A) Psoriasis
B) Lichen planus
C) Molluscum contagiosum
D) Measles
E) Arsenic poisoning
4. Erythroderma can occur in, (2011.MED.MCQ.16)
A) Eczema
B) Psoriasis
C) Reaction to drugs
D) Fungal infection
E) Lichen planus
5. Regarding erysipelas (2012.MED.MCQ.07)
A) Edges are discrete and merge into the skin
B) Caused mainly by group A beta hemolytic streptococci
C) Affect dermis and superficial subcutaneous tissue
D) Systemic spread can cause multi organ dysfunction
E) Penicillins are the treatment of choice
6. 28-year-old female presented with generalized rash. It is 12 cm in size, concentric rings surrounded by central
lesion. She also complains of pruritus & itching & some of the lesions are seen inside the oral cavity. She was
recently treated for chest infection. What is the condition? (2010.MED.SBR)
A) Erythema multiforme
B) SLE
C) Secondary syphilis
90
D) Urticaria
E) Pemphigus vulgaris
7. 68 years old male patient diagnosed with type 2 DM presented with painful, red, swollen, hard, tender lesion
in the calf of right leg, what is organism responsible for this presentation? (2011.MED.SBR)
A) HSV
B) Staph aureus
C) Staph epidermidis
D) Streptococcus pyogenes
E) Strep pneumonia
8. 32-year-old female presents with swollen lips for 5 days. She is on oral hypoglycaemic medication for diabetes
which is well controlled. No other medical co morbidities. No pruritus or past allergic history. She has recently
used a new hair dye one week back. What is the most likely diagnosis? (2012.MED.SBR.25)
A) Urticaria
B) Angioedema
C) Anaphylaxis
D) Erythroderma
E) Pemphigus
ENT
1. Which of the following is/are examples of mitochondrial inheritance (2011.MED.MCQ.17)?
A) Charcot marie tooth disease
B) Friedreich Ataxia
C) Leber hereditary optic neuropathy
D) Progressive external ophthalmoplegia
E) Myoclonic epilepsy with ragged red fibers (MERRF)
Surgery – Special Senses
Ophthalmology
1. Elderly patient C/O Mashes and floaters in the eye. O/E there is a visual field defect. What is the condition?
(2008.SUR.SBR.29)
A) Retinal detachment
B) Vitreous hemorrhage
C) Glaucoma
D) Retinal artery occlusion
E) Posterior uveitis
2. Viral conjunctivitis (2009.SUR.MCQ.18)
A) Causes pain
B) Causes halos around light
C) Does not impair the visual acuity
D) Has gritty sensation
E) Causes small irregular pupils
Dermatology
1. 50 years old farmer presented with a skin lesion on his right foot. It was an ulceration with smoothly rolled up
margins & pearly color. What is the most likely diagnosis? (2010.SUR.SBR.44)
A) Basal cell CA
B) Cock's peculiar tumor
C) Melanoma
D) Seborrheic dermatitis
E) Squamous cell CA

91
ENT
1. A 25yr old man received a direct blow to his nose during a fight. What is the best Indication for ENT referral?
(2008.SUR.SBR.28)
A) Swelling of nose
B) Laceration over nose
C) Septal deviation
D) Septal hematoma
E) Epistaxis
2. Middle ear infections (2009.SUR.MCQ.16)
A) Vertigo is a feature
B) Common in children
C) Majority are hematogenous
D) Pain is a prominent feature in chronic otitis media
E) Is a cause of meningitis
3. 28-year male, ear pain, vertigo and unilateral facial palsy. There are vesicles around the ear. What is the
diagnosis? (2011.SUR.SBR.37)
A) Acoustic neuroma
B) Bell's palsy
C) Lyme disease
D) Mucoepidermoid carcinoma of parotid gland
E) Ramsey Hunt syndrome
Paediatrics – Special Senses
Ophthalmology
1. 8.5 yr old girl from Medawachchiya. Mother complains she is clumsy. During the latter part of the day she falls
knocking on furniture and objects fall from her hand. Has a greyish lesion on her conjunctiva. What is the
management? (2008.PAED.MCQ)
A) Dietary modifications
B) Estimate retinol levels in blood
C) Give Vitamin A mega dose
D) Reassure
E) MRI scan of her brain
Dermatology
1. 8-year-old a febrile boy presented with 2 weeks history of ecchymotic patches. No mucosal bleeding or
lymphadenopathy. Hb-12, WBC 10500, N 58%, L 40%, PLT 24000. Management? (2008.PAED.MCQ)
A) Manage as an outpatient.
B) Start with immunoglobulin followed by steroids
C) Give only immunoglobulin
D) Admit, observe as an Inward patient for new bleeding and repeat plt count
E) Give prednisolone following a bone marrow study.
2. 6 months old baby chronic diarrhoea, perianal and peri oral erythematous ulcers & peeling off skin. What is
the management? (2008.PAED.MCQ)
A) Antidiarrheals
B) Steroid cream
C) Zinc supplementation
D) NSAIDs
E) Stop breast feeding
3. 2 months old Infant with scabies, What is the best treatment? (2008.PAED.SBR)
A) Ivermectin
92
B) 2% sulphur ointment
C) Benzyl Benzoate
D) Permethrin
E) Crotamiton
4. Girl with hair loss, oral ulcers, small joint arthritis. What is the best test to diagnose her condition?
(2008.PAED.SBR)
A) ANA
B) Antibodies to double stranded DNA
C) ASOT
D) ESR
E) Rheumatoid factor
5. Benign neonatal skin conditions, (2009.PAED.MCQ)
A) Erythema toxicum
B) Mongolian blue spots
C) Epstein pearls
D) Mila
E) Ash leaf macules
6. Causes for oral pigmentation, (2009.PAED.MCQ)
A) Addison's disease
B) Pellagra
C) Peutz-Jeghers syndrome
D) Crohn's disease
E) Fanconi anaemia
7. Direct tissue involvement in Mycobacterium tuberculosis is seen in, (2009.PAED.MCQ)
A) Adenoma sebaceum
B) Erythema nodosum
C) Lupus vulgaris
D) Phlyctenular conjunctivitis
E) Scrofuloderma
8. Regarding atopic dermatitis, (2009.PAED.MCQ)
A) Commonly associated with allergic rhinitis
B) Itching is a main feature
C) Commonly the affected child has a dry skin
D) Typically occurs in late childhood
E) 1st line treatment is corticosteroids
9. A 2-month-old baby presented with generalized itching and widespread papules. Mother also had similar
lesions in palms. Most suitable treatment for the conditions is, (2009.PAED.SBR)
A) Benzyl benzoate
B) Malathion
C) Crotamiton
D) Sulphur
E) Permethrin
10. Regarding childhood skin disorders, (2010.PAED.MCQ)
A) Target lesions are seen in erythema multiforme
B) Palpable rash in buttocks is characteristic of hsp
C) Perineal candidiasis rash spares the flexures
D) Purple colour papule with central umbilication in molluscum contangiosum
E) Secondary infection in scabies not cause rheumatic fever

93
11. Fever for 4 days cough and cold rash on the day of the admission maculopapular rash starting behind the ear.
What is the most likely aetiology? (2010.PAED.SBR)
A) Parvovirus
B) Roseola infantum
C) Measles
D) Coxsachie
E) .
12. Post streptococcal glomerulonephritis, (2011.PAED.MCQ)
A) Common between 5-15 years of age
B) Is a type 2 hypersensitivity reaction
C) Bullous impetigo is a predisposing factor
D) Hypertensive encephalopathy is a complication
E) Causes CKD in majority
13. Toxin mediated skin conditions, (2011.PAED.MCQ)
A) Staphylococcal scalded skin syndrome
B) Toxic shock syndrome
C) Erythema marginatum
D) Bullous impetigo
E) Erythema infectiosum
F) Bullous pemphigus
14. Atopic dermatitis, (2012.PAED.MCQ)
A) Common in children than adolescents
B) Associated with filaggrin gene mutation in majority
C) Pruritus is a feature
D) Mimic seborrheic dermatitis
E) Best treated with systemic immunosuppressive
15. 8-month-old boy presented with itchy rash which is most discomforting at night. On examination the rash is
distributed predominantly on the palms and soles, groin, head and neck areas. Skin excoriation was there.
What is the best therapeutic option? (2012.PAED.SBR)
A) Aqueous cream
B) Micanozole
C) Topical hydrocortisone 5%
D) Permethrin cream
E) Benzoyl benzoate
16. 1yr old boy presented with fever and erythematous palms and soles. He has also cervical lymph nodes. Fever
didn't respond to simple antipyretics. He has mild redness on eyes. No organomegaly. WBC-28300, N=75%,
plt-increased, ESR=78mm/1" hr, Blood cultures no growth. What is the most probable diagnosis?
(2008.PAED.SBR)
A) JIA
B) Dengue
C) HSP
D) Kawasaki
E) SLE
17. Which of the following causes gum hypertrophy as side effect? (2010.PAED.MCQ.10)
A) Sodium valproate
B) Phenytoin sodium
C) Cyclosporine
D) Methotrexate
E) Sulphasalazine

94
ENT
1. A 2-year-old boy was transferred from a local hospital for further management for everlasting for 10 days.
Child was ill and febrile. There was redness of eyes and an inflamed cavity with cervical lymphadenopathy.
• WBC-78000/mm3 (N 78%, L 25%)
• ESR - 75 mm/1st hour
• Platelets - 635 000/mm3
What is the most likely diagnosis? (2009.PAED.SBR)
A) IMN
B) Kawasaki disease
C) Miliary TB
D) Measles
E) Roseola infantum
2. A child has a mild nasal bleeding due a direct blow to his nose. Initially the bleeding stopped and while
examining started to bleed again. What is the next step in the management? (2012.PAED.SBR)
A) ENT referral
B) Anterior nasal packing
C) Apply direct nasal pressure
D) Posterior nasal packing
E) .

95
11. Mental Health Module - Final MBBS Common MCQs
Psychopathology and MSE
MCQ
1. What are the normal experiences?
A) Hypnagogic hallucinations
B) Forgetfulness
C) Obsessional ruminations
D) Persecutory delusions
E) Illusions
F) Sad feeling
2. Regarding aetiology of psychiatric disorders.
A) High expressed emotions in the family is protective against relapse in schizophrenia.
B) Generalized anxiety disorder has 90% heritability.
C) Immigration is a risk factor.
D) Panic disorders has Mendelian inheritance.
E) Cortisol is elevated in depression
3. Which of the following psychological treatments are correctly paired with the psychopathology?
A) Narrated exposure therapy - specific phobia
B) Assertive training – aggressive behaviour
C) Dialectical behavioural therapy – antisocial personality
D) Cognitive behavioural therapy- social phobia
E) Eye movement desensitization and reprocessing –PTSD
4. Which of the following has a risk to others?
A) Anxiety disorders
B) Paranoid delusions
C) Ruminations
D) PTSD
E) .
5. Regarding insight,
A) Aware about the morbid change in oneself
B) In sight in mental state examination is stated as present or absent
C) Good insight has good compliance
D) Absent in neurotic disease
E) .
6. Obsessional thoughts are
A) Thoughts that enter an individual’s mind again and again in a stereotypical manner
B) Hoarding is an obsessional thought
C) .
D) .
E) .
SBR
7. A 30-year-old man is seen in the outpatient department. He complains that a group of men are following him
with the intension to harm him. He also complains that the newsreader on the television is talking about him.

96
He believes that the newsreader is sending special messages warning him that he is in danger. Which
psychopathology best explains the belief of this patient that the newsreader is sending him special messages
A) Grandiose delusion
B) Delusion of control
C) Persecutory delusion
D) Delusion of reference
E) Auditory hallucination

Bipolar Affective Disorder


MCQ
1. Regarding Bipolar Affective disorder
A) More prevalent in females
B) Suicide risk is increased
C) Can have only manic episodes
D) Prognosis is better than schizophrenia
E) Rapid cycling is more common in males
2. Regarding bipolar affective disorder
A) Time period between 2 episodes increases with time
B) Depressive episodes are commoner than manic episodes
C) Prevalence is equal in men and woman
D) Mean age of onset is 4th decade
E) Associated with anxiety disorders
3. Regarding bipolar disorder,
A) Irritable mood is a recognized feature of mania
B) Risk of second episode of mania is about 10%
C) Average duration of a manic episode is 1 week
D) Mood stabilizers should be given for more than 1 year
E) Prophylactic treatment is given only if patient has two manic episodes
SBR
4. 26-year-old married women presents in 1st manic episode with poor sleep disinhibition very aggressive and
threatening to go home. Most appropriate management in acute stage?
A) Li
B) Na valproate
C) Topiramate
D) ECT
E) Lamotrigine

Depressive Disorders
MCQ
1. Regarding postpartum depression
A) Occurs after 2-3 months after childbirth
B) If not treated adversely affect the cognitive development of infant
C) In Sri Lanka, Edinburgh depression scale use as screening tool
D) Sertraline is an effective treatment
E) ECT is an effective treatment
2. Regarding depression in old age

97
A) White matter changes in the brain are known to occur
B) Suicidal risk is not increased
C) Somatic complains is a common presentation
D) ECT is contraindicated
E) TCA are 1st line treatment
3. Following neurotransmitters are altered in depression
A) Noradrenaline
B) Dopamine
C) Serotonin
D) Glutamine
E) GABA
4. 40 y old female developed progressive low mood, loss of appetite, loss of weight after witnessing her
husband's death by drowning while going on a boat in a lagoon. What is more suggestive of depression than
grief?
A) Refuse to give his belongings to anyone
B) Crying while watching his favourite tv programmes
C) Worthlessness
D) Guilt that she could not ask him to wear a life jacket
E) .
5. Regarding perinatal psychiatric illnesses
A) SSRI is the treatment of choice in maternal blues
B) ECT is considered early in postpartum depression
C) Sertraline is contraindicated in pregnancy
D) Na valproate is given during breast feeding
E) Olanzapine is used as a mood stabilizer during pregnancy
6. Regarding depression
A) Occur in late adolescents
B) It is comorbid with substance use disorders
C) Abuse in childhood is predisposing factor
D) .
E) .
7. Organic causes of depression
A) Cushing’s syndrome
B) Hypothyroidism
C) Parkinson's disease
D) Multiple sclerosis
E) Cerebrovascular accidents
8. Regarding psychiatric disorders in post-partum period
A) mild depressive symptoms are common in early postpartum days
B) .
C) .
D) .
E) .
9. Regarding Depression,
A) Not working outside is a risk factor
B) Common among males
98
C) If the cause is found, the use of antidepressants is not necessary
D) Lithium is effective in unipolar depression
E) 2/3 of patients with a single episode of depression will have another episode of depression
SBR
10. A 45 yr male who had recent MI complain with lack of mood suicidal ideas, lack of interest. He doesn't want
to go to work thinking that it is meaningless. Best drug to manage these symptoms
A) Imipramine
B) Quetiapine
C) Fluoxetine
D) Sertraline
E) Venlafaxine
11. 60 years old lady diagnosed with recurrent depression presented to the ward after an attempt of jumping into
a moving train. On examination, she is restless. Further questioning revealed that she had been taking reduced
amount of food for past 3 months. She believed that her intestines are not working although she had normal
bowel habits. She was already on Risperidone and Venlafaxine. What is the next step of the management?
A) ECT
B) Sedate with haloperidol
C) Start antipsychotic
D) Surgical referral to rule out bowel pathology.
E) Insert an enema to relive the distress
12. A 24-year-old undergraduate who is often seen to be crying. His mother passed away 7 months ago. He has
lost 5 kg of weight over the last 2 months, and his marks at the university exams have decreased. He believes
that life is not worth living and wants to join his mother soon. What is the most likely diagnosis?
A) Dysthymia
B) Depressive episode
C) Normal grief
D) Adjustment disorder
E) Post-traumatic stress disorder
13. A 65-year-old male presented to the clinic with complaints of poor memory, reduced sleep and poor appetite
for 3 months. He also complains of being anxious and agitated. He is sad about the death of his wife 2 years
ago. He wished he were dead as he felt it was useless to live with failing memory. His children report that they
had not observed any significant problem in his memory
A) Grief reaction
B) Organic mood disorder
C) Depressive episode
D) Dementia
E) Generalised anxiety disorder
14. A 37-year-old mother 6 weeks’ post-partum admitted to neonatal ward. She refuses to feed the baby. She says
she is a strict vegetarian and believes it is the cause for the pyloric stenosis of the baby and wants her baby
euthanized to prevent it from further suffering. What is the best management option?
A) ECT
B) Imipramine
C) Sertraline
D) Alprazolam
E) Risperidone

99
Schizophrenia and Other Delusional Disorders
MCQ
1. Regarding delusional disorder
A) Occurs in late 20s
B) Delusional that are fleeting in nature
C) Antipsychotics are effective
D) It increases the risk of violence
E) Persistent hallucinations present
2. Regarding schizophrenia
A) Occur in early twenties
B) Genetic susceptibility plays a significant role in causation
C) Commoner in females than males
D) Around 1/3 of people develop treatment resistance
E) Majority poor Insight
3. Morbid jealousy
A) Common in male than female
B) False confessions by partner reduces jealousy
C) Comorbid harmful alcohol use is seen
D) Prognosis is often poor
E) Has low risk of violence
4. Factors that predict good prognosis in schizophrenia includes,
A) Family history
B) Acute presentation
C) Affective symptoms
D) Co-morbid substance abuse
E) Prominent negative symptoms
5. Which of the following favour Schizophrenia over depression?
A) Blunt mood
B) Delusion of nihilism
C) Thought broadcasting
D) Passivity
E) Delusion of guilt
6. Regarding pathological jealousy,
A) Common in males
B) It can be an overvalued idea
C) It is pathological if the person believes it on unsound reasoning
D) It is not pathological if the person seeks evidence in clothes of the partner
E) If it is a delusional thought, it is treated with antipsychotics
7. Which of the following symptoms if present more than 1 month, suggest schizophrenia rather than mania?
A) Delusions of control
B) Persecutory delusions
C) Flight of ideas
D) Tactile hallucinations
E) Thought echo

100
SBR
8. A 23-year-old university student is admitted to psychiatry unit with a history of changed behaviour for one-
month duration What is the most likely feature, if present, will suggest schizophrenia.
A) Believing batch mates are plotting to kill him
B) Believing he is controlled by a computer
C) Hearing his teachers voice commanding him to harm someone
D) Believing the news reader is referring to him on television
E) Claiming that a famous actress is in love with him
9. 22-year-old computer software engineer presented with persistent low mood and persecutory delusions and
Schizophrenia was diagnosed. Most appropriate management for him?
A) Haloperidol
B) Trifluoperazine
C) Risperidone
D) .
E) .
10. A young man with schizophrenia complains of hearing 3 criminals talking about him. He does not know where
they are. They don't mention his name but refer to him as the man with a mental illness. He hears them
planning to kill him. What is the most likely psychopathology?
A) Persecutory delusions
B) Delusion of reference
C) 2nd person auditory hallucinations
D) 3rd person auditory hallucinations
E) Delusion of control

Child Psychiatry
MCQ
1. Child sexual abuse in SL
A) Majority are penetrating sexual abuse
B) Perpetrator is rarely a known person
C) Majority are reported to the police
D) It doesn't impact on psychological well being
E) Males are rarely become a victim
2. Recognized causes of faecal soiling
A) Poor toilet training
B) Fear of using toilet
C) Hyperthyroidism
D) Constipation faecal overflow
E) Defiant behaviour
3. MCQ regarding features a 5-year-old child to suggest autism than another developmental disorder.
A) Delay in speech development until 3years
B) Avoid eye contacts
C) Hyperactive at school but not home
D) Play with one selected toy
E) Inter on maintaining the same routine
4. Conduct disorders,
A) Impulsivity is a recognised feature
101
B) ADHD is a comorbidity
C) Risperidone is mainstay of treatment
D) Parental criminality is a risk factor
E) Younger onset of presentation indicates good prognosis
5. Regarding a 9-year-old child with ADHD, which of the following may be seen.
A) Recurrent accidents
B) Good school performance
C) Socially disinhibited behaviour
D) Comorbid with anxiety
E) Situation specific behaviour problem
6. Regarding temper tantrums in a 3-year child
A) It is reduced by giving child’s demands
B) Ignoring temper tantrums initially causes escalation
C) Ignoring the temper tantrums will minimise them
D) It is effectively treated by fluoxetine
E) Punishment is effective
7. Selective mutism
A) Is a neurodegenerative disorder
B) Occur commonly in 3-5 years of age group
C) Is seen in autism
D) Best managed with sertraline
E) Comorbid with anxiety
8. Child sexual abuse
A) Exposing to pornography is method of non-contact sexual abuse
B) The abuser is usually a stranger
C) Is known to present with deteriorating school performance
D) Behavioural therapy is the mainstay
E) Later causes depression in adult life
SBR
9. 8-year-old girl was referred by the teacher because she was performing poorly academically. She is attentive
in the class and follows in instructions. She is unable to read her grade books, but she understands what is
taught in the classroom. See answers all questions when asked verbally but struggles to read and answer
questions in the paper. Her IQ test is normal. What is the best explanation for her problem?
A) Attention deficit hyperactivity disorder
B) Anxiety in the classroom 3
C) Bullying in the classroom
D) Specific reading disability
E) Mild mental retardation
10. 5-year-old boy presented with restlessness, and poor interaction with playmates. His mother says that he
would not listen to her commands. She also noticed that he does not play with his toy cars but likes to watch
their wheels rotating. He dislikes being touched. He prefers to eat the same food every day and gets distressed
when presented with anything different
A) Oppositional defiant disorder
B) Social anxiety disorder of childhood
C) Autism spectrum disorder
D) Obsessive compulsive disorder

102
E) Hyperkinetic disorder

Dementia, Delirium and Psychiatric Emergencies


MCQ
1. Features of Alzheimer's dementia
A) Aphasia
B) AP Agnosia
C) Anosmia
D) Avolition
E) Alogia
2. What are the reversible causes of dementia?
A) Alzheimer's
B) Folic acid deficiency
C) Normal pressure hydrocephalus
D) Hypothyroidism el Alcoholic
E) .
3. Regarding Alzheimer dementia
A) Recent memory loss is a feature.
B) Medial temporal lobe atrophy is seen in MRI
C) Hyper intensive bodies are evident in MRI D. Amyloid deposits can be seen inside the neuroses
D) Cognitive assessment aids the diagnosis
E) .
4. Delirium
A) Cause fragmented delusions
B) Attention is impaired
C) Alertness reduced
D) Preserved environmental perception
E) Consciousness is altered
5. Regarding neuroleptic malignant syndrome,
A) Fever
B) Persecutory delusions
C) Rigidity
D) Pressure of speech
E) Confusion
6. Delirium,
A) Does not present in children
B) Normal MMSE
C) Olanzapine is effective in treatment of aggression in delirium
D) Best managed in psychiatry Ward
E) Attention and concentration are not affected
7. Alzheimer’s disease,
A) Tau protein can be seen in intracellularly
B) .
C) .
D) .
E) .
103
SBR
8. A 40 yr old male is admitted following RTA. He has multiple lower limb fractures days nurses report him having
poor sleep restless and agitated. Feature most suggestive of delirium?
A) Auditory hallucination
B) Suspicion that ward staff is trying to harm him
C) Disoriented in time and place tactile hallucination
D) .
E) .
9. 30 yr old woman admitted with aggression was given IM Haloperidol 10mg, 3 hrs after giving haloperidol she
developed rolling up of eyes, neck deviated to a side protrusion of tongue. What is the best management
option for her?
A) O. Clonazepam 2mg stat
B) IM Benztropine 2mg stat
C) O. Benzhexol 4mg stat d
D) IV Diazepam 10mg stat
E) IV Hydrocortisone 100mg stat
10. A 60-year-old male presented to hospital with pneumonia. He has diabetes, non-alcoholic cirrhosis, and renal
impairment. During the 1st night at hospital he is observed to noisy, disoriented, argumentative, and
disturbing other patients. What is the medication of choice to manage his behaviour?
A) Diazepam
B) Amitriptyline
C) Haloperidol
D) Chlordiazepoxide
E) Chlorpromazine
11. 70-year-old female with dementia on donepezil. He daughter complains about recent onset wandering
behaviour of mother searching for their father who died 3 years ago. She worries that this will cause harm to
her mother. What is the best therapeutic option?
A) Add low dose risperidone
B) Low dose risperidone
C) Behavioural therapy
D) Explore contributory factors for her behaviour & modified them
E) Arranging cognitive Stimulation therapy
12. A 27-year-old diagnosed patient with bipolar affective disorder, started abusing others and assaulting them.
What is the next best step in the management?
A) Sedate with olanzapine
B) Talk to him and try to calm him done
C) Restrain him physically
D) Contact the police
E) .

Anxiety Disorders
MCQ
1. Regarding phobias
A) Phobias are commoner in females
B) Depression is a common cause of specific Phobias
C) Feeling loss of control is a feature of agoraphobia
D) Behavioural therapy targets exposure to avoided situations
104
E) RI can be used as a treatment
2. Regarding social phobia
A) It rarely occurs in males
B) Occur in late 20s
C) Behavioural therapy includes exposure to feared situation
D) .
E) .
3. Physical illnesses present with symptoms of anxiety include
A) Pheochromocytoma
B) Hyperthyroidism
C) Narcolepsy
D) Temporal lobe epilepsy
E) .
4. Regarding OCD
A) Obsessive thoughts are regard as senseless by the patient
B) Compulsive behaviour results in completion of inherently useful tasks
C) Counting is an overt compulsion
D) Undoing is a feature
E) Symptoms must be present for at least one month for diagnosis
5. Regarding panic disorder
A) Carpopedal spasm is a presenting feature
B) Screening for organic causes is indicated
C) Episodes occur in a predictable manner
D) Treated with SSRI effectively
E) Treated with CBT effectively
6. Characteristic features of phobic anxiety disorder,
A) Anxiety for specific stimuli
B) Avoidance of stimuli
C) Anticipatory anxiety
D) Autonomic symptoms
E) Depression
7. In agoraphobia,
A) Panic attacks are frequent
B) Patient fears open spaces
C) Anxiety reduces when accompanied by a close friend
D) Commoner in males
E) Has higher frequency of developing schizophrenia
8. Regarding OCD
A) Patient believes obsessional thoughts are his/her own
B) Associated with anxiety disorders
C) Obsessional thoughts are rational
D) Compulsions are related to content of obsessions
E) Occurrence of obsessional thoughts are unpleasant
F) Occurrence is commoner in males
G) Obsessional images are a form of hallucination

105
SBR
9. A 16-year-old girl presents with repeated hand washing. She fears she will fall ill and won't be able to sit for
her O/L exams if she doesn't do so. She knows that these thoughts are irrational but is unable to resist them.
What is the best management option for her?
A) Graded exposure
B) Exposure and response prevention
C) Supportive therapy
D) Cognitive therapy
E) Mindfulness
10. A 22-year-old female university student experiences severe palpitation, sweating and anxiety, when she must
make at a presentation at the examination. She says that she always felt anxious when speaking in public. She
has now considered leaving the course due to this condition. What is the most likely diagnosis?
A) Depressive episode
B) Exam phobia
C) Generalised anxiety disorder
D) Social phobia
E) Panic disorder
11. A 25-year-old male present with recurrent urge to stab his family members which increase when seeing
kitchen knives. This made him avoid going to kitchen. He is very distressed. What is the most appropriate
therapeutic method for him?
A) Graded exposure
B) Cognitive therapy
C) .
D) .
E) .

Suicide and Psychoactive Substance Use


MCQ
1. The following indicate a high suicidal Intent
A) Planning the attempt for 2 weeks
B) Seeking for medication right after the act
C) .
D) .
E) .
2. WOF are ICD 10 criteria for alcohol dependence?
A) Consumed for more than five years.
B) Tolerance
C) Presence of seizures due to alcohol withdrawal
D) Early morning intake
E) Difficulty controlling the amount of drinking
F) Strong desire
3. Regarding psycho active substance
A) Hashish is processed by cannabis sativa
B) Heroin withdrawal causes constipation
C) Chronic cannabis use causes psychosis
D) Amphetamine is a CNS depressant
E) Methadone is use heroin dependent
106
4. Drugs used in Heroin withdrawal
A) Clonidine
B) NSAIDS
C) Haloperidol
D) Methadone
E) Thiamine
5. Regarding substance abuse
A) Cannabis will be present in urine up to 3 weeks after withdrawal in chronic abusers
B) Methadone has a long half-life than heroin
C) Delirium occurs in benzodiazepine withdrawal
D) Formication is seen in cannabis withdrawal
E) Formication is experienced in cocaine abuse
6. Regarding Wernicke’s encephalopathy,
A) Occurs due to thiamine deficiency
B) Hyperemesis is a cause for this
C) Ophthalmoplegia is a feature
D) Confusion is a feature
E) Glucose should be administered before treating with thiamine
7. Measures effective in reducing suicide rate at population level
A) Widespread media coverage of suicide
B) Banning highly toxic agrochemicals
C) Making attempting suicide a punishable offence
D) Making protective fences to cover suicidal heights
E) Improving access to medical services
8. 18yr old girl coming with paracetamol overdose. Blood paracetamol level not available. What is the best
indicator for future attack?
A) Excessive crying while in ward
B) Frequent conflicts with boyfriend
C) Past history of self-harm
D) Aggressive behaviour towards the staff
E) .
9. Regarding Disulfiram, treatment in alcohol dependence
A) Reduce craving
B) Inhibit alcohol dehydrogenase
C) Safer to use in those who are on alcohol
D) Known to cause headache
E) Has a metallic taste
10. To reduce alcohol consumption in the community,
A) Reduce the number of sales outlets
B) Reduce the number of open hours during which alcohol is sold
C) Restrictions for advertisements
D) Increase the price of alcohol
E) Reducing taxes for beer
11. Regarding substance use,
A) Heroin can cause respiratory depression with single iv dose
B) T 1/2 of nicotine is 12 hrs
107
C) Chronic alcohol use can cause myopathy
D) .
E) .
SBR
12. 45-year M Male with features of alcohol dependence presenting abdominal pain. He doesn't think his drinking
is a problem and to matter. What is the best intervention for this man?
A) Tech refusal skills
B) Prescribe disulfiram
C) Discuss about advantages and disadvantages of alcohol
D) Advice to stop associating with alcohol consuming friends
E) Cognitive therapy
13. A 16-year-old girl who is on treatment for moderate depression has poor sleep. She claims that her family
members don't love her anymore. She feels worthless about her future and has stopped her medications. She
doesn't like to go to school and wants to stay alone at home. Her mother has four 50 paracetamol tablets
hidden in her cupboard. What is the most immediate important management?
A) Treatment of depression
B) Management of suicidal risk
C) Family therapy
D) Explore underlying stressors
E) Dialectical behavioural therapy

Stress related, Personality, Somatoform Disorders


MCQ
1. Dissociative disorder
A) Caused deliberately
B) Gradual onset of symptoms
C) Those affected are suggestible
D) Outcome is good if symptoms identified earlier
E) Treatment with antidepressant is useful
2. Acute stress reaction
A) Occurs after 3 days of situation
B) Ré experiencing of the trauma is a feature
C) Disorientation indicate organic pathology
D) Presence of depersonalization suggest underlying psychiatric disorder
E) By giving antidepressants can prevent developing into PTSD
3. 28-year-old previously healthy man presents to hospital with acute onset inability to speak His wife complains
that this happened after she discovered an extramarital relationship of him. What is the most likely diagnosis?
A) Malingering
B) Acute stress reaction
C) Adjustment disorder
D) Dissociative disorder
E) Post-traumatic stress disorder
4. Patients with somatoform disorder
A) Fakes their symptoms
B) Worries about having serious physical illness
C) Improves with cognitive behavioural therapy

108
D) Treated with placebo
E) Excessive usage of health care facilities
5. In PTSD,
A) Flashbacks of the incident is a feature
B) Caused after an overwhelming stressor
C) Debriefing immediate after a major disaster will reduce the risk of PTSD
D) Antipsychotic are the first line in treatment
E) Avoidance of the things associated with the incident
6. Regarding PTSD
A) Re-experiencing is a characteristic feature
B) More affected after natural disaster than man-made trauma
C) Depressive symptoms are common
D) Persistent avoiding of stimuli associated with trauma
E) Eye movement desensitisation and reprocessing is a treatment option
SBR
7. A man has assaulted his female partner at a party. He claimed that she has disobeyed him by refusing to dance
with him. He claimed that his word was not respected by his wife and that she deserves what she got. The
woman was bleeding from the nose, but he did not express any regret for his actions. He has used alcohol
regularly but was not intoxicated at the time of intoxication. On investigation, it was found that he has a long
history of violence, with several imprisonments,
A) Morbid jealousy
B) Alcohol withdrawal syndrome
C) Manic episode
D) Antisocial personality disorder
E) Narcissistic personality disorder
8. A 35-year-old female brought to the hospital by rescue workers after her village was destroyed by landslides.
You see the patient one day after the incident. She is tearful and distressed. What is the most appropriate
1ststep in the management of this child?
A) Psychological 1st aid
B) Trauma focussed cognitive behavioural therapy
C) Prescribing antidepressants
D) Interpersonal therapy
E) Progressive muscle relaxation
9. A 27-year-old army soldier was admitted to the hospital due to headache without informing his senior officers.
He was normal 1 week ago. He is not cooperating for the physical examination. His wedding is to be held in
another 3 days, and he has texted his girlfriend that however he will make it to the wedding, even though he
knew that there is a training session on that day. What is the most likely diagnosis?
A) Malingering
B) Somatoform disorder
C) Adjustment disorder
D) Depressive episode
E) .
10. 34-year-old single unemployed women suddenly started to act like her grandmother in her sister’s wedding.
She was distressed by her relations because they were continuously being asking why her younger sister is
marrying before her. Most suitable act for her behaviour is,
A) Acute stress reaction

109
B) Malingering
C) Dissociative disorder
D) Adjustment disorder
E) .

Disorders of Eating and Sex


MCQ
1. Which of the following are disorder of sexual preference?
A) Voyeurism
B) Exhibitionism
C) Paedophilia
D) Fetishism
E) Homosexuality
2. Physical features of Anorexia nervosa include
A) Menorrhagia
B) Parotid gland enlargement
C) Bradycardia
D) Heightened sensitivity to cold
E) Dry skin
3. Regarding disorders of sexual preference
A) Paedophilia has high prevalence among males
B) Sexual sadism involves experiencing suffering or humiliation as a sexual pleasure
C) Wearing dressing of opposite gender is called transvestism
D) Frotteurism is having sexual excitement when observing others having a sexual intercourse
E) Fetishism is getting sexual pleasure by inanimate objects
4. Regarding anorexia nervosa
A) Weight is maintained at least 15% below the standard weight expected
B) Patient identify her body weight is low
C) Excessive thinking about body weight appear as a delusion
D) Sudden increase in weight to achieve normal weight is recommended
E) Wants to be very thin
5. Which of the following are causes for erectile dysfunction?
A) Depression
B) Diabetic neuropathy
C) Propranolol
D) Tobacco smoking
E) Performance anxiety
6. Regarding anorexia nervosa,
A) Difficulty in swallowing
B) Distorted body image
C) Excessive consumption of carbohydrate rich food
D) Amenorrhoea
E) .
SBR
7. A 28-year-old unemployed married man for 2 years presented with difficulty in getting erection. Initially there
was no such a problem. Recently he had quarrels with his wife due to financial problems. He has early morning
110
erection and no problem with masturbation. He is not a smoker but occasionally consumes alcohol. He is not
diagnosed to have diabetes mellitus. What is the most likely cause for his sexual dysfunction?
A) Personality trait/ alcohol
B) Organic pathology
C) Depression
D) Marital disharmony
E) Reduced libido

Psychiatry and Law


MCQ
1. Risk factors for criminal behaviours
A) Low intelligence
B) Child of a small family
C) Psychosocial deprivation
D) Substance abuse
E) Harsh parenting
2. Which of the following are regarded as instances where confidentiality can be breached?
A) Disclosing the details about the disease to get further opinion on management from another doctor in an
emergency
B) Safeguarding a partner who is about to get seriously injured by her the partner
C) Disclosing the details about a disease when asked at the court of law
D) Disclosing the paedophilic act of a teacher to the relevant authorities
E) Details about the sexual fantasies of one to his/her partner in couple therapy
3. Patient with psychiatric illness
A) Not able to give evidence to the court of law
B) Not punishable for his offence
C) Able to make a will
D) Can get a job in government setup
E) Is at risk of divorce being issued due to his/her psychiatric illness

Epilepsy in Psychiatry
MCQ
1. Regarding epilepsy and mental disorders,
A) Patients with epilepsy have increased suicidal rate than normal
B) Olfactory hallucinations occur in temporal lobe epilepsy
C) Delirium in the post Ictal phase
D) Schizophrenia like psychosis is seen in Epilepsy
E) .
2. Regarding epilepsy
A) Post-ictal drowsiness is preceded by a lucid interval
B) Religious delusions in the inter ictal phase
C) Is a risk factor for depression
D) Predispose for suicides
E) .

111
Clinical use of Psychotropics
MCQ
1. Regarding Lithium carbonate
A) Is contraindicated in lactation
B) Cause polyuria as a side effect following drugs and side effects are correctly matched
C) Imipramine – diarrhoea
D) Chlorpromazine – nausea
E) Risperidone – galactorrhoea
2. Side effects
A) Clozapine - increase prolactin secretion
B) Risperidone - weight gain
C) Venlafaxine - sexual dysfunction
D) Fluoxetine - hyponatraemia
E) Chlorpromazine - tardive dyskinesia
3. When compared to 1st generation antipsychotics, 2nd generation antipsychotics,
A) Causes more extrapyramidal side effects
B) Causes more neuroleptic malignant syndrome
C) Causes more metabolic changes
D) Causes more hyperprolactinaemia
E) Causes more akathisia
4. Lithium is contraindicated in,
A) IHD
B) Hypothyroidism
C) Fine tremors
D) Epilepsy
E) History of toxicity with NSAIDS usage
5. SSRIs,
A) Are known to increase risk of bleeding
B) Are not safe for patient with angina
C) .
D) .
E) .
SBR
6. 35-year-old female who is on Olanzapine for schizophrenia for the past 1 year, complains of weight gain. She
has had her last episode 2 years back. What's the best management option?
A) Change olanzapine to aripiprazole
B) Increase the olanzapine dose to 20 mg
C) Reduce olanzapine to 5mg
D) Continue Risperidone with exercise and diet control
E) Stop olanzapine and monitor for relapse
7. A 25-year-old male with a depressive episode is seen in the clinic, 4 weeks after the initial presentation. He is
on sertraline 50mg in the morning. He reports only mild improvement despite treatment adherence. What is
the most important next step in the management of this patient?
A) Change the antidepressant
B) Add a benzodiazepine
112
C) Reassure the patient and monitor for further 2 weeks
D) Reduce the dose of sertraline
E) Add another antidepressant
8. A 55-year-old man with bipolar affective disorder and diabetes is prescribed 1 week of diclofenac sodium for
his headache. He presents with vomiting and diarrhoea. The patient is tremulous, unsteady and dehydrated.
He is admitted to the medical casualty unit. His lithium level is 1.5mmol/L. What is the most important
immediate management of this patient?
A) Start IV normal saline
B) Increase oral fluid intake
C) Investigate for further infection
D) Arrange urgent haemodialysis
E) Make an urgent psychiatric referral
9. A 23-year-old male recently diagnosed with schizophrenia, is reviewed a month after being discharged from
inpatient care. He complains of drooling of saliva, tremors, and rigidity of both hands. He is on oral risperidone
4mg daily. The psychiatrist decides to change the antipsychotic as the side effects significantly impair patient’s
functioning. He has not been on any other antipsychotic before. What antipsychotic is the best choice for this
patient
A) Clozapine
B) Haloperidol
C) Chlorpromazine
D) Olanzapine
E) Fluphenazine
10. 55-year-old diagnosed patient with schizophrenia, treated with olanzapine 40mg for 4 months, later changed
to risperidone 8mg for 4 months, comes to clinic with poor response. What is the next treatment option?
A) Two antipsychotic combination
B) Stop olanzapine and add haloperidol
C) Stop olanzapine and add clozapine
D) Stop olanzapine and add fluphenazine
E) ECT

113
12. ML Module - Final MBBS Common MCQs
• No MCQs found from Medicolegal Module

114
13. MSK Module - Final MBBS Common MCQs
Medicine – MSK (Rheumatology)
MCQ
1. Regarding acute gout
A) Low dose aspirin lowers uric acid excretion
B) Uric acid concentration reduces after an acute attack
C) Allopurinol should be given within the 1 week of an acute attack
D) Common in 1 metatarsophalangeal joint
E) Responds well to NSAIDS
2. What are the features of osteoarthritis?
A) Increased ESR
B) Crepitus over the joint
C) Osteophytes in the radiograph
D) Limb deformity
E) Joint effusion
3. Regarding gout
A) Subcutaneous nodules
B) Symmetrical involvement in small joints of hands and feet
C) Aggravated by alcohol
D) Metabolic syndrome is a risk factor
E) Urate crystals in synovial fluid
4. Features of sero negative spondyloarthritis?
A) acute anterior uveitis
B) inflammatory type low back pain
C) anti CCP
D) bilateral symmetrical small joint arthritis
E) Sacroiliitis
SBR
5. 56-year-old man presented with tender swollen 1" metatarsophalangeal joint. He was on frusemide for ankle
oedema, Investigation revealed Hb - 10.1g/dl, WBC - 19,000, ESR - 95, Scr - 3.42. what is the most likely
diagnosis?
A) Gouty arthropathy
B) Cellulitis
C) TB arthropathy
D) Rheumatoid arthritis
E) Septic arthritis
6. 30-year-old male coming with 6 m hx of lower back pain. Ole limited flexion and extension in spine. Diagnostic
investigation.
A) HLAB27
B) ALP
C) X RAY
D) SERUM CALCIUM
E) .

115
7. 40-year-old male with history of back pain currently presents with a painful knee joint that has persisting
morning stiffness for more than two hours duration. Which of the following describes the most likely
condition?
A) Ankylosing spondylitis
B) Psoriatic arthritis
C) .
D) Reiter’s syndrome
E) Rheumatoid arthritis

Surgery – MSK (Orthopaedic Surgery)


MCQ
1. Regarding shoulder dislocation,
A) Commonly occurred in abducted & externally rotated position
B) Is associated with rotator cuff tears
C) Recurrent dislocations are due to rotator cuff tears
D) Commonest is posterior dislocation
E) Needs open reduction
2. Regarding osteomyelitis
A) Dead bone inside the medullary canal is known as involucrum
B) Usually affects the metaphysis of the bone
C) Cause by Staphylococcus aureus.
D) Salmonella is the cause in patient with sickle cell disease.
E) New bone formation beneath periosteum called as sequestrum.
3. Causes for avascular necrosis of head of femur
A) Excessive alcohol consumption
B) Steroids
C) Radiotherapy
D) Hyperlipidaemia
E) Fracture of the sub trochanter of the femur
4. Regarding acute osteomyelitis in children,
A) Commonly lead to chronic osteomyelitis
B) Most common site is the metaphysis of the bone
C) Treatment is with parenteral antibiotics
D) Most common organism is staphylococcus aureus
E) Surgical intervention is the treatment of choice
SBR
5. A 7yr old boy was taken to surgery casualty ward with U knee joint swelling, with pain. WBC 22000, with
neutrophil predominance ESR 106mm/1st hr, CRP 9.6mg/dl. what is the management?
A) Start IV antibiotics
B) IV broad spectrum antibiotics.
C) Joint aspiration
D) Open surgical drainage
E) Splinting
6. 25-year-old drunk man was accidentally fallen and injured medial aspect of the elbow. What is the least likely
result that can be occurred?
A) Complete Ulnar nerve palsy
116
B) 1" dorsal interosseus paralysis
C) Loss of sensation on anterior aspect of the 5th finger
D) .
E) .
7. 45yr old female with above Knee POP cast for fracture of tibia fibula, 2 hr after complaining of severe pain. on
passive extension of fingers pain get worse what is the most appropriate management?
A) Immediate bivalve the cast with limb elevation
B) Diclofenac sodium suppository and cut window at fracture site
C) Diclofenac suppository & limb elevation
D) Oral nsaid
E) Complete removal of pop and limb elevation
8. A 40-year met with a motorbike accident and admitted to E room at 6 PM. He had suffered from fracture of
left tibia with an open wound. Distal pulses were present. X Ray showed a comminuted fracture of tibia. The
most appropriate management would be,
A) Urgent antibiotics and wound toileting in the following day morning
B) Wound toilet & internal fixation
C) Wound toilet & external fixation
D) Wound toilet & back slab
E) Wound toilet & skeletal traction
9. A 9-year-old girl presented with a right supracondylar fracture. On examination there was pallor and numbness
in that hand. Her distal radial pulse was absent in the same side Spo, was 84%. What is the most appropriate
next step in management?
A) Emergency open reduction & internal fixation
B) MUA & reassess the state of pulses
C) External fixation
D) Immediate vascular reconstruction
E) Reduction & back slab
10. 60-year-old male with patellar fracture immobilized for 1 week presented with calf pain. Duplex scan showed
thrombus in common femoral artery. Most appropriate management is
A) Heparin bolus plus infusion
B) Heparin infusion plus stocking
C) Heparin infusion plus warfarin
D) LMWH plus stocking
E) LMWH plus warfarin
11. 6-year-old child with fever, swelling, pain on right knee joint for 1-day duration. Leg was resistant to
movement. No trauma. WBC-25,000 (N-95%), ESR-110, CRP.96. What is the next step of management?
A) arthroplasty
B) joint fluid aspiration and culture
C) blood culture followed by antibiotics
D) US scan of the knee joint
E) Arthrotomy and drainage
12. 72 years old diabetic men, on oral hypoglycaemic, has high fever for 2 days with right knee joint swelling, he
had extremely limited joint movement and uss shows joint effusion. What is the most appropriate
management?
A) joint aspiration
B) IV antibiotics
C) Arthrotomy
117
D) MRI
E) Above knee pop cast
13. 50-year-old man present with right sided foot drop and numbness confined to lateral aspect of right foot.
What is the site of lesion?
A) L4 L5 level lesion
B) Spinal cord compression at T12 level
C) Cauda equina
D) Common peroneal nerve
E) L5/S1 root lesion
14. 74 years old lady who is having postural hypotension and recurrent falls, presented to the ward following a
fall with a shortened left leg which was externally rotated. X-ray hip showed an intracapsular neck of femur
fracture of the left leg. What is the most appropriate treatment for this patient?
A) Arthrodesis
B) Dynamic hip screw
C) Hemiarthroplasty
D) Skeletal traction
E) THR
15. 16-year-old presented with a swelling of lower one third of left thigh for 2 months duration. She has a low-
grade fever. On examination she has a hard lump on lower one third of left thigh which is warm to touch. X-
ray revealed a lesion arising from diaphysis of femur with onion skin appearance. What is the most likely
diagnosis?
A) Osteosarcoma
B) Ewings sarcoma
C) Osteomyelitis
D) Chondrosarcoma
E) Paget’s disease

Paediatrics – MSK
MCQ
1. Regarding JIA
A) Oligoarticular JIA will persist throughout as same
B) Permanent deformity of joints limited to polyarthritis JIA
C) High Remittent fever is a characteristic in systemic onset type
D) NSAIDs are the first line treatment in polyarticular type
E) Early physiotherapy prevents joint contractures
2. Girl with hair loss, oral ulcers, small joint arthritis. What is the best test to diagnose her condition?
A) ANA
B) Antibodies to double stranded DNA
C) ASOT
D) ESR
E) Rheumatoid factor
3. Causes of monoarthritis in a 4-year old
A) Psoriatic arthritis
B) Acute Lymphoblastic Leukaemia
C) Gout
D) Haemophilia A
E) Von Willebrand disease
118
SBR
4. A 10 yr old girl was diagnosed to have polyarticular JIA. She is having severe pain with morning stiffness even
though she is on high dose ibuprofen What is the most appropriate management?
A) Change NSAID to Diclofenac sodium
B) Start antibiotics
C) Intra articular steroids
D) Oral prednisolone
E) Methotrexate
5. 9 years old girl presented with right knee Joint swelling to the paediatric ward. Other than diarrheal episode
4 weeks ago medical examination is unremarkable. Temperature 36.9 C. Joint examination reveals limited
right knee joint movement and R/S KI Swelling. What is most likely diagnosis?
A) Pauciarticular JIA
B) Perthes disease
C) Reactive arthritis
D) Septic arthritis.
E) SLE

119
14. Trauma Module - Final MBBS Common MCQs
Surgery – Trauma
MCQ
1. Regarding suture materials
A) Silk is non absorbable
B) Polyglactin is hydrolysed in the body
C) Braiding increases the strength of sutures.
D) Monofilaments are more susceptible to bacterial infections than braided.
E) 2/0 polypropylene is used to suture a laceration in the face
2. 35-year-old patient after polytrauma was transfused with 15 U of blood. After 8 hours what are the
complications you can expect,
A) Bleeding from puncture site
B) Hypercalcemia
C) Hyperkalaemia
D) Hypothermia
E) Transfusion related acute lung injury (TRALI)
3. Regarding neurotrauma
A) Intubation is indicated when GCS is less than 8
B) Compound depressed skull fracture is a surgical emergency
C) Head is propped up to reduce intracranial pressure
D) Lumbar puncture can be done to reduce intracranial pressure
E) Brain atrophy has a high risk of SDH
4. Indications for thoracotomy in a patient with blunt trauma to the chest
A) 200ml /hour for 4 hours
B) Persistent cardiac tamponade
C) Massive unilateral pulmonary contusion with hypoxia
D) Haemothorax of 1500ml of volume at IC tube insertion
E) Flail chest
5. Which of the following are done during the primary survey?
A) ECG trace
B) Blood pressure and pulse measurement
C) Cervical spine stabilization
D) Examination of thoracolumbar spine
E) Ensuring adequate airway supply
6. Regarding triage
A) ATLS guideline should be used to sort injury pattern
B) Children are oven priority.
C) Do best for each individual
D) Done by most senior and experienced person
E) Done in daily hospital admissions
7. In ATLS adjuncts to primary survey are
A) Per rectal examination
B) blood pressure measurement
C) chest X-ray
D) FAST scan
120
E) Cervical X ray
8. Chest injuries that respond well to mechanical ventilation,
A) Cardiac tamponade
B) Flail chest
C) Open pneumothorax
D) Pulmonary contusion
E) Tension pneumothorax
9. Regarding abdominal trauma,
A) Damage control surgery can reduce irreversible physiological insult
B) Interventional radiologist can be involved in achieving haemostasis.
C) Laparoscopy is alternative to laparotomy in stable patients
D) Surgical intervention is mandatory for liver resection
E) .
SBR
10. 23 years boy admitted to the causality ward with a run over injury. He is conscious and rational; Chest X ray
shows air fluid level of the left lower hemithorax. After NG tube Insertion, it curved upward to the left lower
thorax What is the most appropriate management?
A) Laparotomy
B) Thoracotomy
C) IC Tube insertion
D) Observe and monitor
E) Peritoneal lavage
11. WOF is the most likely clinical finding in class 2 haemorrhagic shock.
A) HRC 100 bpm
B) Normal systolic blood pressure
C) Low diastolic blood pressure.
D) UOP > 30ml/hr
E) Confusion
12. 25-year-old male is admitted to ER after falling from a tree. He is dyspnoea, tachypnoea and tachycardic. On
examination his blood pressure is 80/45. R/side chest is hyper-resonant and there is reduced air entry on same
side. What is the most appropriate next step in management?
A) lC tube insertion
B) Wide bore needle insertion
C) Observe and monitor
D) .
E) .
13. Motor cyclist admitted to the ward after RTA. his PR 130 bpm, BP 80/50 mmHg.Hb 7g/dl. X-ray pelvis shows
multiple fractures. 4 units of blood given. best management option is?
A) Angiogram and embolism
B) Pelvic binding
C) Lupen and pelvic packing
D) .
E) .
14. A 25yr old man received a direct blow to his nose during a fight. What is the best Indication for ENT referral?
A) Swelling of nose
B) Laceration over nose
121
C) Septal deviation
D) Septal hematoma
E) Epistaxis
15. A 50-year-old male was crashed by a van. On examination there is guarding & rigidity in the right
hypochondrium FAST scan showed moderate amount of free fluid. Patient is stable. What is the next important
step in management?
A) Admit to ICU & close observation
B) Contrast enhanced CT abdomen
C) Diagnostic laparoscopy
D) Diagnostic peritoneal lavage
E) Exploratory laparotomy
16. A 20-year-old male was knocked down by a car. He could speak immediately but he was confused on admission
1 hr after the accident, He was found to be deeply unconscious GCS was 5/15 & had decorticate flexion to
pain. Pupils were asymmetrical but both pupils reacted to light slightly. What is the most likely diagnosis?
A) EDH
B) SDH
C) Concussion
D) Intracerebral haemorrhage
E) .
17. 19-year-old male was admitted to the emergency department with a crush injury to the pelvis Clinical diagnosis
of class 2 haemorrhagic shock was made. What is the most appropriate fluid resuscitation regime he should
receive?
A) Rapid infusion of 250ml Hartmann i MAP drops below 60 mmHg
B) Rapid infusion of 1000ml Hartmann
C) Rapid infusion of 250ml hypotonic saline
D) Rapid infusion of 1000 ml hetastarch
E) Rapid infusion of 4 units of O negative blood
18. A polytraumatic patient 25-year-old male admitted to ETU. He is dyspnoeic, GCS 9/15, brought to hospital.
He is on 50% Oxygen. His ABG results were, pH 7.25, PCO2 90 mmHg, pO2-60 mmHg, HCO3 30, BE 3 mmol/l.
What is the most appropriate management?
A) CPAP
B) Endotracheal intubation and ventilation
C) IV 8,4% NaHCO3
D) IV mannitol infusion
E) .
19. A 26-year-old man admitted following a road traffic accident. He is dyspnoeic. Neck veins are engorged.
Trachea midline. Lungs normal. PR 136 bpm, BP 80/50 mmHg, RR 35/min. What is the most likely condition?
A) Cardiac tamponade
B) Flail chest
C) Massive haemothorax
D) Pulmonary contusion
E) .
20. 46 years old man presented with closed fracture of tibia 5 hours ago. He has numbness of his toes & pain on
movements of toes, distal tibial pulse is present. What is the next best step in management?
A) Splinting
B) Limb elevation
C) Fasciotomy
122
D) Internal fixation
E) POP cast
21. 28year old man was admitted to the emergency unit following a fall from height with blunt trauma to the right
chest. PR-120, blood pressure -70/50, Spo2-84%. Hyper resonance over right lung fields with reduced breath
sounds. What is the immediate management?
A) FAST scan
B) Wait till ICU available
C) Chest tube
D) Needle thoracostomy
E) Urgent way
22. What are the Indications for thoracotomy in a patient with blunt trauma to the chest?
A) 200ml /hour for 4 hours
B) Persistent cardiac tamponade
C) Massive unilateral pulmonary contusion with hypoxia
D) Haemothorax of 1500ml of volume at IC tube insertion
E) Flail chest
23. 28-year-old male following an RTA was brought to the A & E department. He was having a compound tibial
fibular fracture with a significant tissue loss (>10cm). Which of the following method is best to replace the
tissue loss?
A) Flap repair
B) Healing by primary intention
C) Healing by secondary intention
D) Split skin graft
E) Secondary suturing
24. A 30-year-old man had sustained a flail chest after a blunt injury to chest following RTA. On examination he is
mildly dyspnoeic and maintaining normal Spo2 levels with 02 via nasal cannula. Chest x-ray had found to have
fractured ribs. No other injuries detected. What is the most appropriate management of this patient?
A) Insert an IC drain
B) Give analgesia and chest physiotherapy
C) Plan a surgery to correct rib fracture
D) Apply chest corset and Immobilize
E) Endotracheal tube insertion and ventilation
25. 20-year-old man presented with a wrist drop following a road traffic accident. There are no collateral injuries,
but he can’t recall details of the impact of fall. On examination there was impaired sensation in the anatomical
snuff box and impaired extension of fingers at metacarpophalangeal joints. All the upper limb reflexes are
preserved. What is the anatomical site of the lesion?
A) Anterior interosseous nerve
B) Posterior interosseous nerve
C) Radial nerve damage at shaft of humerus
D) Radial nerve damage at the wrist
E) Radial nerve damage at the axilla
26. Man fallen from a height. Neck pain and all 4-limb weakness. Upper limb weakness more than lower limb
and sensory loss more in upper limbs,
A) Anterior cord syndrome
B) Central cord syndrome
C) Complete transverse myelopathy
D) Cauda equina syndrome
123
E) Spinal shock syndrome
27. A burn patient weighing 50 kg, sustained a partial thickness burn of 30% body surface area. What is the
amount of fluid that should be given within first 8 hours of injury?
A) Hartmann 2000 mL
B) Hartman 3000 mL
C) Hartmann 6000 mL
D) Hartmann 1000 ml
E) Hartmann 1500 mL

124
15. Reproductive Health Module - Final MBBS Common MCQs
Obstetrics – Reproductive Health
Normal and Abnormal Labour
MCQ
1. Regarding occipito-posterior position
A) Face to pubis delivery occurs
B) Leads to deep transverse arrest
C) Leads to prolonged second stage of labour
D) Majority rotate to occipito-anterior position
E) The presenting diameter is suboccipito-bregmatic
2. Which of the following causes prolonged labour?
A) Chorioamnionitis
B) Epidural Analgesia
C) Inadequate uterine contraction
D) Malposition
E) Previous Caesarean section
3. The progress of labour is best determined by
A) Descent of the head in abdominal palpation
B) Dilatation of the cervix and descent of the head
C) Effacement and dilatation of the cervix
D) Frequency and intensity of contractions
E) Mother's behaviour during the labour
SBR
4. A 26-year primigravida is admitted with a history of intermittent abdominal at weeks of period of gestation.
Her antenatal period is uncomplicated, and frequency of contraction is 1 in 10 minutes. Fetal heart rate is 142/
min. vaginal examination reveals 2 cm dilated cervix with intact membranes.
A) Augment labour with oxytocin
B) Encourage ambulation and review her in four hours
C) Insert prostaglandin pessary and review in four hours
D) Keep her fasting and review in four hours
E) Send her to the labour room for observation
5. A woman in her second pregnancy has been in the second stage of labour for two hours. The foetus is in
cephalic presentation and the head is not palpable abdominally. On vaginal examination, the head is felt 2 cm
below two ischial spines in the right occipito lateral position. The foetal heart rate is 90/minutes. The
estimated foetal weight is 3 kg. What is the most appropriate management?
A) Apply Neville Barnes forceps
B) Apply Simpson's forceps
C) Perform a caesarean section
D) Perform manual rotation of the head
E) Perform vacuum extraction
6. A 36-year-old primigravida in labour is found to have a foetal heart rate of 80/ min lasting for four minutes.
Two fifths of the foetal head is palpable abdominally. Vaginal examination reveals 7 cm dilated cervix and
vertex is felt at the level of the ischial spines. There is no cord prolapse. What is the best management option
for her?

125
A) Immediate delivery by caesarean section
B) Immediate delivery by forceps
C) Immediate delivery by vacuum extraction
D) Keep her in the left lateral position and monitor
E) Perform foetal scalp blood sampling
7. A primigravida with an uncomplicated pregnancy was in labour. At 8 am the cervical dilatation was 4cm and
she had progressed to 6cm by 12 noon. The membranes were absent with clear liquor. The head was at the
level of the Ischial spines in the occipito anterior position with no caput or moulding. Uterine contractions
were two per 10 minutes. The foetal heart rate was 140 beats per minute. What is the most appropriated
management for the patient?
A) Administer 75mg of pethidine intramuscularly
B) Allow labour to continue until the cervicogram reaches the action line
C) Commence an intravenous infusion of oxytocin
D) Commence continuous foetal heart rate monitoring
E) Improve her hydration with 0.9% saline infusion
8. A 28 year old primi presented with spontaneous onset of labour at 40 weeks of POA in 1st stage of labour.
After 1 hour, examination shows that the cervix is fully dilated. 1/5h of head is palpable through abdomen FHS
is 120 bpm, caput and moulding, light meconium stained liquor is present. CTG was normal What is the most
appropriate management?
A) Administration of Pethidine
B) Emergency-LSCS
C) Oxytocin infusion
D) Review in 1 hour
E) Vacuum extraction
9. A 26-year low risk primigravida presented for labour 38 POG with cervical dilatation 4cm and normal CTG.
What are the steps in management of this patient?
A) Amniotomy
B) Encourage supine position
C) Labour companion
D) Start the partogram
E) Oral clear fluids should be given
10. 18-year-old primi mother presented with severe abdominal pain and PV bleeding. PR- 100bpm, BP – 100/70
mmHg. On abdominal palpation the abdomen was woody hard. Fetal demise was noted on USS. Bishops score
was 8. What is the most appropriate management option?
A) Assess and correct coagulation
B) Blood transfusion
C) Caesarian section
D) Forceps delivery
E) Induction of labour with prostaglandin

Foetal Distress
MCQ
1. Regarding foetal heart rate in CTG
A) Maternal fever causes foetal tachycardia
B) Cord compression causes variable decelerations
C) Obstruction of foetal head causes late decelerations
D) Pethidine causes type 2 decelerations
E) Tocolytics cause type 2 decelerations
126
SBR
2. A 23-year-old primigravida at term was induced with artificial rupture of membranes and Oxytocin infusion.
She was fully dilated for 30 minutes and the station of the head was at +2. Position of the head was left occiput
anterior. The liquor was moderately meconium stained and the CTG showed late decelerations. The most
appropriate next step in the management of this patient is to
A) Encourage the woman to bear down
B) Perform a wide episiotomy
C) Perform an emergency caesarean section
D) Perform vacuum extraction
E) Wait for 30 more minutes
3. 30-year prime at POA 38 with cervical dilatation of 8cm with a CTG showing variability of 4 per 30 minutes,
heart rate 146 with no acceleration, decelerations for 5 minutes that is still persisting. What is the best
management?
A) Category 1 caesarian section
B) Category 2 caesarian section
C) Fetal scalp blood sampling
D) Forceps delivery
E) Vacuum delivery

Instrumental Delivery
MCQ
1. Contraindication for vacuum delivery include
A) Face presentation
B) Gestational age less than 34 weeks
C) Maternal immune thrombocytopenia
D) Occipito transverse position
E) Previous caesarean section
2. Criteria that must be fulfilled for an instrumental delivery,
A) Moulding should not be there
B) Cervical dilatation is 8cm
C) One fifth or lesser is abdominal palpable
D) Presence of Bandl's ring
E) Rupture of membranes
3. Indications for forceps delivery,
A) Prolonged 2nd stage of labour
B) Deliver the aftercoming head in breech
C) Difficult delivery of shoulder
D) Maternal exhaustion
E) Face presentation
SBR
Induction of Labour
MCQ
SBR
1. A woman who is in her 4 pregnancy with previous 3 normal vaginal deliveries presented at active phase of 1st
part of labour. On examination cervix is 5cm dilated, 3 contractions per 10 minutes, each lasts 4 minutes and
127
an intact bulging membrane is present. Fetal heart sounds are normal. Her pregnancy is otherwise
uncomplicated. What is the most appropriate management?
A) Pethidine injection
B) Allow labour to progress
C) Syntocinon infusion
D) ARM and monitor progression
E) ARM and Syntocinon

Postpartum Haemorrhage and Obstetric Emergencies


MCQ
1. Drugs which can be used in obstetric haemorrhage,
A) Intramyometrial carboprost
B) IV ergometrine
C) Intravenous factor seven
D) Oral tranexamic acid
E) Rectal PGE2
2. Risk factors for post-partum haemorrhage,
A) Advanced maternal age
B) Chorioamnionitis
C) Iron deficiency anaemia
D) Obstetric cholestasis
E) Prolonged labour
3. Risk factors for shoulder dystocia
A) Diabetes mellitus
B) Past history of shoulder dystocia
C) Epidural anaesthesia
D) Preterm labour
E) Large baby
SBR
4. 40-year-old lady at 41 weeks of POA was induced with prostaglandins. Labour was prolonged and there was a
blood loss of 21. IV ergometrine and tranexamic acid and rectal prostaglandins were Even 2 units of blood
given. 2 hours later patient developed fever of 104F with rigors. Most likely cause for the fever is,
A) IV ergometrine
B) Blood transfusion
C) Induction with PG
D) Prolonged labour
E) Rectal prostaglandins
5. 35-year-old mother of one in her second pregnancy at 39 weeks of POA delivered a 3.2kg baby boy by a vaginal
delivery. She developed heavy bleeding after one hour after the delivery. Uterus is well below the umbilicus
and is contracted. Placenta is complete and no retained products inside. All the medical management failed.
Perineum was examined and no tears detected. 3 pints of blood were transfused. What is the most
appropriate next step of management?
A) Balloon tamponade
B) Lynch suture and modified compression suture
C) Uterine artery ligation
D) Postpartum hysterectomy
E) Selective embolization
128
6. A 25-year-old female with past history of LSCS, currently in her second pregnancy, is admitted for trial of
labour. What is the most consistent feature in relation to scar dehiscence/uterine rupture?
A) Cessation of uterine contractions
B) Foetal distress in CTG
C) Continuous pain
D) Maternal tachycardia
E) Vaginal bleeding
7. A primigravida who has had a normal delivery one hour ago has lost 750 ml of blood. Her heart rate was 80
beats per minute and blood pressure was 100/70 mmHg. She had been administered two bolus doses (5 IU)
of oxytocin, a 40 IU infusion of oxytocin and two doses of ergometrine 0.5 mg intravenously. The uterus was
not well contracted, and she continued to have a trickle of fresh blood. What is the most appropriate next
intervention?
A) Continue close observations
B) Exploration of the uterus
C) Packing of the uterus
D) Repeat ergometrine 0.5 mg
E) Start blood transfusion
8. A woman delivered the first of twins weighing 2.8 Kg. The second twin was in breech presentation. On vaginal
examination, the membranes were absent, and the cord was felt in the vagina. Cord pulsations were felt. The
breech was felt 1cm below the level of the Ischial spines. The foetal heart rate was 100 bpm. The best
management option is to
A) Fill the bladder with one litre of normal saline
B) Perform a breech extraction
C) Perform a caesarean section
D) Perform an assisted breech delivery
E) Replace the cord into the uterus
9. A 28-year-old woman with gestational diabetes complicated by polyhydramnios induced at 38 weeks. The
cardiotocography showed repetitive deep variable decelerations 5 minutes after amniotomy. What is the most
appropriate next step in the management?
A) Administer oxygen via face mask
B) Perform a vaginal examination
C) Prepare for an emergency caesarean section
D) Rapid infusion of normal saline
E) Turn the woman to the left lateral position
10. A 32 yr old mother delivered first twin vaginally Birth weight is 2.6kg, 2 twins is in breech presentation. On
vaginal examination, cord is felt, and it is pulsating. Breech is felt. What is next step in management?
A) Fill the bladder with normal saline
B) Put the mother to knee chest position
C) Emergency LSCS
D) Perform breech extraction
E) Pack the vagina with sterile packs
11. A woman starts to bleed profusely from cervix soon after delivery. Placenta and membranes were delivered
and complete. Uterus is relaxed and fails to contract with oxytocic drugs. Patient is pale, pulse rate is 120 bpm.
Blood pressure is 100/60 mmHg. What is the next step?
A) Suturing
B) Balloon tamponade
C) Uterine artery ligation d

129
D) Hysterectomy
E) .
12. 38-year-old primi with DM in her 2nd stage of labour. Cervix is fully dilated. Head is delivered completely. But
failed to deliver the shoulder. What is the immediate next step of management?
A) Release of the posterior arm
B) Caesarean section
C) Put legs into McRobert’s position
D) Symphysiotomy
E) Apply supra pubic pressure

Antepartum Haemorrhage
MCQ
1. Features of placental abruption
A) A blood-stained amniotic fluid
B) High presenting art
C) Low platelets
D) Foetal bradycardia
E) .
SBR
2. A 23-year-old primigravida presented at 36 weeks of gestation with continuous abdominal pain and mild
vaginal bleeding. On examination, she is moderately pale. Her blood pressure is 120/70 mmHg. Her blood
group is O negative. Uterus was tender and large CTG showed tachycardia what is the most appropriate
management option?
A) Give dexamethasone injection
B) Anti d immunoglobulin
C) Blood transfusion
D) Expedite delivery
E) Ultrasound scan to confirm placental abruption
3. A woman with an uncomplicated antenatal history presented for induction of labour for postdates. On
amniotomy, profuse vaginal bleeding and foetal bradycardia are developed. The most appropriate diagnosis?
A) Praevia
B) Placental abruption
C) Uterine rupture
D) Cervical tear
E) Vasa praevia
4. 28 years old, mother of 2 children, is in the 28th week of POA. Last child was delivered by LSCS. She was found
to have an anterior low-lying placenta. What is the next step of your management?
A) Give dexamethasone at 34 weeks for lung maturation
B) Elective caesarean section at 37 weeks
C) Immediate admission for observation
D) Pelvic MRI
E) Repeat USS at 32week

Malpresentations and Malposition


MCQ
1. Regarding brow presentation

130
A) Wait for spontaneous version until 41 weeks
B) Foetal scalp blood sampling is indicated if CTG is abnormal
C) Forceps delivery is indicated
D) Further deflexion causes face presentation
E) Presenting diameter is submento bregmatic diameter
2. Favourable features for ECV,
A) Anterior foetal back
B) Engaged breech
C) Flexed breech
D) Polyhydramnios
E) Posterior placenta
3. Landmarks of brow presentation
A) A posterior fontanelle
B) Supraorbital ridges
C) Nasal bridge
D) Alveolar margin
E) Mentum
SBR
4. A 32 years old woman in her second pregnancy at 37 weeks of gestation is in labour. She has had a previous
vaginal delivery of a baby weighing 3000 g. The antenatal period of the current pregnancy is uncomplicated.
The foetal head is 3/5th palpable abdominally. The estimated foetal weight is 2900 g. Vaginal examination
reveals a fully dilated cervix and a brow presentation. Liquor is clear. What is the most appropriate
management of this patient?
A) Apply vacuum extractor
B) Apply Wrigley’s forceps
C) Caesarean section
D) Observe and repeat vaginal examination in one hour
E) Start an oxytocin infusion and review in 30 minutes
5. 30 years old primi has a delayed second stage. On vaginal examination gum margins and malar eminences are
felt, station is 1. Chin is towards coccyx. What is the next step of management?
A) Perform emergency LSCS
B) Manual rotation and delivery??
C) Perform direct application of forceps and trial of forceps delivery
D) Observe for 1 hour
E) Vacuum delivery
6. A 32-year-old multiparous woman presented in her 3 pregnancy at a POA of 41 weeks. She has undergone 2
past vaginal deliveries. Foetus is in transverse le. No symptoms of labour and no other pregnancy related
complications or USS abnormalities. Most appropriate next step of management?
A) Await spontaneous onset of labour till 42 weeks
B) Perform LSCS
C) Perform external cephalic version immediately
D) Perform internal podalic version during 2 stage
E) Perform stabilising induction
7. A 34-year-old second para with a previous vaginal delivery of a baby of a 3.2kg attended the antenatal clinic
at 39 weeks. On examination the foetus was in transverse lie. Ultrasound scan confirmed a normally sited
placenta, adequate liquor volume and an average sized foetus. On vaginal examination the cervix was closed.
What is the most appropriated next step in her management?
131
A) Admit for observation
B) Elective caesarean section
C) External cephalic version
D) Re-examination after onset of labour
E) Review in one week
8. A multipara is in stage of labour with a fully dilated cervix. On vaginal examination, presenting part is soft and
eyes, nose and mouth are palpated in mento anterior position pelvis is adequate. Foetal heart rate on
auscultation is 90 bpm. What is the best management option?
A) Forceps delivery
B) Emergency LSCS
C) Do a CTG
D) Vacuum delivery
E) Reassure mother
9. A 30 year old p2c1 mother, c1 is uncomplicated NVD, presented at 36 weeks of POA with a diagnosis of breech
presentation. USS abdomen reveals a fundal placenta. Liquor is adequate and no foetal movements are
detected what is the appropriate management of this patient?
A) Admit her to the ward for ECV
B) LSCS at 38 weeks
C) Repeat USS after 1 week
D) Do clinical pelvimetry to assess the pelvis
E) Reassure and wait for Spontaneous onset of labour
10. 37 years old women in her 3rd pregnancy is in labour. Cervix is fully dilated, and head is fully engaged. Initially
the posterior fontanelle was felt at 5’o clock position. 1 hour later it was felt at 3’o clock position. Vaginal
examination reveals caput and grade 1 moulding. CTG is normal. What is the next step of management?
A) Caesarean section
B) Use of forceps
C) Rotate the head manually
D) Vacuum delivery
E) Review in 1 hour
11. 32-year-old mother in her 3rd pregnancy presented at 40 weeks of gestation. Foetus is in breech presentation.
Pregnancy is otherwise uncomplicated. CTG is normal. She is not in labour. What is the most appropriate
management option for her?
A) Wait until 41 weeks for spontaneous onset of labour
B) Beech extraction when cervix is fully dilated
C) Caesarean section
D) External cephalic version
E) External cephalic version after onset of labour
12. Pregnant women presented at 39 weeks of POA. Head fully engaged and cervix fully dilated. Chin is underneath
the pubis symphysis. CTG shows late deceleration. What is the most appropriate management?
A) Caesarean section
B) Forceps delivery
C) Manual rotation of head
D) Suprapubic pressure
E) Vacuum delivery

132
Twin Pregnancy
MCQ
SBR
1. A 32-year-old multigravida with a dichorionic diamniotic twin pregnancy is admitted at 37 weeks of gestation
in labour. The blood pressure is 140/90 mmHg. She also requests sterilization. Which of the following is the
most important factor in deciding the mode of delivery?
A) Blood pressure of 140/90 mmHg
B) Chorionicity of the pregnancy
C) Gravidity of the woman
D) Maternal request for sterilization
E) The presentation of the leading twin
2. A 35-year-old woman with dichorionic diamniotic twin pregnancy comes to the antenatal clinic at 15 weeks of
POA. Her sister has a child with Edward syndrome. She is anxious about her pregnancy. Which of the following
test is most reliable to exclude chromosomal abnormality in both foetus?
A) Amniocentesis
B) Alpha fetoprotein level
C) Foetal cell free DNA
D) Pregnancy associated plasma protein a level
E) Quadruple test

Medical Disorders
MCQ
1. Which of the following are preconception assessment care done for women with pre-existing diabetes
mellitus?
A) Perform OGTT
B) Daily supplementation of folic acid 400 g
C) Retinal examination
D) Target HbA1c below 6.1%
E) Test for microalbuminuria
2. Regarding epilepsy complicating pregnancy includes
A) Delivery at 38 weeks of gestation
B) Intake of 5mg of folic acid daily
C) Monotherapy is preferred over drug therapy
D) Screening for foetal anomalies
E) Withholding antiepileptics during the first trimester
3. Which of the following are recognized complications of uncontrolled hyperglycaemia occurring after 24 weeks
of POA?
A) Cardiomyopathy in the infant
B) Foetal sacral agenesis
C) Pregnancy induced hypertension
D) Neonatal hypoglycaemia
E) Placental abruption
4. Features of MgSO4 toxicity,
A) Bradycardia
B) Uterine hyperstimulation
C) Reduced urine output
133
D) Respiratory depression
E) Slurred speech
5. Mother in her POA of 38/52. Which of the following need referral to a cardiologist?
A) Heart rate of 90 bpm
B) Soft systolic murmur at right upper sternal edge
C) Dry cough with shortness of breath 1 month after delivery
D) Nocturnal cough and breathlessness in a multipara at POA of 36 weeks
E) .
6. Main cause for GORD during pregnancy
A) Increased acid secretion
B) Lower oesophageal sphincter relaxation
C) Delayed gastric emptying
D) Relaxation of upper oesophageal sphincter
E) Increased intra thoracic pressure
SBR
7. A 20-year-old primigravida at 38 weeks of gestation is admitted with increasing swelling of both her legs during
last three days. Her antenatal period is uncomplicated. Her blood pressure is 150/100 mmHg and modified
bishop score is 5. Her cardiotocograph is normal. What is the most important next step in her management?
A) Advise her to have bed rest and reassess in four hours
B) Carry out investigations
C) Induce with prostaglandin
D) Oral nifedipine to control the blood pressure
E) Perform emergency caesarean section
8. A 25-year-old primigravida at 12 weeks of gestation is found to have haemoglobin of 7 g/dl and she is
asymptomatic. What is the most important next step in her management?
A) Advise her to protein rich diet and review in one month
B) Arrange a blood transfusion
C) Perform full blood count and blood picture
D) Prescribe double dose of oral iron
E) Prescribe mebendazole
9. A previously healthy primigravida is admitted with a blood pressure of 220/120 mmHg at 31 weeks of
gestation. Urine dipstick reveals 2+ proteinuria. What is the management?
A) Administer a bolus dose of 4g of magnesium sulphate
B) Administer intravenous hydralazine
C) Administer oral labetalol
D) Catheterize the woman
E) Perform an emergency caesarean section
10. 37-year-old primi at 22 weeks POA previously not followed up in an antenatal clinic presented with acute onset
shortness of breath for one day duration. On examination, she was afebrile and pale, her pulse rate 110bpm,
blood pressure 110/60 mmHg, SpO2, 85% and RR 28. There was a diastolic murmur heard over apex and
bibasal fine crepitations. What is the most likely cause for?
A) Acute respiratory distress syndrome due to pre-eclampsia
B) Diabetic ketoacidosis
C) Her presentation. Influenza induced bronchopneumonia and sepsis
D) Submaximal pulmonary embolism due to cardiac lesion
E) Tight MS causing pulmonary oedema

134
11. A 25-year-old primigravida was admitted with a history of epigastric pain and severe frontal headache at a
period of amenorrhea of 35 weeks. Her blood pressure was 160/110 mmHg. She had a single foetus in the
cephalic presentation. She was not in labour. The CTG was normal. Her urine contained a significant amount
of albumin. The coagulation profile was normal. She was given a bolus dose of 4g of magnesium sulphate. The
cervix was uneffaced. The best management option is to
A) Administer oral nifedipine 20mg twice daily
B) Administer an intravenous injection of 10 mg of diazepam
C) Deliver the baby by caesarean section as soon as possible
D) Insert a prostaglandin pessary
E) Observe in the intensive care unit
12. A 30-year-old primigravida with severe pre-eclampsia at 30 weeks was treated with magnesium sulphate. Two
hours later examination revealed absent knee reflexes and a respiratory rate of 12/min. The next step in the
management is to
A) Administer intravenous dexamethasone
B) Discontinue MgSO4
C) Give calcium gluconate
D) Give oxygen by mask
E) Measure serum magnesium level 9
13. An obese primi attended to antenatal clinic at 28 weeks of POA with post-prandial blood sugar of 250 mg/dl.
What is most important next step in management?
A) Weight reduction
B) Dietary control
C) OGT
D) Six-point BSS
E) Start metformin
14. A 28-year-old primi mother presented to hospital at 36 weeks of POA with hypertension. On admission, her
bf was 150/100 mmHg, albumin was 2, pulse rate 82 bpm, respiratory rate 16/minute, SpO2 99% after 12
hours of admission her BP was 160/110 mmHg she complained- of headache, vomiting, visual halos and right
hypochondrial pain. On examination, ankle clonus was present, neck stiffness. What is the most appropriate
next step in management?
A) CTG
B) Iv MgSO4
C) .
D) Diazepam
E) .
15. 36year old primi who is on medical nutritional therapy for GDM found to have abdominal circumference of
92nd centile at 28 weeks of POA. What is the best management option for her?
A) Glibenclamide
B) Insulin
C) Metformin
D) Moderate exercise
E) Strict diet control

Foetal Growth Restriction


MCQ
1. Risk factors for foetal growth restriction include
A) Asymptomatic bacteriuria

135
B) Gestational diabetes mellitus
C) Hypothyroidism
D) Multiparity
E) Teenage pregnancy
2. Regarding FGR
A) Detailed structural survey is important.
B) Absent umbilical artery end diastolic flow warrants immediate
C) Section c. Karyotyping is indicated
D) If doppler parameters are normal pregnancy can be continued up to 40 weeks
E) Vaginal delivery is contraindicated
SBR
3. A 27-year-old primigravida is referred at 34 weeks of gestation to a tertiary hospital as her symphysio fundal
height is found to be 30cm. Her dates had been confirmed. What is the next step in the management of this
woman?
A) Administer dexamethasone
B) Perform an ultrasound scan for the foetal size
C) Perform biophysical profile of the foetus
D) Perform umbilical artery doppler studies
E) Re measure the symphysio fundal height in two weeks
4. A 28-year-old primigravida undergoes ultrasonographic examination at 36 weeks of gestation. It shows a
foetus with asymmetrical growth restriction. What is the ultrasonography finding most likely to support the
diagnosis of asymmetrical growth restriction?
A) Uterine artery, umbilical artery and middle cerebral artery doppler
B) High grade placenta??
C) Low AFI
D) Reduced biophysical profile
E) Reduced abdominal to head circumference ratio
5. A primigravida presenting a period of amenorrhea of 36 weeks is found to have a symphysis fundal height of
30cm. The pregnancy has been uneventful so far. What is the most important information required for her
further management?
A) A careful dietary history
B) Her body mass index
C) The current biparietal diameter of the foetus
D) Whether she had a dating scan
E) Whether she had a normal glucose tolerance test
6. A primi at 32 weeks of POA presents with foetal growth restriction and oligohydramnios. What is the most
appropriate next step in management?
A) Measure uterine artery blood flow
B) Amniocentesis
C) Measure umbilical artery blood flow
D) Perform CTG
E) Cordocentesis
7. Referred from a peripheral unit, a 25-year-old primi mother at 33 weeks of POG, USS reveals EFW below 3rd
centile, umbilical artery doppler EDV is absent. Steroids were given. Immediate next step of mx?
A) Biophysical profile
B) Caesarean section delivery

136
C) Daily CTG
D) Perform a ductus venosus doppler
E) Perform a middle cerebral artery doppler

Deep Vein Thrombosis


MCQ
SBR
1. A 30-year-old pregnant woman in her 3rd pregnancy with a POA of 8 weeks with viable foetus on USS. Her first
pregnancy was a miscarriage at 15 weeks. Second pregnancy was complicated by severe preeclampsia at 26
weeks which ended up in an intrauterine death, after second pregnancy she was found to have positive
anticardiolipin antibodies on 2 occasions 12 weeks apart. Which one of the following is the best management
option?
A) Start aspirin and LMWH immediately
B) Start aspirin and LMWH after 12 weeks of gestation
C) Start aspirin from 12 week onward
D) Start LMWH immediately
E) Start unfractionated heparin immediately

Iron Deficiency Anaemia


MCQ
SBR
1. A 30-year-old primigravida at 26 weeks of gestation was found to have a haemoglobin level of 7.5g/dl at a
routine check. Blood picture shows microcytic hypochromic anaemia. Which of the following is the first step
in the management of this patient?
A) Carry out coagulation studies
B) Nutritional advice
C) Oral iron therapy
D) Parenteral iron therapy
E) Urgent blood transfusion
2. A 29-year-old p6c5 mother in her 35 weeks of POA presented with several faintish attacks. Her hb was 7.5g/dl.
What is the best management option?
A) Treat with parenteral iron
B) Worm treatment
C) Blood transfusion
D) Dietary advice
E) Double dose oral iron treatment
3. A primi at 10 weeks POG, a diagnosed patient with beta thalassemia minor wants to know about the outcome
of her child. What is the best initial investigation?
A) Amniocentesis
B) Chorionic villi sampling
C) Measure hba2 of the father
D) Hba2 level in both parents
E) Osmotic fragility test of mother

Hyperemesis Gravidarum
MCQ
1. Regarding management of hyperemesis gravidarum,
137
A) Combination of antiemetics are avoided
B) Rapid correction in hyponatremia leads to maternal pontine myelinolysis
C) Steroids are contraindicated
D) Thromboprophylaxis is indicated in severe cases
E) Use of dextrose solution may precipitate Wernicke’s encephalopathy
SBR
2. A 23-year-old primigravida at 11 weeks of gestation presents with intractable vomiting reduced urine output,
loss of weight and inability to carry out her day to day work. What is the most appropriate first line treatment
for her?
A) Intravenous 0.9% NaCl solution
B) Intravenous 10% dextrose solution
C) Intravenous vitamin b12 with thiamine
D) Oral metoclopramide
E) Rectal domperidone

Perinatal Infections
MCQ
1. Regarding chicken pox in pregnancy
A) Immunity is confirmed by detecting igg antibodies in maternal serum
B) Infection at term is an indication for immediate delivery
C) Is prevented by post exposure administration of acyclovir
D) Is complicated by maternal pneumonia
E) Maternal infection before 20 weeks of gestation carries a risk of congenital varicella syndrome
2. Regarding GBS infection
A) Can be treated with co amoxiclav
B) Can cause postpartum endometritis
C) High vaginal swab is used for diagnosis
D) May lead to preterm labour
E) Is a cause of puerperal sepsis
SBR
3. A 26-year-old woman delivered a baby weighing 2.3 kg at40 weeks of gestation. The baby was normal at birth
but later developed chorioretinitis, blindness and strabismus. What is the most likely maternal infection which
caused this condition?
A) Chickenpox
B) Cytomegalovirus infection
C) Listeriosis
D) Rubella
E) Toxoplasmosis
4. 26yr old primipara, presented at POA of 30 weeks with positive hepatitis B surface antigen what is the best
management to prevent the baby being a chronic carrier?
A) Antivirals to neonate
B) Give antiviral treatment to the mother
C) Avoid breastfeeding
D) Elective caesarean section
E) Vaccination of the new-born

138
5. A primi mother presents at 6 weeks of POA with a contact history of varicella 2 days ago. Her varicella igg
antibodies are negative. What is the most appropriate next step of management?
A) Administer pooled human immunoglobulin
B) Check IgM levels in blood
C) Administer mother with VZIG (varicella zoster virus immunoglobulin) immediately
D) Advice on termination of pregnancy
E) Treatment with acyclovir
6. A woman in early labour at tem with intact membranes presented with acute onset multiple small painful
vulvar ulcers. She has not had similar episodes previously. The most appropriate management is to
A) Administer acyclovir intravenously
B) Administer broad spectrum antibiotics intravenously
C) Counsel the mother regarding the neonatal outcome
D) Obtain further instructions from a venereologist
E) Perform an emergency caesarean section
7. HIV positive primi mother on Zidovudine mono therapy for 28 weeks of POG. Admitted at 36 weeks with viral
count less than 20. What is the best management option?
A) Augmentation of labour
B) Caesarean section
C) Combined anti-retroviral therapy and wait for spontaneous induction of labour
D) Intramuscular corticosteroids and induction of labour 24 hours later.
E) Intramuscular corticosteroids and spontaneous onset of labour

Prenatal Diagnosis
MCQ
1. Regarding prenatal diagnosis,
A) Amniocentesis causes foetal limb deformities
B) Amniocentesis is an indication to give anti d antibody for rh negative mother
C) Biochemical tests are appropriate in multiple gestations
D) Increased nuchal translucency indicates chromosomal abnormality.
E) The accepted time for chorionic villous sampling is 20 weeks
SBR
Rhesus Isoimmunisation
MCQ
1. Indications for anti d prophylaxis in a Rh-negative woman include
A) Amniocentesis
B) Ectopic pregnancy
C) Induction of labour
D) Intrauterine death
E) Therapeutic termination of pregnancy
2. Which of the following sensitizing events causes a significant feto-maternal haemorrhage?
A) Amniocentesis
B) Ectopic pregnancy
C) ECV
D) First trimester miscarriages
E) Intrauterine death

139
SBR
3. A 22yr mother of 1 child is admitted for the management of rh isoimmunization. She is at 28 weeks of POA
and EFW is 1.2kg. The amniotic fluid volume is normal. She had rising antibody the for last 4 weeks. What is
the most appropriate step to assess the foetal condition?
A) Amniocentesis
B) CTG
C) Foetal blood sampling
D) MCA doppler
E) Umbilical artery doppler

Caesarean Section
MCQ
SBR
1. A patient presented with fever and severe abdominal pain one week after caesarean section. Physical
examination revealed tender abdomen and an offensive vaginal discharge. Initial management should include
A) USS abdomen and pelvis
B) Exploratory laparotomy
C) IV antibiotics
D) Taking a high vaginal swab
E) Withholding breast feeding
2. A 28-year-old mother of 2 children is in her 3 pregnancy. Her 1 delivery was an emergency LSCS before 4 years
and then she had a vaginal birth after caesarean section 2 years ago. Her current pregnancy is uncomplicated
and her BMI is 28. What is the best indicator for a successful vaginal delivery in this pregnancy?
A) BMI <30kgm
B) Emergency caesarean section
C) Previous vaginal delivery
D) Inter pregnancy interval of 2 years
E) Spontaneous onset of labour

Preterm Labour
MCQ
1. Which of the following are indicated in preterm labour with intact membranes at 32 weeks POA?
A) Dexamethasone
B) Nifedipine
C) Erythromycin
D) Progesterone
E) MgSO4
2. Maternal risk factors for pre-term birth
A) Exercise
B) Obesity
C) Pregnancy following assisted reproduction
D) Small inter pregnancy interval
E) Teenage pregnancy

140
SBR
3. A 36-year-old primigravida presents at 30 weeks of period of gestation with prelabour rupture of membranes,
confirmed by speculum examination. Her vital signs are normal. There are no uterine contractions and the
cardiotocograph is normal. What is the most important next step in the management of this woman?
A) Administer co-amoxiclav
B) Administer steroids
C) Caesarean section
D) Induction of labour
E) Ultrasound scan to assess foetal growth
4. A primigravida presented at 32 weeks of gestation with a history of leakage of amniotic fluid of 3 hours
duration and abdominal pain. Speculum examination confirms rupture of membranes. What is the most
appropriate management of this patient?
A) Give tocolytics and dexamethasone
B) Maintain a temperature chart
C) Manage conservatively with providing sterile pads
D) Perform full blood count
E) Treat with oral antibiotics
5. A 30-year-old primigravida presenting in her 32 weeks of POA with abdominal pain for 6 hours she has uterine
contractions 3/10 minutes, cervix - 3cm, bulging membrane was present. CTG was normal. What is the most
appropriate next step in the management?
A) Wait for spontaneous delivery
B) MgSO4 infusion
C) Emergency LSCS
D) Give IM dexamethasone
E) Do arm and syntocinon infusion and let the labour to progress
6. A 28 year old preemie at 34 weeks of POA. Presented with a water discharge of 2 hours duration. on abdominal
examination, foetus is in transverse lie. What is the most appropriate next step in management?
A) Administer dexamethasone
B) Administer prophylactic antibiotics
C) Perform a CTG
D) Perform a speculum examination
E) Perform an USS
7. 24 years old primi mother at 33 weeks of POG presented with premature rupture of membrane. No PV
bleeding or abdominal pain. What is the most suitable next investigation?
A) CTG
B) CRP
C) FBC
D) Measure cervical length
E) USS and measure the growth of the foetus

Prescribing in Pregnancy
MCQ
1. Complication of warfarin therapy during pregnancy include
A) Foetal intracranial haemorrhage
B) Hypoplasia of foetal nasal bridge
C) Maternal osteoporosis
D) Maternal thrombocytopaenia
141
E) Postpartum haemorrhage
2. Which of the following drugs are unsafe during pregnancy?
A) Digoxin
B) Enalapril
C) Furosemide
D) Methotrexate
E) Sildenafil
3. Absolute indication for low dose aspirin during pregnancy,
A) APLS
B) Chronic diabetes mellitus
C) Past history of death-in-utero
D) Past history of pregnancy induced hypertension at 37 weeks of gestation
E) Multiple pregnancy 4
SBR
4. A woman using long term oral medication for epilepsy, booked for antenatal care at six weeks gestation. Her
last epileptic seizure was three months ago. Which one of the following drugs will require urgent substitution
due to high teratogenicity?
A) Carbamazepine
B) Lamotrigine
C) Phenobarbitone
D) Phenytoin sodium
E) Sodium valproate
5. A two-year-old child is brought to the paediatric clinic regularly to manage congenital abnormalities of the
limbs. The child's mother gave a history of taking treatment for acne when she was pregnant with this child.
What is the likely drug that would have caused the child's condition?
A) Benzoyl peroxide
B) Clindamycin
C) Doxycycline
D) Erythromycin
E) Tretinoin
6. 2 year old child came to the paediatric clinic for abnormal limb development. His mother has taken medicine
for acne during pregnancy. Commonest drug which causes this problem is,
A) Benzoyl peroxide
B) Doxycycline
C) Erythromycin
D) Clindamycin
E) Retinoids

Antenatal Care
MCQ
1. Regarding antenatal care
A) A positive VDRL test require treatment with penicillin
B) Dating scan is the best performed between 7-9 weeks of gestation
C) Nuchal translucency is best measured between 14-16 weeks of gestation
D) Screening for HIV is recommended for all pregnant women in Sri Lanka
E) Triple test includes assessment of serum AFP, HCG and oestriol. 2

142
2. Diagnosis tests for trisomy 21 include
A) Amniocentesis
B) Chorionic villus sampling
C) Detection of absent nasal bone on ultrasound scan
D) Nuchal translucency by ultrasound scan
E) Serum HCG and alpha-fetoprotein measurement
3. Regarding antenatal surveillance
A) Foetal movement is an integral part of biophysical profile
B) Every pregnancy should be offered with a growth scan
C) CTG can diagnose FGA
D) MCA doppler studies are useful in suspected FGR
E) She can be used as a screening method for small for gestational age babies
SBR

143
Gynaecology – Reproductive Health
Uterovaginal Prolapse and Urinary Incontinence
MCQ
1. Overactive bladder
A) Causes urgency
B) Overflow incontinence
C) Trans obturator tape (tot) is a treatment option
D) Oxybutynin can be given
E) Botulism toxin injected to the detrusor is an option
2. Complication of mid urethral sling procedure include
A) Acute retention of urine
B) De novo bladder over activity
C) Injury to the ureter
D) Tape erosion
E) Thigh pain
3. Urodynamic studies are indicated in woman with urinary incontinence.
A) Following surgery for prolapse
B) Prior to start drug treatment
C) Prior to surgery when complaining of stress incontinence only
D) Prior to surgery if woman complains mixed on
E) Failed medical management
4. Risk factors for developing stress urinary incontinence,
A) Increased use of caffeine
B) Menopause
C) Multiparity
D) Obesity
E) Past history of hysterectomy
SBR
5. An otherwise healthy 70-year-old woman with diabetes mellitus presented with a third degree uterovaginal
prolapse and a large perineal deficit. The best management option is to
A) Insert a vaginal ring pessary
B) Perform a Manchester repair
C) Perform a posterior repair and insert a vaginal ring pessary
D) Perform a vaginal hysterectomy and repair
E) Perform an anterior and posterior repair
6. A 66 year old past menopausal woman presented with urinary incontinence with coughing and sneezing, she
has 3 children from vaginal deliveries. Which of the following is the best management option for this patient?
Management option for this patient?
A) TVT
B) Keily's procedure
C) Pelvic floor exercises
D) Oxybutynin
E) Burch colpo suspension

144
7. A 53-year-old woman presents with a lump at vulva of six months' duration. On examination, she is found to
have a minimal cystocele. The lump appears only while engaging in heavy lifting or particularly strenuous
activities. What is the most appropriate management option for this woman?
A) Burch colpo suspension
B) Local oestrogen therapy
C) Reassure and review in six months
D) Ring pessary
E) Vaginal hysterectomy
8. 64-year-old woman presented with urinary incontinence, increased frequency, nocturia, and urgency for 6
months. What is the most appropriate initial investigation?
A) Cystoscopy
B) FBS
C) Urine analysis
D) Bladder diary
E) Urodynamic studies

Pelvic Infections
MCQ
1. Which of the following causes multiple painful genital ulcers?
A) Bacterial vaginosis
B) Chancroid
C) Genital herpes
D) Lymphogranuloma venereum
E) Primary syphilis
2. Microscopic examination is useful in the diagnosis of
A) Bacterial vaginosis
B) Candidiasis
C) Chlamydia
D) Gonorrhoeae
E) Trichomonas
3. A 27-year-old recently married female presents with dysuria, poor stream, vulval pain and vulval ulcers for 2
days duration. What is the most likely diagnosis?
A) Genital herpes
B) Gonorrhoea
C) Primary syphilis
D) UTI
E) Vulval candidiasis
4. Which of the following investigations are useful in diagnosing a suspected pelvic inflammatory disease?
A) CRP
B) High vaginal swab
C) Urine microscopy
D) Serum beta HCG
E) Transvaginal ultrasound scan
SBR
5. An 18-year-old sexually active woman presented to the casualty ward with lower abdominal pain, fever,
nausea, vomiting and malodourous vaginal discharge of three days duration ultrasound scan was
unremarkable and urine HCG negative. What is the most appropriate management of this patient?
145
A) Arrange an urgent diagnostic laparoscopy
B) Arrange an urgent surgical referral
C) Choate treatment with doxycycline, metronidazole and a cephalosporin
D) Screen her partner for sexually transmitted infections
E) Treat symptomatically until the result of the high vaginal swab is available
6. A woman who had an intrauterine contraceptive de inserted one month ago at her local MOH clinic attends +
gynaecology clinic with lower abdominal pain and vaginal discharge. On examination, she is afebrile and there
is a purulent discharge with a tender uterus. What is the most appropriate next step in her management?
A) Perform full blood count and CRP
B) Remove the intrauterine contraceptive device
C) Take a swab from the discharge and start oral antibiotics
D) Treat with analgesics
E) Treat with broad spectrum intravenous antibiotics
7. 28-year-old lady with 2 children presents with a malodourous vaginal discharge that increases in intensity
following sexual intercourse. What could be the diagnosis?
A) Bacterial vaginosis
B) Trichomoniasis
C) Chlamydial infection
D) Gonorrhoea
E) Candidiasis

Primary and Secondary Amenorrhoea


MCQ
1. Which of the following are causes of primary amenorrhoea with normal development of secondary sexual
characteristics?
A) Androgen insensitivity syndrome
B) Gonadal agenesis
C) Imperforate hymen
D) Kallman's syndrome
E) Mullerian agenesis
2. Features of androgen insensitivity syndrome,
A) Absent breast
B) Scanty pubic and axillary hair
C) Short stature
D) Structures originating from mullerian duct system are absent
E) Undescended testis
SBR
3. A 15-year-old girl presented with primary amenorrhea. She had normal breast and pubic hair development.
For the past two years she complained of monthly cyclical lower abdominal pain. What is the most likely
diagnosis?
A) Constitutional delay
B) Imperforate hymen
C) Mullerian agenesis
D) Polycystic ovarian syndrome
E) Testicular feminization syndrome
4. A 14 year old schoolgirl presents with delay in menstruation. She has a recent history of cyclical lower
abdominal pain. Breast development is normal. What is the likely cause?

146
A) Ovarian resistance syndrome
B) Transverse vaginal septum
C) Testicular feminization
D) Constitutional delay
E) Turner syndrome
5. A 42-year-old woman presents to the gynaecology clinic with a period of amenorrhoea of six months
complaining of galactorrhoea, headache and aches and pains of the body. Her urine HCG is negative. Her
serum prolactin level was 458 ng/ml. What is the next most appropriate investigation you will carry out?
A) MRI scan of brain
B) Perimetry test
C) Serum FSH and LH levels
D) Transvaginal scan
E) X-ray skull lateral view
6. A 20-year-old girl presents with primary amenorrhoea. Her height is normal. Her breast development and
public hair growth appear normal. She is developing crampy abdominal pain for a few days every month. What
is the next most appropriate investigation?
A) Full blood count
B) Laparoscopy
C) Serum FSH and LH levels
D) Ultrasound scan of abdomen and pelvis
E) X ray of left wrist
7. A 20-year-old lady presents with a history of amenorrhoea, galactorrhoea and frequent headaches, her
pregnancy test was negative. What is the next step in management?
A) MRI of pituitary
B) Check visual perimetry
C) Check serum prolactin level
D) Treat with a dopamine receptor agonist
E) Check FSH/LH levels
8. 20 years old girl presented with primary amenorrhea. Height and breast development are normal with absent
axillary and pubic hair. She is found to have short and blind ended vagina. Uss abdomen reveals absent uterus.
Most useful investigation to arrive at a diagnosis is.
A) Buccal smear for bar bodies
B) Diagnostic laparoscopy
C) Karyotyping
D) Serum FSH/LH level
E) Urinary 21 hydroxyprogesterone level assay

Abnormal Uterine Bleeding


MCQ
1. Following are elevated in pcos,
A) Plasma insulin
B) Cortisol
C) LH: FSH
D) TSH
E) Sex hormone binding globulin
2. Regarding abnormal uterine bleeding,
A) Adenomyosis is a cause
B) FSH, LH and TSH levels are assessed routinely
147
C) Intermenstrual bleeding is associated with endometrial polyps
D) Lng-ius is not recommended as a long-term treatment option
E) Progesterone is not indicated in acute heavy menstrual bleeding
SBR
3. 17-year-old girl presented with menorrhagia and dysmenorrhoea during the first two days of menstruation.
She has regular 28-day cycles, with a heavy flow for 5 days. Her BMI was 21kg/m2. Haemoglobin level was 9
g/dl. The ultrasound scan was normal. The most appropriate initial management is to administer
A) Combined oral contraceptive pills for 3 cycles
B) Depot medroxyprogesterone acetate injections 150 mg once a month for 3 months
C) Mefenamic acid and tranexamic acid during menstruation
D) Norethisterone 5 mg twice daily for 3 cycles of 21 days
E) Tranexamic acid during menstruation
4. 24-year-old obese unmarried lady presented to the gynaecology clinic with a history of irregular menstrual
bleeding and hirsutism. Her BMI is 30kg/m. She is expecting to get married within next 3-month period and
worries about her appearance. What is the most appropriate management?
A) Start on cyproterone acetate
B) Start on metformin
C) Ovarian drilling
D) Exercise for weight reduction
E) COCP
5. 47 a 20 year old unmarried girl presents with 3 months history of heavy menstrual regular bleeding
(menorrhagia). What is the best treatment for this girl?
A) Tranexamic acid with mefenamic acid during periods
B) COCP
C) Norethisterune
D) Depo Provera
E) Levonorgestrel IUS
6. A 38 year old woman presented with 3 months history of menorrhagia despite of medical treatment. Further
evaluation found out 3x3 cm size submucosal fibroid. She has completed her family. What is the most
appropriate management?
A) TAH
B) Laparoscopic removal of fibroid
C) Hysteroscopic removal of fibroid
D) LHRH agonist for 6 months
E) Tranexamic acid 5mg tds for 6 months
7. A 48-year-old woman presents with heavy regular vaginal bleeding of three months' duration. Transvaginal
scan shows normal uterus and ovaries what is the most appropriate management option?
A) Ask her to return if the symptoms persist for more than six months
B) Perform endometrial biopsy
C) Treat with mefenamic acid and tranexamic acid
D) Treat with norethisterone
E) Treat with oral iron
8. 46-year-old mother of four children presented with dysmenorrhoea and heavy menstrual bleeding for one-
year duration, bimanual examination reveals a tender uterus of 10 weeks POA. What is the most likely
diagnosis?
A) Adenomyosis
B) Uterine polyp

148
C) Pelvic endometriosis
D) Pelvic inflammatory disease
E) Submucosal fibroid
9. 25-year-old unmarried women presented to gynaecology clinic with amenorrhea for 6 months and uss
showing polycystic ovarian morphology. Blood investigations and hormone profile are normal. What is the
most rational management option?
A) Ovarian drilling
B) Induce a withdrawal bleeding
C) Treat with clomiphene citrate
D) Treat with metformin
E) Weight reduction
10. 17-year-old schoolgirl presented with 6-month history of irregular heavy menstruation. What is the best
treatment option for her?
A) Combined oral contraceptive pills
B) Cyclical norethinesterone
C) NSAIDs
D) Oral iron
E) Tranexamic acid
11. 40-year-old otherwise healthy lady presented with heavy menstrual bleeding. On investigation she was found
to have endometrial hyperplasia without atypia. BMI is 32kg/m2. Most appropriate management options.
A) Combined oral contraceptive pills
B) Levonorgestrel IUS
C) Total abdominal hysterectomy and BSO
D) Tranexamic and mefenamic acid therapy
E) Hysteroscopic endometrial resection

Post-Operative Complications
MCQ
1. Operative infection risk is increased by
A) Early mobilization
B) Short surgery
C) Shaving the area
D) Catheterization
E) Obesity
2. 50-year-old mother of 2 admitted to ward 6 hours ago following tah + bso. During last two hours there's no
urine output.hr 80bpm, bp 120/80mmhg. What will you do next?
A) Iv furosemide
B) Blood transfusion
C) Check the catheter
D) Inform theatre
E) Iv fluid bolus
3. Risk factors for developing post-operative thrombosis,
A) Heavy blood loss during surgery
B) Prolonged surgery
C) Malignancy
D) Obesity
E) Use of li carbonate

149
SBR
4. A 40-year-old woman had a total abdominal hysterectomy for menorrhagia. On the second post-operative day
she developed tense abdominal distension and profuse vomiting. Her temperature was normal, and an erect
abdominal x-ray showed multiple fluid levels what is the most appropriate management option?
A) Bisacodyl suppositories
B) Exploratory laparotomy
C) Insertion of a nasogastric tube
D) Intravenous antibiotics
E) Intravenous antiemetic
5. A patient who underwent a total abdominal hysterectomy complained of severe lower abdominal pain and
difficulty in breathing two hours after surgery. Her pulse rate was 110/minute and the blood pressure 85/50
mmHg. Her urine output was 5ml/hour. The oxygen saturation was 98% the most likely cause for her clinical
picture is
A) Intra peritoneal bleeding
B) Blocked urinary catheter
C) Myocardial infarction
D) Pulmonary embolism
E) Ureteric injury
6. A 23 year old woman underwent ERPC. Next day, she complains of abdominal pain, per vaginal bleeding and
fever on examination. She has diffused abdominal tenderness and bowel sounds are sluggish. USS revealed
free fluid in pouch of Douglas. What is the most appropriate management?
A) Admit to surgical ICU
B) Commence iv antibiotics
C) Commence vital monitoring
D) Prepare for laparotomy
E) Prepare for re-examination of the uterus
7. A 44-year-old woman who underwent a total abdominal hysterectomy and bilateral Salpingo-oophorectomy
for severe endometriosis one week ago presents with fever and watery vaginal discharge. What is the most
appropriate next step you will carry out?
A) Ct scan of the abdomen
B) Cystoscopy
C) Intravenous Urogram
D) Speculum examination
E) Ultrasound scan of abdomen
8. 54-year-old lady with uterine fibroid underwent a transvaginal hysterectomy. On post op day 3 she was
discharged without any complications. Again, she admitted with continues urine flow from vagina. Vesico-
vaginal fistula was suspected. Best investigation to support the diagnosis is?
A) Iv Urogram
B) Cystoscopy
C) Diagnostic laparoscopy
D) Vaginal swab test
E) USS abdomen

Contraception and subfertility


MCQ
1. Clomiphene citrate,
A) Acts on hypothalamus

150
B) Causes hot flushes
C) Causes ovarian hyperstimulation
D) Contraindicated in PCOS
E) Effective in resistant ovarian syndrome
2. Regarding Cu T380A
A) Act by inhibiting implantation
B) Increase cervical secretion
C) Increase menstrual flow
D) Is effective for 10 years
E) Can be inserted in immediate postpartum period
3. Regarding male factor infertility
A) Hyperprolactinaemia causes abdominal seminal fluid parameters
B) Obstruction of the vas deferens in anejaculation
C) Pre-pubertal mumps impairs spermatogenesis in later life
D) Retrograde ejaculation is a cause
E) Unprovoked early morning erection in a man with
4. Subdermal contraceptive implant
A) Causes irregular vaginal bleeding
B) Contains oestrogen and progestogen
C) Delays fertility after removal
D) Increase the risk of ectopic pregnancy
E) Is contraindicated in women with hypertension
5. Which of the following are normal parameters of a seminal fluid analysis?
A) Normal morphology
B) >39 million sperm per ejaculate
C) Progressive motility 2 o
D) Sperm concentration > militant
E) Volume more than 15ml
6. Regarding levonegestrol-IUS,
A) A cause for unscheduled vaginal bleeding
B) A case for benign ovarian cyst
C) Can be used treat endometriosis
D) Contraindicated immediately after a miscarriage
E) Given with oestrogen therapy for endometrial protection
7. Regarding anovulatory infertility
A) Hypogonadotropic anovulation can be treated with clomiphene citrate
B) Hyper gonadotrophic hypogonadism is managed with ivf from a donated ovum
C) Progesterone withdrawal test does not induce bleeding in hyper gonadotrophic anovulation
D) GNRH pulses can be used in treatment of patients with hypergonadotropic anovulation
E) TSH and prolactin levels are routinely checked
8. Contraceptive methods that can be used in a patient with venous thromboembolism,
A) COCP
B) Condoms
C) Copper IUCD
D) Injectable progesterone preparations
E) LNG-IUS

151
SBR
9. A couple married for 18 months presented with a history of primary subfertility. The wife had regular 30-day
menstrual cycles and a history of surgery for ruptured appendix five years ago. The husband's seminal fluid
analysis was normal. Their general practitioner had treated the woman with four cycle of clomiphene citrate
without any success. What is the most appropriate next investigation?
A) Day 21 progesterone level
B) Hysterosalpingography (HSG)
C) Laparoscopic examination
D) Repeat the seminal fluid analysis
E) Serum testosterone level of the male
10. A 22-year-old sub fertile woman complained of amenorrhoea and frequent headaches of recent onset. On
examination she was found to have a discharge from both nipples. Subsequent investigations showed elevated
serum prolactin levels. What is the most appropriate investigation?
A) Ct scan of the head
B) Cytological examination of the nipple discharge
C) Thyroid function test
D) MRI of the head
E) X ray of the pituitary fossa
11. Which mechanism is least key to contribute to the contraceptive effect of copper containing intra uterine
device?
A) Sperm effect of copper ions
B) Change of tubal motility
C) Inflammation of the endometrium
D) Inhibition of ovulation
E) Interference with implantation
12. A 28-year-old woman who had delivered her second child two months ago attended the family planning clinic.
She had gestational diabetes mellitus during pregnancy. Her body mass index is 28 kgm2 and the fasting blood
sugar 120 ml/d. The most suitable contraceptive method for her is
A) Combined oral contraceptive pills
B) Depot medroxyprogesterone acetate
C) Intrauterine contraceptive device
D) Ligation and resection of the fallopian tubes
E) Sub-dermal progesterone implant
13. A 34 year old woman with secondary amenorrhoea for 2 years presents to infertility clinic. Her FSH levels were
checked two months apart. Both were above 40 iu/l. TSH, prolactin, uterine morphology and husband's SFA
were normal what is the best treatment option for sub fertility of this couple?
A) Clomiphene citrate to induce ovulation and intra uterine insemination
B) FSH and LH to induce ovulation and in vitro fertilization
C) Intra uterine insemination of surrogate woman with husband's sperms
D) Donor oocyte and in vitro fertilization
E) GNRH to induce ovulation and in vitro fertilization
14. A 30 year old mother of 3 children, with the last child being 6 weeks old, has a BMI of 30 kg/m she has no
menorrhagia or dysmenorrhea. What is the best contraceptive method for her?
A) DMPA
B) Copper IUD
C) LNG-IUS
D) Ligation & resection of tubes

152
E) Progesterone only pills
15. A39 year old lady comes for contraceptive advice, she is having regular heavy periods & no other
complications. Her BMI is 30 kg/m, pelvic USS is normal best contraceptive method for her is?
A) Combined oral contraceptive pills
B) Ligation & resection of tubes
C) LNG-IUS
D) Depo Provera
E) Condoms
16. A 35-year-old infertile woman with anovulation is found to have a serum prolactin level of 1200 mlu/i (normal
range 35-350 mlu/i). Thyroid stimulating hormone (TSH) level is has bilateral patent, tubes normal. She on a
hysterosalpingogram and the seminal fluid analysis is normal. What is the next most appropriate step in the
management?
A) MRI skull for a pituitary tumour
B) Ovulation induction with clomiphene citrate
C) Ovulation induction with gonadotropin
D) Treatment with cabergoline
E) X-ray skull
17. A 28-year-old woman married for four years presents with a history of inability to conceive despite having
regular unprotected sexual intercourse. Her husband's seminal fluid analysis reveals azoospermia. What is the
most appropriate next step in the management of this couple?
A) Advice on adoption
B) Advice on donor insemination
C) Measure FSH and LH levels of the husband
D) Perform testicular biopsy
E) Treat husband with oral testosterone
18. A couple presents with a history of sub fertility. Male is a 40-year-old. Sfa analysis was done and the result
came as asthenospermia. Female partner had no complaints and her investigations were normal. What is the
next best step?
A) Measure FSH, LH and serum testosterone
B) Do a vasogram
C) IUI with donor sperm
D) Repeat the SFA
E) Aspirate from the testis
19. A couple presented with primary subfertility for 2 years. Female has pain free periods with 40-45 days cycle
duration. Her BMI is 24kgm. Husband's SFA showed sperm count of 12*106/mm3, progressive motility>50%
and normal morphology>30%. What is the most appropriate next step?
A) Hysterosalpingogram
B) Laparoscopy and dye test
C) Ovulation induction with clomiphene citrate
D) Ovulation induction and iui
E) Male partner should be treated with coenzyme q
20. A 25-year-old female complains of primary subfertility and history of oligomenorrhea. On examination she
was found to have acne and her BMI was 24kgm ultrasound abdomen and pelvis revealed a polycystic
morphology of the ovaries. What is the most appropriate step in the management to gain her fertility?
A) Perform IUI
B) Ovulation induction with clomiphene citrate
C) Laparoscopic ovarian drilling

153
D) Treatment with metformin
E) Weight reduction
21. 28-year-old mother of 2 children who delivered her 2" child 2 months ago presented to the family planning
clinic she had GDM during the pregnancy and her BMI is 28 kg/m2. Her fasting blood sugar value is 120 mg/ dl
what is the most appropriate contraceptive method for her?
A) Combined oral contraceptive pills
B) Depot medroxyprogesterone acetate
C) Intrauterine contraceptive device
D) Ligation and resection of fallopian tube
E) Subdermal implant
22. A lady with 3 children presented with missing thread of IUCD which has been put 4 years back. On uss the
thread is in the uterine cavity, what's the best management option?
A) Reassure her and remove IUCD when needed.
B) Remove IUCD under GA and insert a contraceptive implant
C) Remove IUCD under GA and insert a new one
D) Remove under GA and offer sterilization
E) Remove IUD under GA and insert contraceptive device
23. A 30-year-old lady presented with inability to conceive for 2 years. She has painless menstrual cycles occurring
at 25-30-day regular intervals. Seminal fluid analysis of the partner is normal. What is the next management
option?
A) Measure d21 progesterone levels
B) HSG
C) Lap and dye test
D) Ovulation induction and IUI
E) Treat the male partner with co enzyme q
24. 20-year-old diagnosed patient with Eisenmenger syndrome who is newly married requests for a family
planning method. What is the most appropriate contraceptive method for her?
A) Copper IUD
B) OCP
C) Injectable progesterone
D) Condoms
E) Sterilization

First Trimester Bleeding


MCQ
1. Investigations carried out before suction evacuation for complete mole,
A) Ca 125
B) CXR
C) FBC
D) Serum beta HCG
E) Thyroid function tests
2. Complete hydatidiform mole
A) Carries a lower tendency for malignant transformation than partial mole
B) TSH level is elevated
C) Contain only paternal chromosomes
D) Can be treated with misoprostol
E) Is triploid

154
SBR
3. A 24-year-old woman was admitted with lower abdominal pain and mild vaginal bleeding of two hours
duration at seven weeks of amenorrhoea. She was pale with cold and clammy extremities, pulse rate of 120
beats per minute, blood pressure of 70/50 mmHg and lower abdomen is tender. What is the most appropriate
first step of the management?
A) Catheter the bladder
B) Insert a wide bore cannula and cross match blood
C) Perform a trans vaginal ultrasound scan
D) Perform a urine pregnancy test
E) Send blood for full blood count
4. A 41 year old multipara presents with per vaginal bleeding at 12 weeks of POA. On abdominal examination,
uterus is 14/52; USS shows snow-storm appearance without foetal parts. What is the best management?
A) Hysterectomy and evacuation
B) Insert misoprostol
C) Induction with oxytocin
D) Suction evacuation
E) Total abdominal hysterectomy
5. A 34-year-old woman has undergone suction evacuation for complete hydatidiform mole one week ago. What
is the best contraceptive option for her?
A) Calendar method
B) Combined oral contraceptive pill
C) Depot medroxyprogesterone acetate injection
D) Intrauterine contraceptive device
E) Male condoms
6. A 34-year-old woman is admitted with fever, vaginal bleeding and vaginal discharge. She has taken misoprostol
with the intension of termination the pregnancy one week ago. She is haemodynamically stable, and the
ultrasound scan shows retained products. What is the most appropriate next step in the management of this
woman?
A) Administration misoprostol vaginally
B) Blood transfusion
C) Evacuation of retained products of conception
D) Start intravenous antibiotics
E) Take high vaginal and cervical and cervical swabs for culture and antibiotic sensitivity tests.
7. A 25-year-old unmarried women presented with a POA of 6 weeks and the transvaginal scan confirms
intrauterine pregnancy. She has sex regularly with one partner and she wants to terminate her pregnancy due
to social stigmata. What is the best management option?
A) Advise her to continue pregnancy
B) Termination of pregnancy
C) Inform the parents
D) Obtain a psychiatry referral
E) Refer to social services
8. 20-year-old Primi at 20 weeks of POG presented with vaginal bleeding, abdominal pain and faintish attack. On
examination OS is open and the products of conception were seen. USS scan showed a 25mm gestational sac
with no foetal heartbeat. Most appropriate course of management?
A) Measure serum beta HCG
B) IM opioids
C) Oxytocin infusion

155
D) Do a speculum examination and remove the products with a sponge forceps.
E) ERCP at operating theatre

Cervical Intraepithelial Neoplasia and Cervical Carcinoma


MCQ
1. Carcinoma of cervix,
A) Caused by HSV
B) Is familial
C) Prevented by vaccination
D) Glomerular type has good prognosis than squamous type
E) Smoking is a risk factor
SBR
2. A 42-year-old woman was referred to the gynaecology clinic with abnormal cervical smear test reported as
high grade squamous intraepithelial neoplasia. A colposcopic examination was performed and the
squamocolumnar junction was visualized completely. What is the most appropriate next step in the
management of this patient?
A) Excision of the transformation zone
B) Local irradiation of the cervix
C) Repeating the smear test in six months
D) Total abdominal hysterectomy
E) Treatment with antiviral medicine
3. A 55 year old woman presents with postmenopausal bleeding. On speculum examination, there is an ulcer in
the cervix. Next step in the management?
A) Pap smear
B) Cervical biopsy
C) Colposcopy and acetic acid staining
D) TAH
E) Loop excision
4. A 45-year-old woman who complains of post-coital bleeding is found to have a cervical growth. What is the
next most appropriate step in the management of this woman?
A) Cervical biopsy
B) Cervical smear
C) Colposcopy
D) MRI scan
E) Radical hysterectomy and pelvic node dissection
5. 42-year-old woman was referred to the gynaecology clinic with abnormal cervical smear with high grade
squamous intraepithelial neoplasia. On colposcopy acetowhite areas are next step in the management?
A) Large loop excision of the transformation zone
B) Cold knife cone biopsy
C) Diathermy cauterization
D) punch biopsy
E) Repeat the colposcopy in 6 months

Benign and Malignant Ovarian Tumours


MCQ
1. Which of the following are benign epithelial ovarian tumours?
A) Androblastoma
156
B) Brenner tumour
C) Granulosa cell tumour
D) Mature teratoma
E) Serous cystadenoma
SBR
2. A 38-year-old woman complained of being unable to pass urine for the past 16 hours. She was in severe pain.
Examination revealed a large pelvic mass. She had undergone a myomectomy 10 years ago for a large uterine
fibroid. What is the initial step in the management?
A) Administer 75mg of intramuscular pethidine
B) Insert an indwelling catheter
C) Prepare the patient for an emergency laparotomy
D) Send urine for culture and antibiotic sensitivity test
E) Ultrasound scan of the abdomen and pelvis
3. A 35-year-old woman was found to have a unilocular cyst measuring 6x6 cm in the right ovary in an ultrasound
scan. The left ovary and the uterus were normal. There was no free fluid and she is asymptomatic. The best
management option for this patient is to
A) Aspirates the cyst
B) Follow up with a repeat ultrasound scan
C) Perform a ca 125 level
D) Perform laparoscopic cystectomy
E) Reassure with no further follow up
4. A 56 year old female is presented with irregular per vaginal bleeding. USS showed that et is 12 mm and 5x5
cm cyst in the right ovary endometrial biopsy confirmed endometrial hyperplasia. What is the most likely
condition she has in her right ovary?
A) Brenner tumour
B) Granulosa cell tumour
C) Mature cystic teratoma
D) Mucinous cystadenoma
E) Serous cystadenoma
5. A 55-year-old unmarried menopausal woman presents to gynaecological outpatient department with an
incidental finding of right ovarian cysts measuring 5x5 cm. What is the most appropriate next step in her
management?
A) MRI scan
B) Repeat ultrasound scan in six months
C) Ct scan
D) Serum ca 125 level
E) Total abdominal hysterectomy and Salpingo-oophorectomy.
6. A 31-year-old nulliparous woman presents with a history of inability to conceive despite having regular
unprotected sexual intercourse for three years. She also complains of dysmenorrhea. Clinical examination
reveals a left adnexal mass and a fixed retroverted normal sized uterus. Ultrasound scan shows a left ovarian
cyst of 6x6 cm size. What is the most appropriate management of this patient?
A) Combined oral contraceptive pills
B) Depot medroxyprogesterone acetate injections monthly
C) Laparoscopic ovarian cystectomy
D) Ovulation induction with clomiphene citrate
E) Ultrasound guided aspiration of the cyst

157
7. 38-year-old lady was diagnosed with a unilocular cyst in the left ovary. On USS, it is 6x5 cm in size and no solid
areas. Uterus and right ovary are normal, no free fluid. Ca 125 is 13iu. What is the most appropriate next step?
A) Follow up with USS in 3 months
B) CECT
C) Laparoscopic cystectomy
D) Laparotomy and cystectomy
E) USS guided aspiration of the cyst
8. 53-year-old postmenopausal women with ca 125 level of 13 IU/l USS suggests 4cm sized unilocular ovarian
cyst in left ovary. She is asymptomatic, cyst persisted in the same size for 1-year duration, what would be the
best management option for her?
A) TAH and BS0
B) Reassure and discharge
C) Repeat the investigations in 6 months’ time
D) Laparoscopic cystectomy
E) USS guided aspiration of the cyst
9. A 30-year-old lady has a family history of ovarian ca. What is the most appropriate test to predict the
probability of developing ovarian carcinoma?
A) Annual USS abdomen
B) Regular ca125
C) Assess BRCA 1 and BRCA 2 mutation
D) Diagnostic laparoscopy
E) Annual doppler study of ovaries

Endometrial Carcinoma and Fibroids


MCQ
1. Leiomyoma of the uterus
A) Carries 10% risk of malignancy transformation
B) Causes polycythaemia
C) Causes urine retention
D) Treated with uterine artery embolism
E) Red degeneration after menopause
2. Treatment options for an intramural fibroid of 5cm diameter in a female with a history of subfertility.
A) Myomectomy
B) GnRH agonists
C) Levonorgestrel-iud
D) Transcervical resection
E) Uterine artery embolization
3. Regarding endometrial malignancy
A) Combined continuous hormone replacement therapy is a risk factor
B) Endometrioid adenocarcinoma is the commonest type
C) It is familial in the majority of cases
D) Serous type has a better prognosis than endometrioid type
E) Tamoxifen reduces the risk
4. Regarding endometrial carcinoma,
A) It is associated with granulosa tumour of ovary
B) Confirmed when endometrial thickness more than 5mm
C) Staged using MRI
158
D) Increased incidence in PCOS
E) Increased risk by using OCP
5. Risk factors for developing endometrial carcinoma,
A) Early menarche
B) Early menopause
C) Hypertension
D) Long term COCP use
E) Obesity
SBR
6. A 56-year-old menopausal woman complains of vaginal bleeding of one-week duration. Speculum examination
is normal. What is the next step in the management of this woman?
A) Administer oestrogen cream
B) Endometrial biopsy
C) Perform ca 125 levels
D) Prescribe norethisterone
E) Ultrasound measurement of endometrial thickness
7. 45-year-old lady presented with an abdominal mass. What is the clinical feature that is more suggestive of a
fibroid rather than an endometrioma?
A) Absence of ascites
B) Smooth surface
C) Firm consistency
D) Limited vertical movement
E) Mobility with the cervix

Menopause and Hormone Replacement Therapy


MCQ
1. Causes for premature ovarian failure
A) Autoimmune disease
B) Chromosomal abnormalities
C) COCP
D) Multiparity
E) Mumps oophoritis
2. Regarding menopause and hormone replacement therapy(hrt)
A) A level of FSH more than 30 IU/l indicates ovarian failure
B) Combination of oestrogen and progestogen contains uterus
C) HRT is contraindicated if there is a past history of breast carcinoma
D) Testosterone is effective in reduced libido
E) The role of Progestogen in HRT is endometrial
3. Contraindications for HRT in a post-menopausal woman includes
A) Diabetes mellitus
B) Coronary artery disease
C) Factor v Leiden deficiency
D) Uncontrolled hypertension
E) Undiagnosed vaginal bleeding
4. In menopause,
A) Complete cessation of ovarian function occurs
159
B) Decline in testosterone of ovarian origin is noted
C) Inhibin b concentration rises
D) Pulsatile secretion of GNRH increases
E) Serum inhibin level decreases
SBR
5. A 49-year-old woman had developed oligomenorrhea and severe hot flushes over the previous 6 months she
had been very well in the past with no family history of metabolic diseases or breast cancer. She leads a healthy
lifestyle. Her BMI is 23 kg/m, BP 120/70 mmhg and her basic biochemistry of metabolic risk is normal. The best
treatment option for her is
A) Antidepressants
B) Combined hormone replacement therapy
C) Evening primrose oil
D) Multivitamin tablets
E) Soy isoflavones

Endometriosis
MCQ
1. Endometriosis
A) Has a genetic predisposition
B) Causes superficial dyspareunia
C) More in multipara than in nullipara
D) Is treated with tibolone
E) Pain outstands the period
2. Regarding endometriosis
A) Drainage an endometrioma compared to cystectomy reduces the risk of recurrence
B) It is treated with the combined oral contraceptive pills
C) Hormone replacement therapy is contraindicated following hysterectomy and bilateral salpingo
oophorectomy
D) It is treated with gonadotrophin releasing hormone
E) Medical treatment improves fertility.
3. Which of the following is true regarding endometriosis?
A) Condition where functional endometrial cells grow outside the endometrial lining
B) Medical treatment of endometriosis entrances fertility
C) Oocyte maturation defects contribute to infertility
D) Ovarian reserve reduces when performing cystectomy for endometriomas
E) Surgical removal of an endometrium increases fertility
SBR
4. 29-year-old married woman with a history of primary subfertility complained of dyspareunia and
dysmenorrhoea of six months duration transvaginal ultrasound scan showed endometrioma in the right ovary
measuring 6x6cm. What is the best modality of treatment for her?
A) Danazol for six-month duration
B) Induction of ovulation with clomiphene citrate
C) Laparoscopic ovarian cystectomy
D) Laparotomy and ovarian cystectomy
E) Treatment with monthly depot medroxy progesterone acetate

160
5. 24-year-old woman presents with secondary amenorrhea. She had regular cycles. She has been trying to
conceive for the past one year. TVS shows an Endometrioma of 5cm. Most reasonable management option
for her is?
A) Laparoscopic aspiration of cyst
B) Laparoscopic cystectomy
C) Back to back COCP
D) TAH and BSO
E) Treatment with danazol
6. A 46-year-old woman presents to gynaecology clinic with severe dysmenorrhea and dyspareunia. Examination
reveals bilateral tender adnexal masses. Her ca 125 is 65 u/ml and ultrasound scan reveal 5 cm sized bilateral
cysts suggestive of endometriosis. Which of the following would you recommend as the most appropriate
treatment option?
A) Laparoscopic bilateral cystectomy
B) Laparoscopic hysterectomy
C) Total salpingo-bilateral and hysterectomy oophorectomy
D) Treat with regular opioids
E) Treat with GNRH analogues

Other
MCQ
SBR
1. A 7 year old girl presented with bleeding per vagina for 1-day duration on examination secondary sexual
characteristics are present. What is the most appropriate investigation?
A) Abdominal USS
B) Serum GNRH
C) Oestradiol
D) Prolactin
E) Pregnancy test
2. A 19-year-old girl presents with clitoromegaly, hirsutism and acne of 3 months duration. What is the most
appropriate investigation?
A) Blood glucose
B) MRI pituitary
C) Serum androgen levels
D) Sex hormone binding globulin levels
E) USS of abdomen

161
Surgery – Reproductive Health
Breast Diseases
MCQ
1. 48-year-old female presented with 2 weeks' history of nipple discharge. Which of the following are suggesting
of malignancy of the breast?
A) Blood stained nipple discharge
B) Discharge from a single duct
C) Painful duct discharge
D) Mass under the nipple areolar region
E) Unilateral nipple discharge
2. Regarding gynaecomastia,
A) Commonly unilateral.
B) Is a feature of testicular tumour
C) Occurs due to glandular proliferation of male breast tissue
D) Pubertal gynecomastia resolves spontaneously.
E) Tamoxifen is a mode of a treatment.
3. T/F regarding benign breast disease
A) Breast cysts are more common in perimenopausal women
B) Cyclical mastalgia is commonly seen in post-menopausal women.
C) Breast cysts are common in perimenopausal age
D) Lactational breast abscesses are usually caused by staphylococcus aureus
E) .
4. Regarding breast carcinoma
A) Ductal carcinoma in-situ is pre-invasive
B) Positive receptor status is a poor prognostic factor
C) Improved survival in breast cancer is mainly due to advancements in surgical techniques
D) Is the commonest cancer in Sri Lanka
E) Prognosis is better in postmenopausal women
SBR
5. 45-year-old woman presents with a breast lump in right upper outer quadrant of right breast for 2 months. It
is 3cm in diameter and hard. Axillary lymph nodes were palpable. USS breast suggestive of malignancy of
breast. What is the next step in management?
A) Core biopsy.
B) Excision biopsy.
C) .
D) .
E) .
6. 60-year-old lady presented to the clinic with an eczematous lesion of the right nipple areolar complex. On
examination, palpable breast lump (3x4cm) in right breast with ipsilateral palpable mobile lymph nodes. What
is the response that gives the most accurate diagnosis?
A) Eczema of right nipple areolar complex with lymph nodes in right axilla
B) Locally invasive right-side breast CA
C) Paget's disease of the right breast
D) Paget's disease with right invasive breast CA
E) Paget's disease T2N1MX
162
7. A 26 years old lady presented with a lump in the right upper outer quadrant of the right breast for 2 weeks'
duration. It was 3cm in size. Mammogram and USS showed features suggestive of malignancy. What is the
next most important step?
A) Excisional biopsy.
B) Incisional biopsy.
C) FNAC.
D) Core biopsy.
E) MRI of the right breast.
8. 30-year-old mother who is lactating presents with right sided breast pain. Has mild fever. On examination of
the right breast there is redness, warmth and tenderness. Which of the following are appropriate initial
management options?
A) Antibiotics
B) Avoid feeding from affected side
C) Incision and drainage
D) Ultrasound scan of the breast.
E) .
9. 40-year-old lady presented with unilateral recent A week) onset blood stained nipple discharge. Examination
skin normal, she has no axillary lymph nodes or palpable breast lumps. No cystic changes. Discharge is from
single duct. What is the diagnosis?
A) Breast carcinoma
B) Duct ectasia
C) Duct papilloma
D) Fibrocystic change
E) Mondor disease.
10. A 56-year-old post-menopausal lady underwent wide local excision and axillary lymph node clearance for
invasive breast carcinoma. The histology report showed that the lymph nodes were free of tumour. The next
treatment option is,
A) Radiotherapy to breast
B) Radiotherapy to axilla
C) Radiotherapy to breast and axilla
D) Chemotherapy
E) Tamoxifen
11. A 35yr old female came with blood stained nipple discharge from the R/ breast. On examination there is no
lump. She has no past hx or family hx of breast carcinoma. On examination there was no palpable lump in the
breast. What is the most likely diagnosis?
A) Duct ectasia
B) Intraductal papilloma
C) Phylloid tumour
D) Atypical duct hyperplasia
E) Fibrocystic disease
12. 18-Year-old patient undergone wide local excision and axillary clearance. Results came as stage 1 Invasive
ductal CA Two core biopsies were taken, but came as normal. What is the next step of management?
A) Tamoxifen
B) Radiotherapy to axilla
C) Radiation to the breast
D) Reassure and follow up the patient
E) 6 cycles of chemotherapy

163
13. 28yr old female R/ side upper R/quadrant breast lump. USS done no significant finding 2 core biopsies also
done & they are normal what is the next step of management?
A) Repeat Core biopsy
B) Breast MRI
C) Excision biopsy
D) PET scan
E) Wide local excision
14. A 50 years old mother of two breast fed children presents with a lump in her right breast There is no personal
or family history of breast cancer. The report on ultrasound and mammogram reads 'suggestive of malignancy.
FNAC report reads c4. What is the next appropriate step in the management?
A) Perform a core biopsy
B) Excisional biopsy
C) Lumpectomy
D) Wide local excision
E) Incisional biopsy

164
Neonatology
New-born Assessment and Prematurity
MCQ
1. Complications of prematurity
A) Bronchopulmonary dysplasia
B) Meconium aspiration syndrome
C) Intraventricular haemorrhage
D) Erb's palsy
E) Paraventricular haematoma
2. Regarding down syndrome
A) Commonest cause is translocations
B) |21,21) carries a 100% recurrence
C) Mosaics have a better 1q
D) Incidence increases with maternal age
E) Commonest genetic cause for low 1q
3. Which of the following are true regarding x linked dominant diseases?
A) When females are affected, they show more signs.
B) Male to male transmission can occur
C) Consanguinity does not affect the outcome
D) Females transmit only to males
E) Affected females will transmit the disease to 25 % of their off springs.
4. WOTF needs further assessment in a 2-day old baby,
A) Pink colour patch in nappy
B) Red colour mark in the back of neck
C) Bluish macule in the buttocks
D) Passing of black colour stools
E) Greenish vomitus
5. In comparison with a term baby, a preterm baby
A) Has a higher risk of developing intracranial haemorrhages
B) Is at a higher risk of getting iron deficiency
C) Is more prone to get hypothermia
D) Needs more ml per kilogram of fluid
E) Is at a higher risk of developing osteopenia
6. Which of the following are indications for admission of a 5-day old neonate?
A) Passage of stools 8 times per day
B) Erythema around umbilicus
C) Green colour vomiting
D) Blue patches over the buttocks since birth
E) Fever for 24 hrs
7. Regarding genetic inheritance of diseases.
A) Human genome contains about 100000 genes
B) Autosomal dominant inherited diseases show vertically transmission
C) Consanguinity increases risk of x linked recessive disease
D) Duchenne muscular dystrophy is an x linked dominant disease
E) Single gene mutations can be assessed by karyotyping
165
8. Characteristics of autosomal dominant disorders,
A) Abnormal genes from both parents are a prerequisite for the disease
B) Probability of affecting the siblings will be determined by the affected 1" child
C) Abnormal gene is dominant
D) Who do not have the disease can't pass the disease?
E) Huntington's disease is an example
SBR
9. 28-week-old premature baby whose birth weight is 1.3kg ventilated due to respiratory distress extubated after
3 days. Next day his bp was 80/40 heart was 130bpm. What is the most appropriate management option?
A) Take blood culture and start iv antibiotics.
B) Intravenous prostaglandin infusion
C) Oral paracetamol
D) Repeat dose of surfactant
E) .

Infections in New-born
MCQ
1. Regarding congenital infection of new-born
A) CMV cause sensorineural hearing loss
B) Rubella causes congenital heart diseases when mother is infected in the 3rd trimester
C) Toxoplasmosis causes hydrocephalus
D) Varicella infections in third trimester causes neurological sequela
E) Syphilis causes bone lesions
SBR
Neonatal Resuscitation
MCQ
SBR
1. You are attending a baby delivered at 38 weeks by LSCS due to foetal distress. Liquor is not meconium stained.
Baby is floppy and having poor respiratory effort and bradycardia. What is the next step in management?
A) Give inflation breaths
B) Give ventilation breaths
C) Give adrenaline
D) Give chest compressions
E) Intubation
2. Baby born by NVD was pale and floppy at birth. No respiratory effort. HR 20/min. Five inflation breath given
with adequate chest expansion. Baby was reassessed and there was only occasional gasping, hr 40/min. What
is the most appropriate next step in management?
A) Give 15 ventilation breaths
B) Iv adrenaline infusion
C) Ventilation with bag and face mask.
D) Give 5 inflation breaths with jaw thrust
E) Intubate and ventilate

166
Seizures in New-Born
MCQ
SBR
1. 3-day old baby boy with 2.2 kg of birth weight presented with poor activity and convulsions. On examination
he was pink, peripheral circulation was good. CBS is 20mg/dl. What is the best management option?
A) 10% glucose bolus + breast feeding
B) 10% glucose bolus + 10% glucose infusion
C) 5% glucose bolus + infusion
D) 50% glucose bolus + 10% glucose infusion
E) 10% glucose infusion + breast feeding

Respiratory Distress in New-Born


MCQ
1. A 3 days old child presents with sob. Saturation of pao2 in room air is 80%, what are the possibilities?
A) Persistent pulmonary hypertension
B) Coarctation of aorta
C) Eisenmenger syndrome
D) Tetralogy of Fallot
E) Transposition of great vessel
2. Congenital diaphragmatic hernia
A) Commonly occurs in the left side
B) Presents with respiratory distress at birth
C) Resuscitated with mask and bag ventilation
D) Needs ng decompression
E) Needs immediate surgical correction
SBR
3. 1-hour old baby is in respiratory distress. On auscultation heart sounds were heard on right side. Abdomen is
scaphoid. Spo2 87%. What is the management?
A) O2 via face mask.
B) Bag and mask ventilation.
C) Intubation and ventilation
D) Needle thoracentesis
E) Reassure
4. Neonate at term becomes apnoeic PR 100 bpm. What is the immediate action?
A) Transfer to ICU
B) Oxygen via head box
C) Give inflation breaths
D) Chest compressions
E) Intubate and ventilate
5. You were called to see a baby who was born 1hour ago by normal vaginal delivery at term. The baby was in
respiratory distress. His respiratory rate was 70 and had nasal flaring grunting and marked intercostal
recessions. Heart sounds were heard on the right side, abdomen was scaphoid, sp02 was 91% on room air.
What is the next best step in the management?
A) Commence broad spectrum iv antibiotics
B) Insert an ng tube

167
C) Give oxygen via nasal prongs
D) Insert an oropharyngeal airway
E) Intubation and ventilation
6. 3kg term baby born by a spontaneous NVD was noted to have a RR of 80 breaths/min after 3 hrs. He also had
chest indrawing & grunting. Membranes were ruptured 24 hrs before delivery. Mother was healthy. She
attended antenatal care regularly and had an uncomplicated pregnancy with normal uss. Probable diagnosis?
A) Diaphragmatic hernia
B) Congenital pneumonia
C) Meconium aspiration Xd
D) Surfactant deficit lung disease
E) TTN

Jaundice in New-Born
MCQ
SBR
1. A 2 weeks old baby was referred by a midwife for jaundice. Mother complains of inadequate breast milk. Baby
was born at term by NVD. Birth weight - 2.8kg. Immediate postnatal period was uncomplicated on examination
baby is active, well perfused, mildly jaundiced, body weight 2.7kg, what is the next step in management?
A) Admit for phototherapy
B) Perform urine culture
C) Reassure and send home
D) Assess and support breastfeeding.
E) Commerce formula feeding.
2. 3days old baby weighing 2.9kg is noted of having yellowish discolouration in face. Birth weight was 3kg baby
is well and examination is unremarkable. Mother's blood group is a positive. What is the immediate
management option?
A) Give advice on breast feeding
B) Reassure the mother and discharge
C) Send blood for investigation
D) Start intravenous fluid
E) Start phototherapy
3. 3-month-old baby is brought with gradual worsening of jaundice, dark urine and pale stools. Child had a birth
weight 2.6 kg with current weight 3.2kg. He is not on medications examination reveals 1cm hepatomegaly
with mild palpable spleen. USS shows distended gallbladder contractile with meals. S. total bilirubin 155 umol/l
direct bilirubin 70 umol/l AST 65 alt 85 what is the most likely cause for the jaundice?
A) Biliary atresia
B) Choledochal cyst
C) Congenital hypothyroidism
D) Dubin Johnson syndrome
E) Neonatal hepatitis syndrome
4. 8 days old baby presented with dark urine and pale stools and he is having low grade fever, loss of appetite
for 5 days’ duration with mild icterus and pallor. On examination liver is palpated 2cm and spleen is 4cm
enlarged. Pulse rate 110 beats/min and other parameters are normal. What is the most probable diagnosis?
A) Acute viral hepatitis
B) Autoimmune hepatitis
C) G6pd deficiency
D) Congenital infection

168
E) Biliary atresia

169
16. Therapeutics and Toxicology Module – Final MBBS Common MCQs

➢ Therapeutics part has been covered in the relevant modules.

Medicine – Toxicology
MCQ
SBR
1. 34 yr old farmer was poisoned with organophosphate is treating with i/v atropine and pralidoxime. On the 2
day he was flushed. He has fever, PR-120 and BP-160/100 mm Which of the following is the best treatment of
choice?
A) IV fluids
B) Stop pralidoxime
C) CPAP
D) Stop atropine
E) Give metronidazole
2. 14 yr old boy after 10hr history of 6 weeds of oleander poisoning presented to casualty ward. He is conscious
and rational. C/o - vomiting and abdominal pain. HR -35. What is the most appropriate next step in Mx?
A) Gastric lavage
B) IV atropine
C) IV isoprenaline
D) Give activated charcoal
E) Start temporary cardiac pacemaker
3. 27 yr old male was bitten by an unknown snake at 8 pm in the night. His R/ leg is swollen where the bitten
mark is present. He complains of pain in the leg. He is conscious and rational. PR 90 , BP - 110/70 mm Hg. What
is most appropriate the first step in management
A) Administer antivenom immediately
B) NSAIDs for pain
C) Administer tetanus toxoid
D) Treat with oral cloxacillin
E) 20min WBCT
4. A 49-year-old male was found unconscious on road. On arrival his temperature is 37°C, BP 160/90 mmHg. PR-
90 bpm. Cardiac auscultation findings are normal. His RR-20/min, lungs clear. There is no neck stiffness. B/L
pupils are 1mm in diameter, poorly reactive to light reflex and fundus is difficult to examine B/L'Lower limb
reflexes are normal except B/L plantar responses, which are extensors. What is the most likely diagnosis?
A) Snake bite envenomation
B) Organophosphate poisoning
C) Pontine haemorrhage
D) Myasthenic crisis
E) Botulinum toxicity
5. A 34-year-old farmer was poisoned with organophosphate and is on treatment with IV atropine and
pralidoxime. On the 2 day he was flushed & agitated. He had dilated pupils,PR120 bpm and BP-160 mmHg.
Which of the following is the best treatment option?
A) IV fluid
B) Stop pralidoxime
170
C) Connect to CPAP
D) Stop atropine
E) Give metronidazole
6. In 18-year, old girl presented following ingestion of 20 tablets of paracetamol, after 3 hours she had recurrent
episodes of vomiting. Her weight is 45kg. What is the most appropriate management?
A) Gastric lavage
B) Observe without any intervention
C) Check plasma paracetamol level
D) Give NAC
E) Give methionine
7. 55-year-old male presented to the medical ward after being bitten by an unidentified snake. He was having
right leg swelling up to the knee joint and blackish discoloration at ankle joint with an oozing puncture mark.
No other physical signs present Clotting test is normal. What would be the immediate decision?
A) Antivenom and surgical referral
B) Surgical referral
C) Limb elevation & antibiotic
D) Antibiotics, antivenom and surgical referral
E) Antibiotics and antivenom
8. A 30-year-old woman with self-ingestion of poison 2 hours ago. On examination, GCS- 12/15, BP-90/50 mmHg
and constituted pruritus, bilateral crepts. What is the best management option?
A) Gastric lavage.
B) Infusion of 250 ml of 0.9% saline over 20 minutes.
C) Endotracheal intubation.
D) Bolus of 3 mg atropine.
E) Transfer to ICU.
9. 25 years old male presents with fever & muscle pain. He had been at his friend's birthday party till late night.
His relatives complain that he seems to see animals & people who do not exist. BP is 150/80, PR = 126/min.
What could be the cause?
A) Heroin toxicity
B) Amphetamine toxicity
C) Codeine toxicity
D) Tramadol toxicity
E) Amitriptyline toxicity
10. A 30 years old farmer admitted following snake bite. There is no bleeding manifestations. No local
implantation, but he is drowsy, GCS- 14/15, BP- 120/80 mmHg, PR- 80 bpm. Antivenom was given successfully
and uncomplicatedly After one hour patient become drowsier. Respiratory rate is 6/min, BP- 80/60 mmHg.
What is the most appropriate next step of management?
A) Normal saline rapid bolus.
B) Continuous Positive Airway Pressure.
C) Repeat antivenom.
D) IM adrenaline
E) Intubation & ventilation
11. 30-year-old women presents to emergency unit with a GCS of 12/15, pinpoint pupils and blood pressure of
70/50. On auscultation she had basal crepitations in both lungs. What might be substance she has taken?
A) Clonazepam
B) Heroin
C) Ketamine
171
D) Lorazepam
E) Organophosphate
12. A 24 yr old woman found unconscious in the bed. RR 8/minOR 80 bpm. Her pupils were constricted. Best Rx
option for her
A) Atropine
B) Flumazenil
C) Methadone
D) Naloxone
E) Naltrexone
13. 52-year-old farmer presents with unknown snake bite while going to the paddy field. He has progressive
slurring of speech and bilateral ptosis. There's a fang mark on his left ankle. His urine output is satisfactory.
Urinalysis showed 12-20 on moderate field. What can be the snake?
A) Common krait
B) Cobra
C) Ceylon krait
D) Hump nosed viper
E) Russell's viper
14. Middle age man presented with one hour after a snake bite. He has Difficulty in breathing and saturation is
normal. Also Having past history of 2 episodes of anaphylaxis. On examination patient is andous, tachycardia,
tachypnoeic, no local swelling or fang marks, but ptosis is there, WBCT<20minutes what is the best
management option?
A) Give full dose of antivenom
B) Observe without giving antivenom
C) Start ventilation
D) Give antivenom with neostigmine
E) Give half dose of antivenom
15. 30 years old male presented with an unknown snake bite. His night leg is swollen and also he is having signs
and symptoms of acute renal failure. Whole blood clotting time is normal. What is the most
A) Cobra
B) Russell's viper
C) Hump nosed viper
D) Common krait
E) Saw scaled viper
16. 20-year-old previously healthy female, presenting to hospital after ingesting an unknown
number of paracetamol tablets. What is the next best management option?
A) Gastric lavage followed by NAC
B) N-acetyl cysteine
C) Active charcoal
D) Methionine
E) Wait for one more hour and do according to nomogram
17. 18-year-old man presented following a hump nosed viper bite. No systemic envenomation
features noted. Only two fang marks are present. What is the next step in management?
A) Arrange WBCT
B) Arrange renal function tests
C) Start antivenom serum
D) Monitor for 24 hours
E) Haemodialysis
172

You might also like